Skip to main content

WDS 101: 'Kha-vaigunya/Srotas (ख-वैगुण्य/स्रोतस्) by Vaidyaraja Subhash Sharma, Prof. S. K. Khandal, Prof. Gurdeep Singh, Prof. L. K. Dwivedi, Dr. Dinesh Katoch, Dr. Sanjay Chhajed, Prof. Arun Rathi, Prof. Giriraj Sharma, Dr. Pawan Madaan, Vd. Raghuram Bhatta, Dr. B. K. Mishra, Vd. Atul Kale, Dr. Bharat Padhar, Prof. Mrinal Tiwari, Vd. Ashok Rathod, Dr. Sukhbeer Soni, Dr. Divyesh Desai, Dr. Bhadresh Nayak, Vd. Radheshyam Soni, Prof. Madhav Diggavi, Prof. Mamata Bhagwat, Dr. Satish Solanki, Dr. Sanjay Khedkar, Vd. Prajakta Tomar, Dr. Shashi Jindal, Dr. Sumeet Arora & Others.

[12/30, 2:58 PM] Dr. Pawan Madan:

 चरण स्पर्श गुरु जी।

तीन प्रकार के हेतु
,,,दोष प्रकोपक 
,,,धातु दुष्टि कारक
,,,ख वैगुण्य कारक

ये तीनों मिल कर व्याधि की उत्त्पत्ति का निश्चय कर देते हैं।

कई बार धातु वैगुण्य कारक हेतु बहुत पहले से कार्य कर रहे होते हैं फिर बाद में कभी भी जब दोष प्रकोप होता है तो एक दम से व्याधि प्रकट हो जाती हैं।

गुरु जी क्या खवैगुण्य कारण व धातु दुष्टि कारण एक ही नही हैं, क्योंकि खवैगुण्य भी धातुओं में ही होता है ?

🙏🙏
 ख-वैगुण्य कारक  हेतु से धातुदुष्टि भी होना आवश्यक नहीं है🙏🏼

[12/30, 5:16 PM] Vaidyaraj Subhash Sharma: 

*शुभ सन्ध्या सहित नमस्कार पवन जी, हेतु केवल तीन ही कार्य करेंगें, दोषों को कुपित अथवा वृद्ध करेंगे जैसे प्रमेह में 
'बहुद्रवः श्लेष्मा दोषविशेषः' 
च नि 4/6 
हेतु से कफ की वृद्धि, 
तृष्णा रोग में 
'पित्तानिलौ प्रवृद्धौ सौम्यान्धातूंश्च शोषयतः' 
च चि 22/5 
पित्त और वात मिल कर तृष्णा, 
गुल्म में वात वर्धक हेतु देखिये  'रूक्षान्नपानैरतिसेवितैर्वा शोकेन मिथ्याप्रतिकर्मणा वा, विचेष्टितैर्वा विषमातिमात्रैः कोष्ठे प्रकोपं समुपैति वायुः'
च चि 5/5 
ये सीधा ही वात दोष प्रकोपक है।*

*अगर वातावरण में जैसे ग्रीष्म ऋतु में  उष्णता अधिक है अधिक धूप में रहने से हेतु आतप, खवैगुण्य  को अतिशीघ्र गति से कर के रक्तपित्त कर देता है, कहीं construction site पर जाते ही dust अर्थात रज खवैगुण्य प्राणवाही स्रोतस में कर के श्वास रोग की उत्पत्ति का कारक बन जाता है।*

*हेतु जो तीसरा कार्य करते हैं वो है धातु क्षय या धातुओं की शिथिलता जिस से अग्रिम धातुओं का पोषण नही हो पाता। ये हेतु मिल कर जितने अधिक होगें, बलवान होंगे अथवा दीर्घकाल तक इनका सेवन किया जायेगा तो उतनी ही दोषों की स्थिति त्रिदोषज अथवा सन्निपातिक होगी।*

*आपके प्रश्न ने उलझा दिया कि जिज्ञासा है क्या ? *

[12/30, 5:32 PM] Dr. BK Mishra: 

🙏🏼उत्तम विशद विश्लेषण !💐

[12/30, 8:25 PM] Dr. Pawan Madan: 

उदाहरण से इसे और भी समझना होगा ।

[12/30, 8:35 PM] Dr. BK Mishra: 

अतिसाहस या व्यायाम से उरक्षत, रज-धूम-शीत से प्राणवहस्रोतस में रचनात्मक विकृति (ख वैगुण्य) होता है। किन्तु तत्काल कोई धातुदुष्टि नहीं होती। 
प्राणाचार्य श्री सुभाष जी ने पूर्व में विस्तृत विवेचन किया है..

[12/30, 9:08 PM] Dr. Pawan Madan:

🙏🙏🙏

गुरु जी !
जिज्ञासा ये है के धातुदुष्टि हेतु व खवैगुण्य हेतु दोनो समान नहीं होंगे क्या ?

[12/30, 9:10 PM] Dr. BK Mishra: 

आवश्यक नही आचार्य !

[12/30, 9:16 PM] Dr. Pawan Madan: 🙏🙏

याने जरुरी नही के खवैगुण्य से 
धातुदुष्टि हो
या
खवैगुण्य मे जरुर anatomical changes हों, बिना धातुदुष्टि के भी ख वैगुण्य हो सकता है
🤔

[12/30, 9:23 PM] Dr. D. C. Katoch sir: 

Kha Vaigunya (imbalance or loss of akashiye bhav) structural aur functional dono prakar ka ho sakata hai. Functional disintegrity leads to structural disintegrity and vice verse.  Coronary artery spasm is a functional kha vaigunya, whereas appearance of atheromatous plaques is structural kha vaigunya.

[12/30, 9:32 PM] Dr. Pawan Madan: 

Pranaam sir.

Thank you very much sir.
💐🙏💐🙏💐

And then this functional kha vaigunya may be acute or chronic / temporary or permanant.

🙏🙏

[12/30, 9:46 PM] Dr. BK Mishra: 

क्षमा चाहता हूं, वस्तुतः उक्त जिज्ञासा आदरणीय   सुभाष जी से की गई थी,  मैंने अनावश्यक हस्तक्षेप  किया🙏🏼

[12/30, 9:58 PM] Dr. Pawan Madan: 

आपने सही से उत्तर दिया। 
और जिज्ञासा का उत्तर कोई भी दे सकता है सर।
इसमें क्षमा वाली तो कोई बात ही नहीं है।
🙏

[12/30, 10:01 PM] Dr. BK Mishra:
 🙏🏼🌹

[12/30, 10:01 PM] Dr. Surendra A. Soni: 

My reply to your query in bracket.

चरण स्पर्श गुरु जी।

तीन प्रकार के हेतु
,,,दोष प्रकोपक 
,,,धातुदुष्टि कारक

*(there is no description of separate dhatu prakop nidanas. Dosha prakopak and srotodushti nidanas are also responsible for dhatu dushti.)*

,,,खवैगुण्य कारक

ये तीनों मिल कर व्याधि की उत्त्पत्ती का निश्चय कर देते हैं।

*( No, these are two only.)

&*

कई बार धातु वैगुण्य कारक हेतु बहुत पहले से कार्य कर रहे होते हैं ।

*(This is dhatu shaithilya and dushti caused by above 2 and ignorance of hitahar.)*

 फिर बाद में कभी भी जब दोष प्रकोप होता है तो एक दम से व्याधि प्रकट हो जाती हैं।

*(Because of decrease in vyadhi kshamatva.)*

गुरु जी क्या खवैगुण्य कारण व धातु दुष्टि कारण एक ही नही हैं, क्योंकि खवैगुण्य भी  धातुओं में ही होता है ?

🙏🙏

*(Right Sir ! Kha-vaigunya term generally used for srotodushti.

*(Please note following instructions if acharya charak.)*
👇🏻👇🏻👇🏻👇🏻👇🏻

*प्रदुष्टानां तु खल्वेषां रसादिवहस्रोतसां विज्ञानान्युक्तानि विविधाशितपीतीये; यान्येव हि धातूनां प्रदोषविज्ञानानि तान्येव यथास्वं प्रदुष्टानां धातुस्रोतसाम् ।*

Ch. Vi. -5/8

Acharya Mishra ji !
Acharya Pawan ji !

🙏🏻🌻

[12/30, 10:10 PM] Prof. Giriraj Sharma: 

मुलातखादान्तर देहे,,,,,
ख शब्द स्रोतस के परिपेक्ष्य में अगर चिंतन करे, ख के गुण, कर्म में होने वाली विकृति विशेष संग विमार्ग गमन आदि से संचय आदि अवस्था जिनमे दोष दुष्य समुर्छना के पूर्व एवं पश्चात जो क्रियात्मकविकृति, एवं रचनात्मक विकृति होती है वह धातु दृष्टि एवं बिना धातु दृष्टि दोनो ही प्रतीत होती है,,,,
आलस्य नयन स्त्राव जृम्भा जैसे लक्षण भी ख़ वैगुण्य है ।।
स्वेदाविरोध संताप  भी ख वैगुण्य ही है ।।
सुधार अपेक्षित विमर्श
🙏🏼🙏🏼🙏🏼🌹🙏🏼🙏🏼🙏🏼

[12/30, 10:17 PM] Dr. D. C. Katoch sir: 

Kha Vaigunya Srotodushti se bhinna poorvavastha hai aur Srotodushti dosh-dusyasammoorchhanaottar parinaam hai.

[12/30, 10:18 PM] Dr. BK Mishra: ✅✅🙏🏼🌹


[12/30, 10:19 PM] +91 89209 00968: 

Khavaigunya in srotas relates with dhatvagni mandya- as a result of dhatvagnimandhya poshak ras is not able to do poshan at particular sthan of a srotas, As and when predisposing factors like mithya aahara vihaara results into dosh prakopa - sthan sansraya of that dushit dosha happens only at the site of khavaigunya - although the srotas is same but due to presence of khavaigunya  at specific sthan only manifestation of disease is seen.
For example - dusht kapha and vata in artava vaha srotas after sanyog with khavaigunya and sthan sanshay as per the site we see different manifestations like - in some ovarian cyst, in some endometriosis, in some endometritis and in some patients uterine fibroids.
Although the dushit dosha and srotas are same.

[12/30, 10:20 PM] Prof. Giriraj Sharma: 

मुलातखादान्तर देहे स्रोतसम इति विज्ञेय,,,,,
दोषदुष्य समुर्छना पूर्व एवं पश्च्यात पूर्वरूप एवं रूप या लिंग का भेद मात्र है जिसमे ख वैगुण्य धातु दुष्टि एवं बिना धातु दुष्टि दोनो में मिल सकते है ।
जिन्हें पूर्वरूप एवं लक्षण के रूप में परिलक्षित किया गया है, मूलतया दोनो है ख वैगुण्य ही,,,,,
ऐसा मेरा मत है 
🌹🌹🌹🙏🏼🌹🌹🌹

[12/30, 10:21 PM] Dr. Surendra A. Soni: 

जी । अत्युत्तम विवेचना । 
मेरा मन्तव्य धातुदुष्टि पर अधिक था जो मूल प्रश्न था ।
आपका सन्दर्भ अधिक सूक्ष्म है और वह थोड़ी अलग बात है ।

Respected Katoch Sir !
🙏🏻🌻

[12/30, 10:21 PM] Dr. Ramesh Kumar Pandey: 

अति उपयोगी जानकारी प्रदान करने हेतु बहुत बहुत धन्यवाद👏👏

[12/30, 10:36 PM] Dr. BK Mishra: 

फिर भी ख-वैगुण्य आयुर्वेदीय विकृतिविज्ञान का महत्वपूर्ण बिन्दु है। 🙏🏼

[12/30, 10:38 PM] Vd. V. B. Pandey Basti(U. P. ): 

हम तो इसे इस तरह समझते है कि चिकित्सा हेतु खवैगुण जनित दोष और प्रकूपित दोनों का समाधान करना चाहिए। प्रत्येक व्याधि का अपना एक खवैगुण होता है जैसे ज्वर का पित ।यदि कफज ज्वर भी होगा फिर भी पित्तशमन करना ही होगा।

[12/30, 10:42 PM] Dr. BK Mishra: 

ख-वैगुण्य निदान से होता है, किसी दोष से नहीं, कृपया प्रो ध्यानी जी की उक्त तालिका अवलोकन करें..🙏🏼

[12/30, 10:46 PM] Dr. Pawan Madan: 

Thank you soni sir.

Ji. The division is only for our understanding but there can be overlapping.

Ji dhatu shaithilya..🙏

My query was there as ....many months back Resp Katoch has cleared that the khavigunya always occurs in dhatus as well as srotodushti is also not without dhatus.

Thanks for the reference.

🙏🙏

[12/30, 10:46 PM] Vd. V. B. Pandey Basti(U. P. ): 

लेकिन निदान भी तो उत्तरोत्तर दोष ही बढ़ाएगा।
Mishra Sir !

[12/30, 10:49 PM] Dr. BK Mishra: 

उत्तरोत्तर तो निदान-दोष-दूष्यात्मक पूरी सम्प्राप्ति घटित होकर व्याधि उत्पत्ति भी हो जायेगी...🙏🏼🙏🏼

[12/30, 10:51 PM] Vd. V. B. Pandey Basti(U.P. ): 🙏

[12/30, 10:57 PM] Dr. Surendra A. Soni: 

ख vaiguny in srotas क्योंकि वो ही aakaashiya है ।  धातुदुष्टि धातुओं में ।
Pawan ji !

[12/30, 11:00 PM] Dr. Pawan Madan: 

शायद आप खवैगुण्य ब हेतु को एक समझ रहे हैं।

[12/30, 11:02 PM] Dr. D. C. Katoch sir: 

"Srortasaam Prakritibhootatvat Na Vikaarai Upsrijayate Shariram" means  kha vigunata is the precursor of vikriti.

Dr. Pawan !

[12/30, 11:03 PM] Dr. Pawan Madan: 

खवैगुण्य ,, हेतु जनित एक अवस्था है
खवैगूण्य,, परीणाम है l

[12/30, 11:04 PM] Dr. Pawan Madan: 🙏🙏🙏

स्रोतस भी धातुओं में ही होते हैं
बिना धातु के स्रोतस नहीं l

[12/30, 11:06 PM] Dr. BK Mishra: 

परिणाम दोषदूष्य सम्मूर्छना जनित स्रोतोदुष्टि है...


[12/30, 11:08 PM] Dr. Pawan Madan: 

जी मेर अभिप्राय था के हेतु के बाद की एक अवस्था खवैगुण्य है, इसलिये परीणाम कहा
सिर्फ हेतु से होने वाला आगे का परीणाम
बस समझने के लिये

[12/30, 11:13 PM] Dr. D. C. Katoch sir: 

Aetiological factors of any disease mentioned in the literature are of four types - 
*khavaigunyakar hetu, *doshaprakopak hetu, *agnidushtikar hetu and *dhatupradooshak hetu. 
Combined effect of all kinds of hetus leads to dosh-dusyasammoorchchana resulting into any kind of srotodusti and that kind of srotodusti produces pratyatm lakshan of the disease to be manifested.

[12/30, 11:13 PM] Dr. Pawan Madan:

 जी
पर एक दूसरे के बिना इनका अस्तित्व भी नहीं


[12/30, 11:14 PM] Dr. Surendra A. Soni: 

Katoch Sir👌🏻👍🏻

[12/30, 11:14 PM] Dr. Pawan Madan: 

जी सर
आपसे यही सीखा है, शुरु से ही।
🙏🙏

[12/30, 11:16 PM] Dr. Pawan Madan: 

Khavaigunya is the predisposition or proneness of a particular srotas or dhaatu where the probability of srotodushti is more due to the already exposure of the disease causing hetus which may be a prolonged or acute situation.

Perhaps this is what I have understood.

🙏🙏

[12/30, 11:20 PM] Dr. Pawan Madan:

 Ji.

Khavaigunya is the poorva avastha of srotidushti.

🙏

[12/30, 11:21 PM] Dr. D. C. Katoch sir: 

Swahetujanit kha vaigunya vastutah vyadhi ki aadi (beginning) avastha hai, anta (outcome/result) nahin.

[12/30, 11:23 PM] Dr. Pawan Madan: 

Ji sir.
I meant the same. Written Parinaam...just to show that it is the result of hetus.
🙏

[12/30, 11:25 PM] Dr. D. C. Katoch sir: 

Right ✔

[12/31, 12:26 AM] Dr. Mrunal Tiwari Sir: 

व्याधी निर्माण प्रक्रिया में सर्व प्रथम खवैगुण्य उपस्थित रहना जरुरी है। खवैगुण्य अपने कारणों की वजह से होता है। खवैगुण्य से उस स्थान पर रचनात्मक, क्रियात्मक बिगाड होता है।आगे चलकर अगर दोषदुष्टी होती है -(अपने अलग कारण से  प्रकुपित होते है) ,,तो वे दुष्ट प्रकुपित दोष  उस स्थान पर जहाँ खवैगुण्य है वहां पर अन्योन्य संमूर्च्छना होने के बाद स्थित होते है और व्याधी उत्पन्न हो जाता है।  चिकित्सात्मक दृष्टिकोण से अगर खवैगुण्य उत्पन्न हुआ है तब उस स्थान का बल वर्धन की चिकित्सा उपयुक्त है।

[12/31, 12:30 AM] Dr. Mrunal Tiwari Sir: 

यह खवैगुण्य व्याधी पुर्व और व्याधी पश्चात भी हो सकता है। चिकित्सा भी तद् अनुरूप करनी चाहिए।

[12/31, 1:15 AM] Vaidyaraj Subhash Sharma: 

*खवैगुण्य रोग उदाहरण और संदर्भ  के साथ ले कर चलते है...*

*'कुपितानां हि दोषाणां शरीरे परिधावताम्, यत्र संगः खवैगुण्याद्व्याधिस्तत्रोपजायते' 

सु सू 25/10 , 

इस पर आचार्य डल्हण की व्याख्या है 'खवैगुण्यात् स्रोतोवैगुण्यादित्यर्थः'*

*आचार्य चक्रपाणि 'खवैगुण्य खवैगुण्यादिति स्रोतोवैगुण्यात् ' 

च चि 15/37 

दोनों आचार्यों के मत से खवैगुण्य स्रोतस की विगुणता है क्योंकि खवैगुण्य स्रोतस में ही होगा जो उनके प्राकृत कर्मों में व्यवधान उत्पन्न करेगा।*

*'मूत्रोत्संग के लक्षणों में भी खवैगुण्य को स्पष्ट किया है
 'खवैगुण्यानिलाक्षेपैः किञ्चिन्मूत्रं च तिष्ठति मणिसन्धौ स्रवेत् पश्चात्तदरुग्वाऽथ चातिरुक् ' 

च सि 9/33 *

*ये खवैगुण्य अर्थात स्रोतस की विगुणता कैसे हो रही है ? इसे हेतु कर रहा है और इसे आरंभ से समझ कर चलें तो पूरा विषय स्पष्ट हो जायेगा कि हेतु खवैगुण्य किन किन प्रक्रियाओं से कर रहा है । ये हेतु अनेक प्रकार के हैं जैसे ...*

*निज - 
जो दोषों को प्रकुपित सीधे ही करते है और 
*आगंतुज- 
जो अभिघात, अभिशाप, अभिचार और अभिषंग से विभिन्न आगन्तुक रोगों को उत्पन्न कर देते हैं।*

*सन्निकृष्ट हेतु - 
जैसे वय, दिन, रात और भोजन के अन्त, मध्य और आरंभ में वात पित्त और कफ का प्रकोप रहता ही है तथा विप्रकृष्ट जैसे हेमन्त में संचित कफ का वसंत में प्रकोप होगा और कफज रोग उत्पन्न करेगा।*

*व्यभिचारी हेतु - 
ये वो निदान है जो दुर्बल होने से पूर्ण सम्प्राप्ति बनाकर रोग उत्पन्न करने में तो असमर्थ है पर लक्षण उत्पन्न कर रोगी को विचलित कर के रखते हैं।*

*प्राधानिक हेतु - 
जो उग्र स्वभाव होने से तुरंत ही दोषो को प्रकुपित कर देते हैं जैसे विष, अति मादक द्रव्य आदि ।*

*उभय हेतु - 
ये हेतु दोष और व्याधि दोनो को उत्पन्न करने की सामर्थ्य रखते हैं जैसे अति उष्ण-तीक्ष्ण-अम्ल विदाही अन्न*

*विभिन्न वायरस, कृमि या जीवाणु*

*इस पूरे प्रकरण में अब देख सकते हैं कि बिना दोषों के चय के भी अचय पूर्वक खवैगुण्य हो रहा है और दोष प्रकोप बाद में होगा ।*

*अब इसे किसी भी रोग के उदाहरण में apply कर के देखते हैं तो हम यहां प्रमेह का उदाहरण ले कर चलेंगे ,*

*'आस्यासुखं स्वप्नसुखं दधीनि ग्राम्यौदकानूपरसाः पयांसि नवान्नपानं गुडवैकृतं च प्रमेहहेतुः कफकृच्च सर्वम्' 
च चि 6/4 
तथा आगे चलें तो 
'जातः प्रमेही मधुमेहिनो वा न साध्य उक्तः स हि बीजदोषात् ये चापि केचित् कुलजा विकारा भवन्ति तांश्च प्रवदन्त्यसाध्यान् ' 
च चि 6/57 
और आगे आचार्य चक्रपाणि का कथन है 
'मेहानामसाध्यताप्रकारान्तरमाह- जात इत्यादि,प्रमेही यः प्रमेहिणो जातः सोऽप्यसाध्यो भवति, अत्रापि हेतुमाह- स हि बीजदोषादिति; प्रमेहारम्भक दोषदुष्टबीजजातप्रमेहित्वात् ।*

*अब इन सब हेतुओं को देखे तो जो 
च चि 6/4 में कफ वर्धक हेतु प्रतीत हो रहे थे वो श्लोक संख्या 57 और चक्रपाणि के अतिरिक्त मत से अब इस प्रकार बन गये, 

1- आहार जन्य - दूध, दही, नया अन्न, गुड़ और अनेक कफ वर्धक हेतु जो हमने सर्वप्रथम लिखे।*

*2- विहार जन्य- सुखप्रद आसनों या शय्या पर आस्या सुख या स्वप्न सुख, अति निद्रा, आलस्यादि।*
*3- मानस भाव जन्य- अति हर्ष- उल्लास*
*4- बीज दोष जन्य- जिसे हम सहज प्रमेह कह सकते हैं।*
*5- कुल में रहने वाली प्रवृत्ति- कुलज प्रमेह*

*अब ये उभय हेतु बन गये हैं जो दोष तथा व्याधि दोनों को भी उत्पन्न करने का सामर्थ्य रखते हैं।*

*इन हेतुओं से खवैगुण्य कैसे हो रहा है ? कफ के अंश देखें तो कफ 
'गुरुशीतमृदुस्निग्धमधुरस्थिरपिच्छिलाः'च सू 1/59-61 हैं। 
कफ वर्धक आहार का अति सेवन पर इस कफ का कोई एक अंश कुपित हो कर मेदोवाही स्रोतस में खवैगुण्य उत्पन्न करेगा जो प्रमेह का कारण बनेगा क्योंकि अगर कफ के सारे अंश कुपित हो तो अनेक कफज रोग उस रोगी को हो जायेंगे। प्रमेह में के निदान तो कफवर्धक है फिर समस्त कफज रोगों में यही निदान क्यों नही लिखे ? क्योंकि कुछ हेतु कुछ रोगियों में दोनों कार्य एक साथ कर रहे हैं जो दोष के एक विशिष्ट अंश को प्रकुपित कर, खवैगुण्य भी कर रहा है, एक विशिष्ट धातु जैसे यहां मेद को दुर्बल अथवा शिथिल भी कर रहा है और इसके स्रोतस में दुष्टि भी ला रहा है। दोष दूष्य सम्मूर्छना तो षडक्रिया काल की व्यक्त अवस्था है।*

*ऊपर हेतु देखें तो कुलज और  बीज गत प्रमेह जो धातु शैथिल्य एवं स्रोतस दुष्टि के साथ आता है वो जन्म के कुछ काल के साथ अपने पूर्ण रूप में व्यक्त होने लगता है ।*

*आप अलग अलग व्याधियों के निदान देखिये और चार्ट बनाईये, दोषों की अंशाश कल्पना के अनुसार व्याधि के दूष्य,स्रोतस में जायें तो स्वतः ही विषय स्पष्ट होने लगेगा पर उस से पहले आवश्यक ये भी है कि अनेक व्याधियों के पूर्वरूप नही लिखे पर वो रोगियों में होते है तो वह हमें स्वः निजी बुद्धि बल से बनाने चाहिये तब खवैगुण्य और अधिक स्पष्ट होता जायेगा।*

[12/31, 5:25 AM] Dr Mansukh R Mangukiya Gujarat: 

🙏 गुरूवर 🙏
अब खवैगुण्य थोड़ा-बहुत समझा हूँ।
🙏 धन्यवाद गुरुवर 🙏

[12/31, 5:26 AM] Vaidya Sanjay P. Chhajed: 

व्याधी पश्चात होनेवाले खवैगुण्य को उपद्रव या उदर्क का असमवायी कारण मान सकते हैं। पूर्व मे घटीत संप्राप्ति में कोई योगदान न देकर नये सिरे से अलग व्याधि के उत्पत्ति के कारण है। 
उदाहरण के रूप में कोविड के पश्चात फुफ्फुसीय खवैगुण्य के कारण श्वासकसष्टता l

[12/31, 6:15 AM] +91 89209 00968: 

🙏🏻
एक ही स्रोतस मे भिन्न विकार मे खवैगुणय को कैसे समझेंगे ?


[12/31, 8:40 AM] Dr. BK Mishra:

 वहां दोष तथा दूश्यों की भिन्नता कारण होता है।
व्याधि की पूर्ण सम्प्राप्ति  निदान-दोष-दूष्य इन तीनों के संयोग विशेष पर निर्भर है🙏🏼

[12/31, 8:48 AM] Dr. D. C. Katoch sir: 

Kha Vaigunya ki bhinnata apabhrist mahabhoot aur uske gunon par nirbhar karati hai.
 Aur apabhrist mahabhoot ki sthiti bhootagni  vyapaar aur aahar ras ke gunon ke anusaar hoti hai.

[12/31, 8:57 AM] Dr. BK Mishra: 

खवैगुण्यकारक निदानांश प्रायः बाह्य होता है, भूताग्नि देह का आंतरिक भाग है, दोनों की स्थिति भिन्न है 🙏🏼

[12/31, 8:58 AM] Vaidyaraj Subhash Sharma: 

*पहले इसको इस प्रकार समझें, शीतल एवं प्रदुषित जल = cold drinks में बाजार से ice मिला कर सेवन की तो शीत गुण ने वात और कफ को प्रकुपित किया, रसवाही- प्राणवाही स्रोतस में खवैगुण्य किया जो आगे चल कर वातज कास में भी परिणित हो सकता है। इसी जल के शीत गुण नें तथा अचय पूर्वक प्रदुषित जल ने वात को प्रकुपित किया तथा शरीर की जलीय धातु, अम्बुवाही स्रोतस और पुरीषवाही स्रोतस में खवैगुण्य किया और अतिसार भी उत्पन्न किया।* 

*मल स्निग्ध आ रहा है और रोगी को शीतता प्रतीत हो रही है तो वात प्रधान कफज व्याधियां भी इसी प्रकार होती हैं और अनुबंधी भी।एक ही हेतु शीत द्रव्य वात का चल, कफ का स्निग्ध, शीत और पिच्छिल गुण भी बढ़ा सकता है।*

*इस शीत गुण ने ही अम्बुवाही स्रोतस, प्राणवाही स्रोतस दोनो स्थान में खवैगुण्य कर के श्वास रोग, pneumonia और वात को कुपित कर पक्वाश्य में अतिसार भी किया ।*

[12/31, 9:00 AM] Dr. D. C. Katoch sir: 

Bilkul theek, vahi nidanansh bhoot vaigunya utpann kar ke Kha Vaigunya ko prakat karata hai.

[12/31, 9:00 AM] Dr. BK Mishra: 

🙏🏼सादर प्रणाम आदरणीय।   अज्ञानतिमिर का आपने ज्ञानांजन से प्रशमन कर दिया है। 💐🙏🏼

[12/31, 9:13 AM] Dr. D. C. Katoch sir: 

Sheet Gun ka kaisa prabhav hoga yeh sharir mein  bhooton ki sthiti (prakriti) par nirbhar karata hai aur is sthiti ka vichalit hona hi Kha Vaigunya hai.

[12/31, 9:27 AM] Vaidyaraj Subhash Sharma: 

*इस प्रकार तो फिर हम देश, काल, संयोग, वीर्य, प्रमाण, अतियोग और सर्वाधिक महत्वपूर्ण अग्नि 
'आयुर्वर्णो बलं स्वास्थ्यमुत्साहोपचयौ प्रभा।ओजस्तेजोऽग्नय: प्राणाश्चोक्ता देहाग्निहेतुका:॥*
 च सू 15/3
*अर्थात आयु वर्ण बल स्वास्थ्य उत्साह शरीर का उपचय प्रभा ओज तेज सभी अग्नियां तथा प्राण ये सभी अग्नि अर्थात जाठराग्नि के अधीन है।*
*यदन्नं देहधात्वोजोबलवर्णादिपोषकम्।*
*तत्राग्निर्हेतुराहारान्न ह्यपक्वाद् रसादय॥* 
च चि 15/5
*अर्थात जो आहार देह धातु ओज बल वर्णादि की पुष्टि करता है उसका आधार जाठराग्नि है, अग्नि में प्रवेश कर जायेंगे तोविषय और विस्तृत हो जायेगा । 10 रोगियों ने ये जल सेवन किया पर एक को अतिसार, एक को कास और एक को श्वास हुआ और खवैगुण्य कैसे हुआ ? 
इसे सरलता से जान लें तो विषष और स्पष्ट हो जायेगा । बस इतना सा निवेदन है ! 🌹❤️🙏*

[12/31, 9:33 AM] Prof. Lakshmikant Dwivedi Sir: 

Ji  Astu kaaya =kaayastha Agni,"Yannshe niyatam nasho,yasmin tishthati"
Deha dhatvagni vigyaniyam ,
"Rogasarvepi mandegnau"🙏

[12/31, 9:36 AM] Prof. Lakshmikant Dwivedi Sir: 

🙏☺️
shesh sab prakrati karti hai.
Cikitsak prakrati ko sahayak hai.पराश्रयी नहीं बनाता।
चिकित्सा के नाम पर।☺️☺️

[12/31, 9:42 AM] Vaidyaraj Subhash Sharma: 

*🙏🙏 इस समय हमें विभिन्न रोग और उनका खवैगुण्य कहां- किस प्रकार हुआ ये जानना है, अन्यथा विषय कहीं और चला जायेगा। रोगों के नाम हो और खवैगुण्य पर रहे तो आगे सभी के लिये सरल होगा अन्यथा बार बार ये विषय आते रहेंगे ... 

नमो नमः* 🌹🙏

[12/31, 9:42 AM] +91 95613 91758: 

Vishisht Nidanansh dwara alag alag srotas ya srotas vishesh  me  kha vaigunya nirman ki prakriya me kuchh vishisht lakshano ki utpatti bhi hoti hogi...isse  bhi aage ke step me yadi Vaat ke sandarbh me vichar kre to kha vaigunya me vaat dwara sthan sanshray prapt karne par bhi vaat vyadhi ke purva rup avyakta rahte hai ? 
Guru Jano se margdarshan hetu jigyasa🙏

[12/31, 9:48 AM] Vaidyaraj Subhash Sharma: 

*आयुर्वेद को सरल बना लीजिये जैसे 80 ही नही अनंत वात व्याधियां हैं पर दो ही लक्षण प्रधान है या चल गुण या स्तम्भ, खवैगुण्य तक पहुंच जायेंगे।*

*मिलेंगे ये ही ।*

[12/31, 9:53 AM] Prof. Giriraj Sharma: 

*मतांतर ख वैगुण्य*

*ख वैगुण्य, ख आदि पंचमहाभूतों के प्राकृतिक कर्मो में विकृति अर्थात भुताग्नि जन्य विकृति,,,*
*विभजति पचति क्लेदयन्ति सहन्ति विवर्धति विवर्धित,,,  भूताग्नि के परिपेक्ष्य में आहारविहार में विविक्तता चेष्टा पक्ति संताप लघुता द्रवता गुरुता एवं मूर्तता का हीन या अति होना जिससे पंचमहाभौतिक आहार द्रव्य शरीर मे सम्यक स्थिति को प्राप्त नही कर पाते,,,, वह अवस्था ख वैगुण्य है ।*
🌹🌹🌹🙏🏼🌹🌹🙏🏼🌹🌹

[12/31, 9:59 AM] Dr. Surendra A. Soni: 

Valid point acharya ji !

👌🏻🌻🙏🏻

[12/31, 10:01 AM] Dr. Bhadresh Naik Gujarat: 

Respected sir !
Kindly inform how empty space occurred in strotus
At micro and cellular level of tissue
Small capillaries
Strotus leads to vyaghi.....

[12/31, 10:04 AM] Dr. Balraj Singh: 

Respected Gurujan and aacharygn. 

What we say Kh- vagunya in easy meaning🙏🏻

[12/31, 10:05 AM] Dr. Bhadresh Naik Gujarat: 

Nidan sannikrust
Viprakust
Sthansa sansray
Khavagruay
Agnibal
Genetic factors
Desh kal vay
Avyayam are basic factors
How can we interpret the shadkal of vyaghi ?

[12/31, 10:06 AM] Dr. Surendra A. Soni: 
[12/31, 09:53] Prof Giriraj Sharma: *मतांतर ख वैगुण्य*

*ख वैगुण्य, ख आदि पंचमहाभूतों के प्राकृतिक कर्मो में विकृति अर्थात भुताग्नि जन्य विकृति,,,*
*विभजति पचति क्लेदयन्ति सहन्ति विवर्धति विवर्धित,,,  भुताग्नि के परिपेक्ष्य में आहारविहार में विविक्तता चेष्टा पक्ति संताप लघुता द्रवता गुरुता एवं मूर्तता का हीन या अति होना जिससे पंचमहाभोतिक आहार द्रव्य शरीर मे सम्यक स्थिति को प्राप्त नही कर पाते,,,, वह अवस्था ख वैगुण्य है ।*
🌹🌹🌹🙏🏼🌹🌹🙏🏼🌹🌹

👆🏻👆🏻👆🏻👆🏻👆🏻

Above post covers answer of your questions....

It's pleasure to see you active.

Dr. Chouhan ji !

[12/31, 10:07 AM] Dr. Surendra A. Soni: 

शरीर अवकाश वैषम्य !

Dr Balraj !

[12/31, 10:07 AM] Dr. Balraj Singh: 🙏🏻

[12/31, 10:08 AM] Dr. Mrunal Tiwari Sir: 

Kha vaigunya is a seat / site which is weak and also a place  where dushta dosha can reside and give rise to disease.

[12/31, 10:08 AM] Dr. Bhadresh Naik Gujarat: 

Stress and physiological factors are important factor.

[12/31, 10:10 AM] Dr. Bhadresh Naik Gujarat: 

How it occurs
Embolism may considered ?

[12/31, 10:12 AM] Dr. Surendra A. Soni:

 👍🏻yes it's result of khavaigunya.

See post of giriraj ji and other previous posts.

[12/31, 10:14 AM] वैद्य सुखबीर सोनी: 

The place where the embolism creat the problem is Kha vaigunya.

[12/31, 10:16 AM] Dr. Surendra A. Soni: 

घटना स्थल😃

[12/31, 10:19 AM] वैद्य सुखबीर सोनी:

 Yes घटना स्थल (स्थान संश्रय) पर खाली जगह is right word for kha Vaigunya 😀😀

[12/31, 10:26 AM] Dr. BK Mishra:

 ख वैगुण्य शब्द में *ख* से तात्पर्य *विभिन्न स्रोतस*  ही हैं, आचार्य ध्यानी जी की विवेचना अधिक व्यावहारिक है🙏🏼

[12/31, 10:27 AM] +91 89014 54426:

Means dosh, dhatu n mal ka equilibrium disturb hone se kha vaigunaya ban sakta hai???

[12/31, 10:29 AM] Dr. Pawan Madan: 

प्रणाम व चरण स्पर्श गुरु जी।

खवैगूण्य = स्रोतो वैगुण्य

प्रमेह के संदर्भ में कफ एक गुणांश की वृधि हो कर वह मेदोवह स्रोतस की दुश्टि कर रहा है, क्या आपका ये मन्तव्य है?

🙏🙏🙏

[12/31, 10:31 AM] Vaidyaraj Subhash Sharma: 

*जी हैं *

[12/31, 10:31 AM] Dr. BK Mishra: 

आचार्य चरक के *यत्र सङ्गः ख वैगुण्याद्* से भी स्रोतस की भौतिक रचनात्मक विकृति ही लक्षित होती है..

[12/31/2021, 10:36 AM] Dr. Pawan Madan: 

Good mng sir.

Need to understand this more...

[12/31/2021, 10:36 AM] Prof. Giriraj Sharma: 

नमस्कार आचार्य वर,,,
स्रोतस ही ख है 
खादान्तरमुलदेहे स्रोतस विज्ञेय,,,,
ख वैगुण्यात स्रोतो वैगुण्यात,,,,
यह पूर्व में उल्लेख कर दिया,,,,
इन्ही स्रोतस समुदाय शरीर में यह सब व्यापार होता है ।
🙏🏼🙏🏼🌹🙏🏼🙏🏼

[12/31/2021, 10:37 AM] Dr. Pawan Madan: 

अर्थात एक ही गुण अलग अलग स्रोत्सो में अलग अलग प्रकार का खवैगूण्य करता हुआ अलग अलग लक्षण उत्पन्न करता है,,



Is the altered state of bhootas ... caused by different hetus....?

Srotas are made of dhaatus, dhatus made of bhootas....

🙏

[12/31/2021, 10:47 AM] Vd. Atul J. Kale: 

गुरुवर आचार्य सुभाषसर साष्टांग नमन 🙏🏻🙏🏻🙏🏻🙏🏻🌹🌹

ये खवैगुण्य कितने प्रकार से हो सकता है ये भी जानना होगा.
धातुशैथिल्य, स्रोतोशैथिल्य, दोषों के विगुण गुणों से स्रोतसों में किस किस प्रकार के लक्षण उत्पन्न होंगे और इन पहलुओं का विचार करके निदानपंचक का चिकित्सा में प्रॅक्टिकली उपयोग कैसे करे ? 
  क्योंकी निदानपंचक की सही सोच कैसी होनी चाहिए......? हर व्याधी में सामान्य रुप से कैसे सोचे और विशेषरुपसे कैसे ? 
गुरुवर कृपया इसपर भी प्रकाश डाले। 😊 🙏🏻🙏🏻🙏🏻

[12/31/2021, 10:51 AM] Vaidya Sanjay P. Chhajed: 

कामिनी जी नमस्कार, बहोत दिनों के बाद आपका दर्शन सुखद है।

समझिए की अन्नवह में ख वैगुण्य उत्पन्न हुआ तो वह मुख से गुदातक अनेक अवयवों में हो सकता है। और यह भी जरूरी नहीं की एक ही कारण हो। एक कालीक अलग अलग हेतुओ द्वारा उसी स्त्रोतस के अलग-अलग भागों में अलग-अलग संप्राप्ति हो सकती है। या एक समान संप्राप्ति द्वारा अलग-अलग अवयवों में भिन्न कालीन अलग अलग या समान स्थान वैगुण्य निर्माण हो सकता है, जो बहुत ही स्पष्ट है।

[12/31/2021, 10:52 AM] +91 89014 54426: 

Srotodushi ke hetu,aahar n vihar jo dosh ke guno ke samanya ho, dhatu vigunit ho, means dosh vridhhi ya dhatuvirodhak swabhav, jaise diva swaap se meda dhatu dusti

[12/31/2021, 10:58 AM] Vd Raghuram Y. S, Banguluru: 

*Kha Vaigunya*

*Good ongoing discussions on this topic*🙏🙏💐

*ADDITIONS From my end (not presentation) 🙏🙏💐*

*It is equally important to look at this topic from* 

👉 *KHA perspective - which is both akasha & srotas* & 
👉 *Vaigunya perspective - which denotes vigunatva - i.e. imbalances of gunas of akashiya tatva.*
✅ *Parinama*
✅ *Prthaktva*
✅ *Samyoga*
✅ *Vibhaga*
are amongst *akashiya gunas*

*Imbalance in mechanisms caused by these gunas will lead to kha vaigunya*

*The Srshti krama starts from Kha (speaking only the chronology of mahabhutas and their entry into other mahabhutas - anupravesha). So all mahabhutas have components of akasha. So the Kha vaigunya can take place in any structure formed by Predominance of any mahabhuta.*

*This is my small understanding & micro presentation*. 

For *magnified version of the same, kindly read this - my writeup / article*👇👇

https://www.easyayurveda.com/2016/05/26/akasha-mahabhuta-qualities-functions-how-to-balance/

( *Note* - The language of the article has been made simple for understanding of common man / students of different geographic regions. A kind request not to consider parts of article as non technical). 

*Humble submission*

*DrRaghuram Y.S.*

🙏🙏💐💐

[12/31/2021, 11:26 AM] वैद्य सुखबीर सोनी: 

ख वैगुण्य में ख शब्द आकाश का पर्यायवाची है लेकिन ये आकाश महाभूत से सम्बंधित नहीं है। 

जैसे आमाशय या कान में खाली स्थान आकाश महाभूत से सम्बंधित है  लेकिन उस स्थान पर दोष दुष्य संमुर्छना नहीं होती और मांस गत व्याधि में आकाश महाभूत नहीं है लेकिन वहां ख वैगुण्य में स्थान संश्रय होकर व्यधि उत्पन्न हो सकती है। 

ख वैगुण्य खाली स्थान मात्र नहीं कमजोर स्थान है l

[12/31/2021, 11:27 AM] वैद्य सुखबीर सोनी: 

ख वैगुण्य को और अच्छे से समझने के लिए निदान पंचक श्रेष्ठ है l

[12/31/2021, 11:29 AM] Dr. Pawan Madan: 

👍👍👍

Weak or predisposed state..

Khavaigunya is the predisposition or proneness of a particular srotas or dhaatu where the probabilty of srotodushti is more due to the already exposure of the disease causing hetus which may be a prolonged or acute state..

[12/31/2021, 11:30 AM] वैद्य सुखबीर सोनी: 

Yes !

[12/31/2021, 11:31 AM] Dr. BK Mishra: 

👍🏼🌹
आकाश महाभूत में तो कोई वैगुण्य हो भी नहीं सकता...

[12/31/2021, 11:35 AM] वैद्य सुखबीर सोनी:

 *क्या व्यधि की उत्पत्ति के लिए ख वैगुण्य होना परम आवश्यक है ?*

[12/31/2021, 11:35 AM] Dr.Raminder kaur: 🙏🏻🙏🏻

[12/31/2021, 11:37 AM] Dr. D. C. Katoch sir: 

Akash ka  pratighat hi to kha vaigunya paida karata hai.

[12/31/2021, 11:42 AM] Dr. D. C. Katoch sir: 

Aise prashanon ka hal nikalane ke liye hi whatsApp group ka avlamban lena padata hai aur phir anekon vicharon aur mantavyon ke saath kai prashanchinha khade ho jate hain.

[12/31/2021, 11:43 AM] वैद्य सुखबीर सोनी: 

प्रतिघात या अप्रतिघात सर !

[12/31/2021, 11:44 AM] Vd. AAkash changole, Amravati: 

अप्रतिघात आकाश का लक्षण है !

Iske विपरीत प्रतिघात yeh विकृत लक्षण है |

[12/31/2021, 11:49 AM] वैद्य सुखबीर सोनी: 

Eg 

*एक व्यक्ति को विटामिन बी12 के कारण Forgetfullness व depression हो गया है। हम यहां पर ख वैगुण्य की स्थिति को कैसे समझ सकते हैं?* 🙏🙏

[12/31/2021, 11:52 AM] Vd Raghuram Y. S, Banguluru: 

I haven't said they are synonymous sir. But while speaking about kha vaigunya how can we leave out origin of kha? We need to understand that root perspective. That's why I hv put my points as additions. 
That is also the reason why I have shared my article to give comprehensiveness to my explanation. 

Having said that I respect your perspective 🙏

[12/31/2021, 11:52 AM] Prof.Vd.Arun Rathi: 

*ख वैगुण्य*:

*अतिप्रवृत्तिः संङ्गोवा सिराणां ग्रन्थयोsपिवा ।*
*विमार्गगमनं चापि स्त्रोतासां दुष्टी लक्षणम् ।।*
च. वि. ५ / २४.

*Four Features of Srotas Morbidtity.*

1. *Increased Flow.*
2. *Obstruction.*
3. *Formation of Nodules.*
4. *Flow of Bodily Fluids or Contents in Wrong or Opposite Directions.*

 *These are the Four Basic संप्राप्ति of "ख वैगुण्य.*

*In ख वैगुण्य, Distortion or Deformation in आकाश and वायु is their.*

[12/31/2021, 11:55 AM] Vd Raghuram Y. S, Banguluru: 

Absolutely yes sir✅✅
That's what I meant in my simple presentation.

I was *about to write from the perspective of sroto dushti lakshanas that you hv mentioned, but you addressed that point*. 

*Key reference to understand this perspective*👌🙏💐

[12/31/2021, 11:57 AM] Dr. BK Mishra: 

आकाश में तो प्रतिघात होता ही नहीं है..
*आकाशस्य अप्रतिघातम्*  
आकाश प्रधान पदार्थ /द्रव्य में ही सम्भव है..

[12/31/2021, 12:05 PM] Prof.Vd.Arun Rathi: 

*In each Cell or Tissue Space is their, means आकाश महाभूत होता है.*

*सो मांसगत व्याधियों मे आकाश महाभूत नही है ये कहना उचित नहीं होगा.*

[12/31/2021, 12:06 PM] Dr. BK Mishra: 

*जी हां*  
 आचार्य चरक के *यत्र सङ्गः ख वैगुण्याद् व्याधिस्तत्रोपजायते* से तो यही प्रमाणित होता है...

[12/31/2021, 12:07 PM] Vd. Yashpalsinh A. Jadeja Gujarat:

 Right sir.. 
ख वैगुण्य = स्रोतस वैगुण्य

आकाश महाभूत में जाने से confusion बढ़ेगी ।🙏🏼

[12/31/2021, 12:07 PM] Vd. Divyesh Desai Surat: 

सर, ख वैगुण्य में 
ख का मतलब स्त्रोतों से है, ओर हरेक स्रोत आकाशिय (अवकाश युक्त)  है।।
Cell (कोष) भी एक स्त्रोत है।

[12/31/2021, 12:10 PM] Dr. Shekhar Singh Rathore Jabalpur: 

👍🏻👍🏻

Not only in cell but if you think of an Atom, Aakaash Is there.

 Atom में ख वैगुण्य से रेडियो एक्टिविटी और एटम बम परिणमित होते हैं।।

[12/31/2021, 12:13 PM] Vd. Divyesh Desai Surat: 

केवल खवैगुण्य  से व्याधि की उत्पत्ति नही होती, संग होना भी व्याधि उत्पति के लिए  जरूरी है।। और दोषो का प्रकोप होना भी जरूरी है, साथ मे आगे सुभाष सर ने बताया था वैसे रोगों के प्रसार में वायु का भी महत्वपूर्ण योगदान है।🙏🏾🙏🏾

[12/31/2021, 12:14 PM] Dr. BK Mishra: 

सर, अधिकांश व्याधियां तो इतने सूक्ष्म की अपेक्षा नहीं करती, पहले ही लक्षण उत्पन्न कर देती हैं...🙏🏼🙂

[12/31/2021, 12:14 PM] Dr. Surendra A. Soni: 

निज व्याधियों में आवश्यक रूप से तथा आगन्तुक में कालान्तर में ।
शरीर स्वभाव भी इसमें एक आवश्यक घटक है यथा जरा ।

[12/31/2021, 12:14 PM] Dr. BK Mishra: 

संग स्रोतोदुष्टि का एक प्रकार है,

बात काफी आगे बढ़ चुकी है. .😄😄

[12/31/2021, 12:19 PM] Dr. BK Mishra: 

स पुमाश्चेतनम् तच्च तच्चाधिकारणं ...🙏🏼

[12/31/2021, 12:20 PM] Dr. Shekhar Singh Rathore Jabalpur: 

जी। विषय स्पष्ट करने हेतु कहा।।

[12/31/2021, 12:21 PM] Dr. Shekhar Singh Rathore Jabalpur: 

बिना ख वैगुण्य, संग कैसे होगा सर ??

[12/31/2021, 12:33 PM] Dr. Surendra A. Soni: 

वायोर्धातुक्षयात् कोपो मार्गस्यावरणेन च (वा) ।

वातव्याधि के पूर्वरूप नहीं है परंतु धातु क्षय के तो हैं ही ? उसी आधार पर हमें ग्रहण करना है । अब प्रश्न आएगा कि इससे उक्त दोनों का एकत्व हो जाएगा ? तो उत्तर ये है कि वात की विकृति से उत्पन्न असाधारण लक्षण युक्त व्याधि वात व्याधि है । आ.  चक्रपाणि जी ने च चि 28 के प्रारंभ में विशद टीका लिखी है वह अवलोकन योग्य है ।

डॉ चौहान साब !🙏🏻

[12/31/2021, 12:34 PM] Vd. Divyesh Desai Surat: 

रोगोत्पत्ति में भी संग (कार्य अवरोध) होना जरूरी है।🙏🏾🙏🏾
कुपितानां हि दोषाणाम,
शरीरे परिधावताम,
यत्र *संगः* खवैगुण्यात
व्याधिस्तत्र उपजायते।।
यहाँ संग का मतलब कार्य हानि ऐसा हमे हमारे काय/ रोग के प्राध्यापक ने सिखाया था, यहाँ ये विद्यापीठ में बहोत सारे ज्ञानवृद्ध काय चिकित्सकों का संगम है, आप भी श्रेष्ठ काय चिकित्सक है, तो संग का मतलब क्या लेना चाहिये, इस पर तद्वित संभाषा हो सकती है।🙏🏾🙏🏾

[12/31/2021, 12:37 PM] Dr. Pawan Madan: यह खवैगूण्य के प्रकार हैं या स्रोतोदुष्टि के

या  खवैगूण्य व स्रोतोदुष्टि एक ही बात है ?

[12/31/2021, 12:45 PM] Dr. Kalpna Monga Gurugram: 

Yes sir.... 🙏🙏🙏🙏 yehi Dharmadhikari sir batate hain..
 Agnimandye k saath agar ahitkar khan pan jeevan shalli rakhenge to dosh sang hoga aur jis dosh ka adhikay hoga ushke anusaar hoga... Kripya bataye....

[12/31/2021, 12:49 PM] Dr. Kalpna Monga Gurugram: 

Y khe vagunya unke anusaar sharir mei kahin b ho skta hai ... Aisa mujhe ab tak samajh aaya .. aap sabi aage marg darshan kare 🙏🙏🙏🙏

[12/31/2021, 1:14 PM] Vd. Divyesh Desai Surat: 

🙏🏾🙏🏾As usual good, informative points highlight by you💐💐
रघुराम सर, आपको देखकर बहोत खुशी हुई।।

[12/31/2021, 1:17 PM] Dr. Bhadresh Naik Gujarat: 

As usual always informative
Compration between morden disease in to ayurved👏🌹👌👆

[12/31/2021, 1:36 PM] Prof.Vd.Arun Rathi: 

*ये स्त्रोतो दुष्टी के लक्षण है, ख वैगुण्य होने के बाद ही स्त्रोतो दुष्टी होगी*

*स्त्रोतो दुष्टी मे ख वैगुण्य किस तरह से और कौनसी संप्राप्ति होगी यह लक्षणों व्दारा विदित होगा*

[12/31/2021, 1:39 PM] Vaidya Sanjay P. Chhajed: 

स्त्रोतों विकृति ४ प्रकार की वर्णित है
अतिप्रवृत्ती
संग
सिरानाम् ग्रंथी
विमार्गगमन

स्त्रोतों शैथिल्य से आप क्या कहना चाहते हैं ?

[12/31/2021, 1:51 PM] Dr. Bhavesh R. Modh Kutch: 

ख-वैगुण्य 
यहाँ ख का तात्पर्य  अवकाश-space
गति के लिए  एक निश्चित  खाली जगह यानी के स्पेश  जरूरी है ।

जब यह स्पेस  नही रहती है तब फंकशन  डिस्टर्ब  या फेइल  होने लगते है ।

वह स्पेस मे बीनअधिकृत दुसरे का घुसना ... वह मूल स्पेस के अधिकारीक  की आलस  या गफलत होती है  जिससे  प्रज्ञापराध  भी कहाँ जाता है ।

यह सारा घटना क्रम  को ख-वैगुण्य

ख-वैगुण्य का  अधिकांश  शुरूआत  मे सबको पत्ता नही चलता  विकार प्रगट होने के बाद  जब सामान्य या प्राथमिक  इलाज  मनोवांछित  सिद्धि  नही देता तब विचक्षण  चिकित्सक  ... ख-वैगुण्य  के बारे मे सोचता है ... 
ख वैगुण्य के विषय मे जानकारी  मिलने से चिकित्सा... व्याधि  अपूनर्भव  की दिशा मे कार्य  हो शकता है ।

[12/31/2021, 2:24 PM] Prof. Giriraj Sharma: 

सादर नमन आचार्य 
स्रोतस विध्द- आचार्य सुश्रुत 

स्रोतस दुष्टि -आचार्य चरक 

स्रोतस (ख) वैगुण्य

स्रोतस शैथिल्य का कोई शास्त्रीय संदर्भ अगर हो तो कृपया निर्देश करे,,,

[12/31/2021, 2:27 PM] Dr. Surendra A. Soni: 

दोषाः पित्तप्रधानास्तु यस्य कुप्यन्ति धातुषु ।
शैथिल्यं तस्य धातूनां गौरवं चोपजायते ॥४॥

प्रमेह, कुष्ठ और भी कई स्थानों पर सन्दर्भ हैं ।

[12/31/2021, 2:28 PM] Prof. Giriraj Sharma: 

स्रोतस शैथिल्य शब्द

[12/31/2021, 2:29 PM] Prof. Giriraj Sharma: 

धातु के परिपेक्ष्य में
मांस शैथिल्य
सिरा शैथिल्य
आदि है 
स्रोतस शैथिल्य अगर कहीं हो तो

[12/31/2021, 2:39 PM] Dr. prajakta Tomar: 

Two possibilities dosha prakop leading to dhatu dusti like sir told in  prameha etc and dhatu dusti first then vitiating dosha. In vata rakta,  rakta dhatu gets vitiated and then along with vitiated vata causes sthan vaigungya results in vatarakta.

Correct me gurujans if wrong 🙏🏻

[12/31/2021, 2:47 PM] Prof.Vd.Arun Rathi: 

*शरीरस्थ स्थाई घातुओं से अवयव, अंग प्रत्यांग और स्त्रोतस् का निर्माण होता है.*

[12/31/2021, 3:12 PM] Dr. prajakta Tomar:

 ✅ normal course but if organ dusti is there it can vitiate dosha  can cause vyadhi.

[12/31/2021, 3:42 PM] Dr. Pawan Madan:

 जी
मै यही कहना चाह रहा था
जब खवैगूण्य हो तो जरुरी नही के व्याधि व्यक्त हो
पर जब स्रोतोदुष्टि  हो जाती है तो व्याधि व्यक्त हो जाती है !

[12/31/2021, 4:06 PM] Dr. Govind Ojha: 

*ख=आकाश=रिक्तता, ख वैगुण्य अर्थात दोषों का स्थान संश्रय, खाली स्थान पर किसी दोष का आ जाना, इसे खवैगुण्य कहेंगे, अल्प मतिनुसार*🙏😊

[12/31/2021, 4:10 PM] Dr. Surendra A. Soni: 

स्रोतस् एवं धातुदुष्टि पृथक नहीं है । धातुदुष्टि निदान अलग से निर्दिष्ट नहीं है उसी तरह स्रोतोशैथिल्य को ग्रहण करना चाहिए ।
कल पवन जी को दिए गए उत्तर में सन्दर्भ उल्लेख किया गया है ।
Prof. Giriraj ji !
🙏🏻 

[12/31/2021, 4:16 PM] Prof. Giriraj Sharma: 

आचार्यो ने धातु दुष्टि रस रक्तादि दुष्टि का वर्णन अलग से किया है स्रोतस दुष्टि का अलग से वर्णित है । 
शास्त्र सम्मत तो यही है व्यवहार सर्व ग्राह्य है

[12/31/2021, 4:19 PM] Dr. prajakta Tomar: 

दोष दुष्य समूर्च्छना होने के बाद ही व्याधी का उद्भव होता है

[12/31/2021, 4:32 PM] Vaidya Sanjay P. Chhajed: 

वहीं तो हम भी  पुछ रहे हैं।

[12/31/2021, 4:37 PM] Dr. Pawan Madan: 

जी , यही कहना था मेरा भी

[12/31/2021, 4:40 PM] Dr. BK Mishra: 

आचार्यश्री ख-वैगुण्य आंतरिक दोष से नहीं प्रायः बाह्य निदान से होता है

[12/31/2021, 4:42 PM] Dr. Pawan Madan: 

दोष तो हर स्थान पर रहते ही हैं।

[12/31/2021, 4:58 PM] Dr. Surendra A. Soni: 

(Prof. Giriraj ji !  My reply in bracket)

आचार्यो ने धातु दुष्टि रस रक्तादि दुष्टि का वर्णन अलग से किया है स्रोतस दुष्टि का अलग से वर्णित है । 

*(ऐसा क्यों किया है यह प्रश्न होना चाहिए ? ऐसा इसलिए किया है कि स्रोतों के मूल भी अलग अलग धातुओं से निर्मित हैं तथा विशिष्ट लक्षणों के उत्पादक हैं । इसी लिए धातु दुष्टि के पृथक निदान नहीं बताए हैं स्रोतोदुष्टि के बताए हैं । दोष ही धातुत्व रूप में भी रहते हैं अतः मात्र दोषज निदान बताए हैं जो धातुओं को भी दुष्ट करते हैं । विधिशोणितीय इसका उदाहरण है । आप शारीरविद अधिक गम्भीर व्याख्या कर सकते हैं ।)*

शास्त्र सम्मत तो यही है व्यवहार सर्व ग्राह्य है

🙏🏻🌹

[12/31/2021, 5:03 PM] Dr. BK Mishra: 

ख-वैगुण्य के *ख* से चक्रपाणि ने आकाश या सामान्य रिक्तता के बजाय आयुर्वेदीय स्रोतस  तातपर्य ग्रहण किया है..
 *ख-वैगुण्यादिति स्रोतोवैगुण्याद* च चि 15 पर

[12/31/2021, 5:05 PM] Dr. Sudha Sharma: 

ज्ञान वर्धन के लिए बहुत बहुत धन्यवाद सर🙏🙏

[12/31/2021, 5:07 PM] Dr. Govind Ojha: 

वातवैगुण्य तो होता है पर पित्त व कफ वैगुण्यक्यों नहीं सर एक जिज्ञासा🙏😊

[12/31/2021, 5:08 PM] Dr. D. C. Katoch sir: 

Seven stages of origin and progression of disturbance and manifestation of disease are- 

1)Predisposition (Kha -vaigunya - 
functional or structural disintegration in the body matrix, 

2) Abnormal accumulation of physiological/ pathological entities (Dosh Sanchaye), 

3)   Exaceravation of accumulated entities to initiate disturbance/ derangement (Dosh Prakop), 

4) Dissemination or diffusion of disturbed/ deranged entities (Dosh Prasar), 

5) Localization of disturbed/deranged entities in cells/ tissues/ organ/ System to cause early mobidity (Sthansanshraye leading to dosh-dusya sammoorchhana and poorvaroopavastha), 

6) Fully blown disease process or clinical manifestation (Vyaktavastha) 

7)  Advanced morbidity with complications or resolution of disease process ( Bhedavastha).

[12/31/2021, 5:16 PM] Dr. Pawan Madan: 

👏🏻👏🏻👏🏻👏🏻

Wonderful practical points, 🙏

Starting from khavaigunya --- through srotodushti + dosh dushya samoorchanna --- till Bhedavastha.

Thanks sir..🙏

[12/31/2021, 5:25 PM] Dr. D. C. Katoch sir: 

First four stages can be best managed with Shodhan Chikitsa and Samyak aahar-vihar, whereas once dosh-dusya sammoorchhna gets underway and progresses further, ubhayevipareet or vyadhivipareet chikitsa comprising of Shodhan-Shamana-Pathyavyavastha can break the samprapti of the disease and its progression.

[12/31/2021, 5:30 PM] Dr. prajakta Tomar: 

Sir !
दोष दुष्य समूर्च्छाना होने के बाद रूप अवस्था आती है ना, उसके पूर्व सम्पूर्ण समूर्च्छना पूर्ण ना हो तो पूर्वरूप अवस्था होती है |

[12/31/2021, 5:37 PM] Dr. Radheshyam Soni Delhi: 

संग से तात्पर्य प्रतिघात से ही है।

परिधावन में बाधा ही प्रतिघात है।

ख से तातपर्य अवकाश, आकाश, रिक्त स्थान अर्थात प्रतिघात रहित स्थल।


ख वैगुण्य- इस अप्रतिघात में विकार अर्थात प्रतिघात की स्थिति। यह स्थिति सामान्य परिधावन में व्यवधान का कारण बनती है। और यही दुःख, क्लेश अर्थात व्याधि का कारण बनती है।

क्रोधित व्यक्ति को कोई रोक कर टोकाटाकी करे तो वो क्रोधित व्यक्ति रोकने वाले को भलाबुरा कहने लगे और हो सकता है झगड़ा करे, झापड़ भी रसीद कर दे जो क्लेश( व्याधि )की अभिव्यक्ति है।😄

[12/31/2021, 5:39 PM] Dr. D. C. Katoch sir: 

Early part of dosh-dusyasammoorchhana is Poorvarupavastha and later part of complete dosh-dushyasammoorchhana is clinical stage when the patient goes/ is taken to doctor or dispensary or hospital for treatment.

[12/31/2021, 5:40 PM] Prof. Giriraj Sharma: 

स्रोतस समुदाय ही पुरुष है 
कर्म पुरुष ,,,,,
😀😀🙏🏼🙏🏼👍🏻

[12/31/2021, 5:46 PM] Dr. D. C. Katoch sir: 

Akashiye bhav (srotas) mein sang ya pratighat hi to functional Kha vaigunya hai.

[12/31/2021, 5:47 PM] Dr. Surendra A. Soni: 

Perfect analysis incorporating and establishing kg-vaigunya prior to Shadkriyakal like Prof. Dhyani ji !

🙏🏻🌹👌🏻
Namo namah !! Respected Katoch Sir !

[12/31/2021, 5:49 PM] Prof. Madhava Diggavi Sir: 

Excellent explanation Sir ji.

[12/31/2021, 5:49 PM] वैद्य ऋतुराज वर्मा: 

क्षिप्यमाणः खवैगुण्याद्रसः सज्जति यत्र सः।
करोति विकृतिं तत्र खे वर्षमिव तोदयः।। 
(च. चि.15/37)

[12/31/2021, 5:52 PM] Dr. BK Mishra:

Katoch Sir !
 👌🏼 Nicely elaborated🌹

[12/31/2021, 5:54 PM] Dr. Radheshyam Soni Delhi: 

ख- वैगुण्य की स्थिति लाने में आहार से अधिक विहार की भूमिका लगती है।

*आहार से दोष प्रकोप एवं धातु/ स्रोतों दुष्टि और विहार से ख वैगुण्य।*
(हालांकि यह सार्वभौमिक सत्य नहीं है।)

कुछ उदाहरण-

1. अर्श के निदान में उत्कटासन।

2. सिरा कुटिलता में अधिक देर खड़ा रहना या अध्व कार्य।

3. वातरक्त में अधिक अध्व, पैदल, दौड़ आदि।

4. उरःक्षत जन्य राजयक्ष्मा में अतिसाहस

5. गुदभ्रंश में- अतिप्रवाहण

6. प्रमेह या स्थौल्य में दिवाशयन

7. अक्षिरोगों में स्पष्ट रूप से लिखा- उष्ण अभितप्त होकर जल प्रवेश, सूक्ष्म निरीक्षण, आदि।

8. उन्माद में देव गुरु द्विज का अपमान।

9.आगंतुज व्रणशोथ में आघात आदि

10. अर्बुद रोग में निरंतर स्थानिक व्याघात।

11.  अतिसाहस/ भार उठाने से ब्रध्न रोग

🙏🙏🙏🌹🌹🌹


[12/31/2021, 6:05 PM] Dr. Surendra A. Soni: 👌🏻👍🏻👏🏻

[12/31/2021, 6:06 PM] Vd Shailendra Mehta: 👌🏻👌🏻🌷🌷🙏🏻🙏🏻

[12/31/2021, 6:08 PM] Vd Shailendra Mehta:

 गुरुदेव श्री,🙏🏻🙏🏻
,,क्या आवरण सिद्धान्त,,,भी,,,ख वैगुण्य पर आश्रित है,,,?
 और  विशेषतः वात के परस्पर आवरण,,,?

[12/31/2021, 6:12 PM] Prof. Giriraj Sharma: 

स्रोतांसि खलु *परिणाममापद्यमानानां* धातूनामभिवाहीनि भवन्त्ययनार्थेन||३||

परिणाममापद्यमानानामिति पूर्वपूर्वरसादिरूपतापरित्यागेनोत्तरोत्तररक्तादिरूपतामापद्यमानानाम्

एक धातु है दूसरा धातु अयन है । 
यथा 
इंद्र एवं इन्द्रिय
आत्मा एवं मन
 इससे ज्यादा कुछ नही समझ सका ।।

🌹🌹🙏🏼🙏🏼🙏🏼🌹🌹

[12/31/2021, 6:14 PM] Dr. D. C. Katoch sir: 

Vaat mein kha vaigunya hone par hi uska aavran hota hai.

[12/31/2021, 6:15 PM] Dr. Radheshyam Soni Delhi:

 सच बात तो ये है कि आवरण प्रकरण को परीक्षा में पास होने के लिये ही पढ़ा था, उसे समझने की ना कभी कोशिश की, न कभी जरूरत पड़ी।😆


पर इस विषय आदरणीय सत्येन्द्र ओझा सर विशेष योग्यता रखते हैं। उनका इस संदर्भ में लेख सम्भवतः ब्लॉग पर उपलब्ध हो, उसे पढ़ना ठीक रहेगा।🙏🙏

[12/31/2021, 6:15 PM] Dr. D. C. Katoch sir: 

Arthat vaat ke akashiye bhav ka pratighat ya sang.

[12/31/2021, 6:16 PM] Vd Shailendra Mehta: 

जी,,,अब अमूर्त वात मे ख वैगुण्य कैसे समझें,गुरुदेव😇😇

[12/31/2021, 6:16 PM] Dr. Pawan Madan: 

आचार्य गिरिराजजी !

क्या बिना धातु के अयन का अस्तित्व हो सकता है ?

[12/31/2021, 6:18 PM] Dr. Radheshyam Soni Delhi: 

स्वयं ख और स्रोतस भी अमूर्त हैं। तीनो दोष अमूर्त हैं। धातुएं भी अमूर्त ही है।

[12/31/2021, 6:18 PM] Dr. D. C. Katoch sir: 

Lo kar lo baat. Vaat ke sangthan mein vayu aur akash ki ulvanta/adhikata hoti hai.

[12/31/2021, 6:19 PM] Dr. Radheshyam Soni Delhi: 

और इन अमूर्त भावों से ही स्थूल शरीर और अवयव निर्मित हैं जो मूर्त हैं।

[12/31/2021, 6:20 PM] Vd Shailendra Mehta: 

🙂🙂🙏🏻🌷
 यानी की,,,pre/अमूर्त ख वैगुण्य,,,pre/ अमूर्त स्रोतस,,,,pre/अमूर्त दोष और धातु,,,,यही मतलब हूआ ना गुरुदेव🙏🏻🙏🏻

[12/31/2021, 6:30 PM] Prof. Giriraj Sharma: 

यावन्तः पुरुषे मूर्तिमन्तो भावविशेषास्तावन्त एवास्मिन् स्रोतसां प्रकारविशेषाः|

🕉️🕉️🕉️🕉️🕉️🕉️

[12/31/2021, 6:31 PM] Dr. Surendra A. Soni: 👍🏻

[12/31/2021, 6:36 PM] Dr. Radheshyam Soni Delhi: 

🙏🙏🙏
पर इस शास्त्र वचन से मेरे वक्तव्य की न पुष्टि होतीहै न ही प्रतिकार🤔

[12/31/2021, 6:37 PM] Prof. Lakshmikant Dwivedi Sir: 

1) ATI pravratti,.............. shiranam  granthyo pi VA.
 *Shroto dushti* ke 4👆🏼(kisi post me) lakshan, bataya.

B) Khavaigunya huva to Ek-hi, sangah se Vyadhi.
 Atipravratti.....ect ka kyo?nahi.

[12/31/2021, 6:40 PM] Dr. Radheshyam Soni Delhi: 

अर्थात ख वैगुण्य और स्रोतों दुष्टि दोनों भिन्न हैं।


*ख* वैगुण्य को मूर्त होना चाहिये,

जबकि स्रोतों दुष्टि को अमूर्त एवं लक्षणों से परिलक्षित होना मानना चाहिए🤔

[12/31/2021, 6:42 PM] Dr. Radheshyam Soni Delhi: 

क्या स्रोतों दुष्टि को क्रियात्मक

और ख वैगुण्य को रचनात्मक दुष्टि मान सकते हैं ?🤔

[12/31/2021, 6:44 PM] Prof. Lakshmikant Dwivedi Sir: 

A) ATI pravratti,.............. shiranam  granthyo pi VA.
 *Shroto dushti* ke 4👆🏼(kisi post me) lakshan, bataya.

B) Khavaigunya huva to Ek-hi, sangah se Vyadhi.
 Atipravratti.....ect ka kyo?nahi.
Shroto dushti ke 4 lakshan.
Khavaigunya me janha sang paridhavan karte doshon  ka janha sanga hojaaye to vanha Vyadhi utpann hoti.✓
To khavaigunya me Atipravratti janha ho  to use Vyadhi kahe ya nahin. ..kya samajhe.

[12/31/2021, 6:50 PM] Prof. Lakshmikant Dwivedi Sir: 

Shroto dushti ke 4 lakshan.
Me ek ghatak sang paridhavan karte me sanga janit badha se vanha Vyadhi utpann hoti hai.
To shesh 3 lakshan hai kya unka Vyadhi utpann me khavaigunya ki pratibhagita ?

[12/31/2021, 7:01 PM] Prof. Giriraj Sharma: 

आदरणीय 
मूर्त एवं अमूर्त से क्या ग्रहण करे ,,,,,,?

[12/31/2021, 7:06 PM] Dr. Radheshyam Soni Delhi: 

प्रणाम बोस🙏

मूर्त अर्थात इन्द्रिय ग्राह्य

[12/31/2021, 7:18 PM] Prof. Giriraj Sharma: 

👍🏻
पूर्वरूप इन्द्रिय ग्राह्य है कुछ रोगी को एवं कुछ चिकित्सक को तो कुछ रोगी एवं चिकित्सक को दोनो को,,,,,
रूप भी इसी तरह इन्द्रिय ग्राह्य है ,,,,
फिर लक्षण या पूर्वरूप सब मूर्त ही है ।

[12/31/2021, 7:18 PM] Prof.Vd.Arun Rathi: 

*क्या स्त्रोतस् संग होने पर वायु का मार्ग अवरोध उत्पन्न होकर स्त्रोतस् मे विमार्गगमनं नही होगा.*

[12/31/2021, 7:22 PM] Prof.Vd.Arun Rathi: 

*मुझे लगता है, स्त्रोतस् दुष्टी मे मूल कारण का विवेचन कर संप्राप्ति को समझना चाहिए और 'ख वैगुण्य" किैस कारण से है यह ज्ञात करना चाहिए*

[12/31/2021, 7:28 PM] Prof. Giriraj Sharma: 

नमस्कार आदरणीय
धमनियों में कोलेस्ट्रॉल बढ़ जाने पर संग होता है परन्तु विमार्ग गमन होगा यह अल्प संग या पूर्ण संग पर निर्भर है एवं साथ मे अन्य सहयोगी मार्ग पर भी,,,,
अगर अल्प संग हुआ तो विमार्ग गमन नही होगा क्षीण गमन होगा ,,, या विकल्प से गमन होगा ,,,,,
पूर्ण संग विकल्प के साथ तो विमार्ग गमन होगा 
वायु के परिपेक्ष्य में ऊर्ध्व गुद (अशोक राजा को ऐसा था स्मृति आधारित) रोग वायु का प्रतिलोम , 
उदावर्त भी एक उदाहरण है ।
संभवत,,,,,

[12/31/2021, 7:31 PM] Prof. Giriraj Sharma: 

संग विकल्प उदाहरण
Portal systematic circulation.

[12/31/2021, 7:39 PM] Dr. D. C. Katoch sir: 


Buddhi sang hi karana hai jo hamein Kha Vaigunya samajh nahi aa raha 🙂😔

[12/31/2021, 7:43 PM] Prof. Giriraj Sharma: 👍🏻👍🏻👍🏻👌🏻👌🏻👌🏻👌🏻

[12/31/2021, 7:44 PM] Prof. Giriraj Sharma: 


मेरी तो अतिप्रवृति दुष्टि है आज
😄😄😄😄😄

[12/31/2021, 7:44 PM] Dr. Pawan Madan: 

खवैगूण्य स्रोतो दुश्टि से पहले की अवस्था है,,,

[12/31/2021, 8:15 PM] Dr. BK Mishra: 

यहां वैगुण्य से आपका क्या तात्पर्य है..

[12/31/2021, 8:17 PM] Dr. Govind Ojha: 

स्थान भ्रष्ट/विमार्ग

[12/31/2021, 8:32 PM] Prof.Vd.Arun Rathi: 

*यह जरुरी नही है*

*स्त्रोतस् दुष्टी पुर्व एवं पश्चात भी ख वैगुण्य बना रह सकता है*

[12/31/2021, 8:39 PM] Prof. Madhava Diggavi Sir: 

Ji
Yatra sangah ...tatra vyadhi 

Yatra khavaigunyat sangah..tatra vyadhi.

[12/31/2021, 8:42 PM] Dr. Gurdip Singh Sir: 

Guna means characteristics
Every dravya has specific characteristics. Any change in it is vaigunya particularly adverse is vaigunya. Here Kha is srotas. That pulsate is dhamani, where saran flowing occurs that is site and where oozing takes place that is srotas. srotas are present in membranes. Khavaigunya predisposes the tissue for disease.
When prakupita dosha  while circulating comes in contact with such tissues Dosh dushya sammurchana  occurs and disease is initiated. Kha vaigunya may occur due to adi bala pravritt.

[12/31/2021, 8:43 PM] Prof. Madhava Diggavi Sir: 

Anti nuclear antibodies are circulating in the blood, when and  where they get suitable site or susceptibility or acceptance for the disease ..then and there only disease manifests.
Disease acceptance or disease rejection is based on pratyaneeka Bala, vyadhijshamatva, failure in cell intelligence, wrong signalling, previous exposure to vyadhi utpadakta hetu, beeja Dosha , ...

[12/31/2021, 8:50 PM] Dr. Gurdip Sing Sir: 

or janm or adibhautik or daiva balance causes. recepters on the cell membrane controls entertaining in the cell particularly of the unusual substances.

[12/31/2021, 8:54 PM] Prof. Giriraj Sharma: 

सादर प्रणाम गुरु जी
🙏🏼🙏🏼🙏🏼🕉️🙏🏼🙏🏼🙏🏼

[12/31/2021, 9:01 PM] Prof. Madhava Diggavi Sir: 

Kha means srotas 
Kha means aakasha 
Vaigunya ..is abnormality,

[12/31/2021, 9:01 PM] Dr. Gurdip Sing Sir: 

Thanks

[12/31/2021, 9:08 PM] Dr. Pawan Madan: 

जी
क्या ऐसी कोई स्तिथि ही सकती है जब बिना खवैगूण्य के स्रोतो दुष्टि हो ?

[12/31/2021, 9:08 PM] Dr. BK Mishra: 

इस अर्थ में तो पित्त और कफ का भी वैगुण्य होता है। उदाहरण स्वरूप शीतपित्त ले सकते हैं, जिसमें तीनों ही दोष स्थानभ्रष्ट होकर बहि अन्तः विसर्पण करते हुऐ विकृति उत्पन्न करते हैं।

शीतमारुतसंस्पर्शात् प्रदुष्टौ *कफमारुतौ पित्तेन सह सम्भूय बहिरन्तः विसर्पतः* 
 शाखाश्रित कामला में भी कफ के द्वारा पित्त का मार्ग अवरुद्ध कर देने के कारण पित्त शाखाओं में विमार्गगमन करता है।
 *आशयापकर्ष* की स्थितियां अवलोकनीय है।

[12/31/2021, 9:12 PM] Dr. Pawan Madan:

 🙏🙏🙏🙏🙏🙏👏🏻👏🏻👏🏻👏🏻👏🏻👏🏻

Pranaam sir.

Khaivaigunya predisposes the tissues to disease.
🙏🙏🙏

Khavaiguny 》》》Srotodushti 》》》Dosha dushya sammoorchanaa 》》》vyadhi

🙏🙏🙏

[12/31/2021, 9:12 PM] Dr. Gurdip Sing Sir: 

For example excessive use of kashay ras causes srotansyavabadhnati 
ch. sure 26


[12/31/2021, 9:14 PM] Dr. BK Mishra: 

ख-वैगुण्य पूर्वक ही दोषदुष्यसंमुर्च्छना होकर स्रोतोदुष्टि होगी...

[12/31/2021, 9:15 PM] Dr. Pawan Madan: 

Ji bilkul.
✔️✔️

[12/31/2021, 9:26 PM] Dr. BK Mishra:
 Khavaiguny➡️Dosha dushya sammoorchana➡️Srotodushti ➡️vyadhi

🙏🙏🙏

[12/31/2021, 9:26 PM] Dr. Gurdip Sing Sir: 

in general prakop of dosh is due to their vaigunya.

[12/31/2021, 9:27 PM] Dr. BK Mishra: 

🙏🏼💐🙏🏼Sir

[12/31/2021, 9:28 PM] Vd Shailendra Mehta: 

परम गुरुदेवश्री को सादर नमन🧎🏻‍♂️🧎🏻‍♂️🌷🌷

[12/31/2021, 10:27 PM] Dr. D. C. Katoch sir: 

Not Khavaigunya》》》Srotodushti 》》》Dosha dushya sammoorchanaa 》》》Vyadhi. It is like Khavaigunya - --Dosh sthansanshraye at the site of Khavaigunya- --Doshdusyasammoorchhna - Srotodushti - Vyadhi uttapati with symptom complex (Lakshan sammuchaye) of dosh prakop, agni dusti, dhatu dusti and srotodusti.

[12/31/2021, 10:33 PM] Dr. Gurdip Sing Sir: 

You may say Dhatu dushti and not sroto dushti.

[12/31/2021, 11:17 PM] Dr. D. C. Katoch sir: 

I think, Srotas being responsible for transformation and transportation srotodusti leads to cause dhatu dushti not vice verse.

[12/31/2021, 11:18 PM] Dr. Mrunal Tiwari Sir: 

वात दोष अमूर्तिक है। वात दोष को उसके गुणों द्वारा उसका परिक्षण किया जाता है। कफ, पित्त मूर्त है। यह एक अनुमान है। अध्ययन करते समय कफ, पित्त वैगुण्य पढ़ा नहीं।

[12/31/2021, 11:51 PM] Dr. Gurdip Sing Sir: 

if amurat is not visible then OK otherwise Vayu is dravya which is perceived by Sprash. kaph and pitta have also Guna and adverse change in their guna may be taken as vaigunya but for them prakop word is used. Yes vaigunya word is also used for vata particularly to indicate it's Pratilom gati.

[12/31/2021, 11:56 PM] Dr. Satish Sharma: 

माननीय Gurudev ke gyan से अभिभूत हो रहे हैं आपने satya व्याख्याएं प्रदर्शित की हैं 🙏🏻🙏🏻🙏🏻🙏🏻🙏🏻🙏🏻

[1/1, 12:10 AM] Dr. Gurdip Sing Sir: 

very good we must think further. Dhatu and it's srotas are inter-related and once disease is produced or Dhatu dushti is done then four types sroto dushti occurs
in this way Khavaigunya and srotodushti are different conditions. one may be related to entering of toxic material and other may be with expulsion  of mala or defect in it.
sang vimarg gaman and atipravatti as functional defect and granthi as physical deffect.


[1/1, 5:45 AM] Vaidya Sanjay P. Chhajed: 

Respected Sir, when we think of three types of tubular structures  it is said
ध्मानात धमन्या:
स्त्रवणात स्त्रोतांसी
सरणात् सिरा:
It immediately reminds me of  the circulatory system 
This has been mentioned in the cha. Su. 30, which is providing information about the phenomenon of circulation as per Ayurved where heart is at center of the system and everything from Rasa to Oja is circulated through out the body through the channel, it is talking about par and apar of everything
 ----ओजोवहाः शरीरेऽस्मिन् विधम्यन्ते समन्ततः||८||

When it is circulated with pulsating tubes carries ,पर ओज these are converted to Oozing tubuler structure
स्त्रवणात----- 
Which carries nutrition to all the dhatus
परीणाम आपद्यमानानाम धातुनाम् अभिवहन्ति--- माधव निदान।
It is not only carrying the precurser of the dhatu , but it oozes out the required nutrition at the specific site. 
Then these tubes continue as
सरणात् सिरा:  the tubes which brings back the अपर ओज।

Capillaries are the smallest blood vessels in the body, connecting the smallest arteries to the smallest veins. These vessels are often referred to as the "microcirculation."

Only two layers of cells thick, the purpose of capillaries is to play the central role in the circulation, delivering oxygen in the blood to the tissues, and picking up carbon dioxide to be eliminated. They are also the place where nutrients are delivered to feed all of the cells of the body.

There are three primary types of capillaries—continuous, fenestrated, and discontinuous or sinusoidal that are found in different regions of the body, and specialized capillaries in the brain make up the blood-brain barrier.

Tests that evaluate the capillaries are important in assessing people medically, and there are several medical conditions associated with these vessels.


Capillaries are very thin, approximately 5 micrometers in diameter, and are composed of only two layers of cells—an inner layer of endothelial cells and an outer layer of epithelial cells. They are so small that red blood cells need to flow through them single file.

It's been estimated that there are 40 billion capillaries in the average human body. Surrounding this layer of cells is something called the basement membrane, a layer of protein surrounding the capillary.

If all the capillaries in the human body were lined up in single file, the line would stretch over 100,000 miles.

Capillaries in the Circulatory System

Capillaries may be thought of as the central portion of circulation. Blood leaves the heart through the aorta and the pulmonary arteries traveling to the rest of the body and to the lungs respectively.

These large arteries become smaller arterioles and eventually narrow to form the capillary bed. From the capillaries, blood flows into the smaller venules and then into veins, flowing back to the heart.

Capillaries are the smallest blood vessels in the body, connecting the smallest arteries to the smallest veins. These vessels are often referred to as the "microcirculation."

Only two layers of cells thick, the purpose of capillaries is to play the central role in the circulation, delivering oxygen in the blood to the tissues, and picking up carbon dioxide to be eliminated. They are also the place where nutrients are delivered to feed all of the cells of the body.

Capillaries are the smallest blood vessels in the body, connecting the smallest arteries to the smallest veins. These vessels are often referred to as the "microcirculation."

Only two layers of cells thick, the purpose of capillaries is to play the central role in the circulation, delivering oxygen in the blood to the tissues, and picking up carbon dioxide to be eliminated. They are also the place where nutrients are delivered to feed all of the cells of the body.

There are three primary types of capillaries—continuous, fenestrated, and discontinuous or sinusoidal that are found in different regions of the body, and specialized capillaries in the brain make up the blood-brain barrier.

Tests that evaluate the capillaries are important in assessing people medically, and there are several medical conditions associated with these vessels.


Structure

Capillaries are very thin, approximately 5 micrometers in diameter, and are composed of only two layers of cells—an inner layer of endothelial cells and an outer layer of epithelial cells. They are so small that red blood cells need to flow through them single file.

It's been estimated that there are 40 billion capillaries in the average human body. Surrounding this layer of cells is something called the basement membrane, a layer of protein surrounding the capillary.

If all the capillaries in the human body were lined up in single file, the line would stretch over 100,000 miles.

Capillaries in the Circulatory System

Capillaries may be thought of as the central portion of circulation. Blood leaves the heart through the aorta and the pulmonary arteries traveling to the rest of the body and to the lungs respectively.

These large arteries become smaller arterioles and eventually narrow to form the capillary bed. From the capillaries, blood flows into the smaller venules and then into veins, flowing back to the heart.

 The Role of Arteries in the Circulatory System

Number of Capillaries Varies by Tissue Type

The number of capillaries in a tissue can vary widely. Certainly, the lungs are packed with capillaries surrounding the alveoli to pick up oxygen and drop off carbon dioxide. Outside of the lungs, capillaries are more abundant in tissues that are more metabolically active.

Types of Capillaries

There are three primary types of capillaries in the circulation:

Continuous: These capillaries have no perforations and allow only small molecules to pass through. They are present in muscle, skin, fat, and nerve tissue.

Fenestrated: These capillaries have small pores that allow small molecules through and are located in the intestines, kidneys, and endocrine glands.

Sinusoidal or discontinuous: These capillaries have large open pores—large enough to allow a blood cell through. They are present in the bone marrow, lymph nodes, and the spleen, and are, in essence, the "leakiest" of the capillaries.

Blood-Brain Barrier

In the central nervous system the capillaries make up what is known as the blood-brain barrier. This barrier limits the ability of toxins (and, unfortunately, many chemotherapy agents and other medications) to pass through into the brain.

Looking for drugs that can pass through the blood-brain barrier, and hence, treat conditions such as brain metastases from a number of cancers, is an active area of research.

 How the Blood-Brain Barrier Works

The capillaries are responsible for facilitating the transport and exchange of gases, fluids, and nutrients in the body. While the arteries and arterioles act to transport these products to the capillaries, it is at the level of capillaries where the exchange takes place.

The capillaries also function to receive carbon dioxide and waste products that are then delivered to the kidneys and liver (for wastes) and the lungs (for exhalation of carbon dioxide).

fluids and nutrients diffuse through selectively permeable capillaries into the tissues of the body, and waste products are picked up in the capillaries to be transported through veins to the kidneys and liver where they are thus processed and eliminated from the body.
If we understand capillary system as स्रोतस् then every स्रोतस् is nothing but the capillary bed at that particular tissue. 
अतिप्रवृत्ति could be
Capillary permeability can be increased by cytokines (leukotrienes, histamines, and prostaglandins) released by cells of the immune system. The increased fluid (third spacing) locally can result in hives. When someone is very ill, this third spacing due to leaky capillaries may be widespread, giving their body a swollen appearance.

विमार्गगमन् could be
Systemic Capillary Leak Syndrome

A rare disorder known as capillary leak syndrome involves leaky capillaries which result in constant nasal congestion and episodes of fainting due to rapid drops in blood pressure.

सिरानां ग्रंथी could be Capillary Malformation

Capillary malformation (arteriovenous malformation syndrome) may occur as part of an inherited syndrome present in roughly 1 in 100,000 people of European ancestry. In this syndrome, there is more blood flow than normal through the capillaries near the skin, which results in pink and red dots on the skin.

The may occur alone, or people may have other complications of this syndrome such as arteriovenous malformations (abnormal connections between arteries and veins) which, when in the brain, can cause headaches and seizures

संग could be the blockage from both ways 
   One way is  that does not allow nutrition to ooze out ,& the other way is that it does not allow waste be brought back to the system .



Capillarries and their functions



Capillaries are very tiny blood vessels — so small that a single red blood cell can barely fit through them.
They help to connect your arteries and veins in addition to facilitating the exchange of certain elements between your blood and tissues.
This is why tissues that are very active, such as your muscles, liver, and kidneys, have an abundance of capillaries. Less metabolically active tissues, such as certain types of connective tissue, don’t have as many.

Capillaries connect the arterial system — which includes the blood vessels that carry blood away from your heart — to your venous system. Your venous system includes the blood vessels that carry blood back to your heart.
The exchange of oxygen, nutrients, and waste between your blood and tissues also happens in your capillaries. This happens through two processes:

Passive diffusion. This is the movement of a substance from an area of higher concentration to an area of lower concentration.

Pinocytosis. This refers to the process through which your body’s cells actively take in small molecules, such as fats and proteins.

The walls of capillaries are made up of a thin cell layer called endothelium that’s surrounded by another thin layer called a basement membrane.
Their single-layer endothelium composition, which varies among the different types of capillaries, and surrounding basement membrane makes capillaries a bit “leakier” than other types of blood vessels. This allows oxygen and other molecules to reach your body’s cells with greater ease.
Additionally, white blood cells from your immune system can use capillaries to reach sites of infection or other inflammatory damage.


There are three types of capillaries. Each has a slightly different structure that allows to function in a unique way.

Continuous capillaries

These are the most common types of capillaries. They contain small gaps in between their endothelial cells that allow for things like gases, water, sugar (glucose), and some hormones to pass through.
The continuous capillaries in the brain are an exception, however.
These capillaries are part of the blood-brain barrier, which helps to protect your brain by only allowing the most essential nutrients to cross.
That’s why the continuous capillaries in this area don’t have any gaps between endothelial cells, and their surrounding basement membrane is also thicker.

Fenestrated capillaries

Fenestrated capillaries are “leakier” than continuous capillaries. They contain small pores, in addition to small gaps between cells, in their walls that allow for the exchange of larger molecules.
This type of capillary is found in areas that require a lot of exchange between your blood and tissues. Examples of these areas include:

the small intestine, where nutrients are absorbed from food

the kidneys, where waste products are filtered out of the blood

Sinusoid capillaries

These are the rarest and “leakiest” type of capillary. Sinusoid capillaries allow for the exchange of large molecules, even cells. They’re able to do this because they have many larger gaps in their capillary wall, in addition to pores and small gaps. The surrounding basement membrane is also incomplete with openings in many places.
These types of capillaries are found in certain tissues, including those of your liver, spleen, and bone marrow.
For example, in your bone marrow, these capillaries allow newly produced blood cells to enter into the bloodstream and begin circulation.

What happens when capillaries don’t function properly?

While capillaries are very small, anything unusual in their functioning can cause visible symptoms or even potentially serious medical conditions.

Port wine stains

Port wine stains are a type of birthmark caused by the widening of capillaries located in your skin. This widening causes the skin to appear pink or dark red in color, giving the condition its name. Over time, they can darken in color and thicken.
While they don’t go away on their own, port wine stains also don’t spread to other areas.
Port wine stains typically don’t require treatment, although laser treatment can help to make them lighter in color.

Petechiae

Petechiae are small, round spots that appear on the skin. They’re typically about the size of a pinhead, can be red or purple in color, and are flat in the skin. They happen when capillaries leak blood into the skin. They don’t lighten in color when pressure is applied over them.
Petechiae are typically a symptom of an underlying condition, including:

infectious diseases, such as scarlet fever, meningococcal disease, and Rocky Mountain spotted fever

trauma from straining while vomiting or coughing

leukemia

scurvy

low platelet levels

Some medications, including penicillin, can also cause petechiae as a side effect.

Systemic capillary leak syndrome

Systemic capillary leak syndrome (SCLS) is a rare condition that doesn’t have a clear cause. But experts think it may be related to a substance in the blood that damages capillary walls.
People with SCLS have recurring attacks during which their blood pressure drops very quickly. These attacks can be severe and require emergency medical attention.
These attacks are usually accompanied by some initial warning signs, including:

nasal congestion

cough

nausea

headache

abdominal pain

lightheadedness

swelling in arms and legs

fainting

SCLS is usually treated with medications that help to prevent these attacks from occurring.

Arteriovenous malformation syndrome

People with arteriovenous malformation syndrome (AVM) have an abnormal tangle of arteries and veins that are connected to each other without capillaries in between. These tangles can occur anywhere in the body, but are most often found in the brain and spinal cord.
This can cause lesions that interfere with blood flow and oxygen delivery. These lesions may also cause bleeding into the surrounding tissue.
AVM usually doesn’t cause symptoms, so it’s usually only discovered while trying to diagnose another condition. However, in some cases, it can cause:

headaches

pain

weakness

issues with vision, speech, or movement

seizures

AVM is a rare condition that’s often present at the time of birth. Treatment usually involves surgically removing or closing the AVM lesion. Medication can also help to manage symptoms, such as pain or headaches.

Microcephaly-capillary malformation syndrome

Microcephaly-capillary malformation syndrome is a rare genetic condition that starts before birth.
People with this condition have smaller heads and brains. They also have widened capillaries that increase the flow of blood near the skin surface, which can cause pinkish red spots on the skin.
Additional symptoms can include:

severe developmental delays

seizures

difficulty eating

unusual movements

distinct facial features, which can include a sloped forehead, round face, and unusual hair growth

slower growth

shorter or smaller stature

finger and toe abnormalities, including really small or absent nails

Microcephaly-capillary malformation syndrome is caused by a mutation in a specific gene called the STAMBP gene. Mutations to this gene can result in cells dying during development, affecting the entire development process.
Treatment for this condition can involve stimulation — particularly through sound and touch — bracing to maintain posture, and anticonvulsant medication therapy for management of seizures.

The bottom line

Capillaries are tiny blood vessels that play a big role in facilitating the exchange of various substances between your bloodstream and tissues. There are several types of capillaries, each with a slightly different structure and function.

सिराणाम ग्रंथी can be conditions like deep vein thrombosis. It is clearly mentioned *siranaam* it definitely means not in Dhamani and strotas but specifically related to Sira only. 

If we understand the strotas as specific capillary system of the particular tissue and organ , then a lot querries can be solved easily.

[1/1, 8:42 AM] Dr Mansukh R Mangukiya Gujarat: 

🙏 Dr Sanjay Chhajed sir 🙏

[1/1, 9:11 AM] Dr. Gurdip Sing Sir: 

wonderful description congratulations Dr sanjay !

[1/1, 9:55 AM] Prof. Giriraj Sharma: 

*सरणात सिरा*
Which *BRINGS BACK* the अपर ओज

ओजोवहां शरीरे अस्मिन विधम्यते समन्ततः
चतुर्विशति धमन्यो नाभिप्रभवा अभिहिता सु शा

 ह्रदय चतुर्विशति धमनी अनु प्रविष्य
*ह्रदय धमनी अनुप्रविष्य*

आदरणीय यहां धमनी से क्या ग्रहण करे 
🙏🏼🙏🏼🌹🙏🏼🙏🏼🙏🏼

[1/1, 10:46 AM] Prof. Giriraj Sharma: 

तेन मूलेन महता महामूला मता दश| 
ओजोवहाः शरीरेऽस्मिन् विधम्यन्ते समन्ततः||८|| 


सम्प्रति धमनीनां महामूलत्वं प्रतिपादयति- तेनेत्यादि| तेनेति हृदयेन महता, ‘युक्ताः’ इति शेषः| विधम्यन्ते विसर्पन्ते [१२] ||८||

[1/1, 10:52 AM] Vaidya Sanjay P. Chhajed: 

Absolutely right sir 
Sira brings back the apar oja
While heart which is seat of par oja - Rasouj- sarva shariraj sneha - blow it away to all the bodily tissue by sending it through Dhamani- which takes it to strotas - which provide nutrition to dhatu by the process of selective permiability - kedarkulya nyay-  which nourishes dhatu- during the activities by the dhatu - apar oja is created - which is carried away from various organs to the heart by Sira.

That is what I wanted to convey it is nothing but the circulatory system. Moreover charak has specified that it is sparshgamya. Only part which is accessed bby touch is Dhamani- or the nadi which carries nutrition to the srotas, can be palpated to understand the variation.

[1/1, 10:56 AM] Prof. Giriraj Sharma: 

🙏🏼
पुनश्च,,,,
अर्द्ध अंजली परिमिति तस्य ओजसो धमन्य एवं ह्रदय आश्रित,,,,

[1/1, 11:09 AM] Prof. Giriraj Sharma: 

पुनश्च ,,,,
श्लैष्मिकस्य इत्यनेंन अष्ट बिंदु व्यतिरिक्तस्य ओजोवह धमनी बाहास्य ,,,, चक्र

[1/1, 11:12 AM] Prof. Giriraj Sharma:

 चतुर्विंशतिर्धमन्यो नाभिप्रभवा अभिहिताः |

*तत्र केचिदाहुः-* सिराधमनीस्रोतसामविभागः, सिराविकारा एव हि धमन्यः
 स्रोतांसि चेति |
धमनी शारीर 9

[1/1, 11:15 AM] Prof. Giriraj Sharma: 

*तत्र केचिदाहुः-*   सिराधमनीस्रोतसामविभागः, सिराविकारा एव हि धमन्यः स्रोतांसि चेति |

धमनी एवं स्रोतस ये दोनों  सिरा के विकार है, 
तो जरूर इस सिद्धांत से SIRA BRING APAR OJAS

[1/1, 11:23 AM] Prof. Giriraj Sharma: 

चतुर्विंशतिर्धमन्यो नाभिप्रभवा अभिहिताः |
तत्र केचिदाहुः- सिराधमनीस्रोतसामविभागः, सिराविकारा एव हि धमन्यः स्रोतांसि चेति |
तत्तु न सम्यक्, अन्या एव हि धमन्यः स्रोतांसि च सिराभ्यः; कस्मात्?
 व्यञ्जनान्यत्वान्, मूलसन्नियमात्, 
कर्मवैशेष्यात्, 
आगमाच्च; 
केवलं तु परस्परसन्निकर्षात् सदृशागमकर्मत्वात् 
सौक्ष्म्याच्च विभक्तकर्मणामप्यविभाग इव कर्मसु भवति ||३||

[1/1, 1:48 PM] +91 79727 51464: 

स्रवणात स्रोतांसि, सरणात सिरा इन सब मे आप महाभूत विद्यमान है,

[1/1, 1:49 PM] +91 79727 51464: 

वायू महाभूत का चल गुण और आप महाभूत का द्रव और सर गुण से ही स्रोतस अपने कार्य के लिए प्रेरित होता है।🙂🙏🏻
प्रणाम सभी गुरुजनो को।

[1/1, 3:14 PM] Vaidya Sanjay P. Chhajed: 

What point you want to make sir ?

[1/2, 9:34 AM] Prof. Giriraj Sharma: 

🙏🏼🙏🏼👌🏻🙏🏼🙏🏼
*सादर नमन आचार्य जी,*
*आपके इस वक्तव्य को पढ़कर  विमर्श का आनंद बढ़ गया ।*
*आपके इस discussion को पुनः पढा,,,,*
*पुनः स्व विवेचित हुआ*
*स्रोतस के इतने पर्याय, स्वरूप, मूल, दुष्टि हेतु व लक्षण को आचार्यो द्वारा प्रतिपादित करने के विषय मे पुनः अध्ययन करने की जिज्ञासा उत्पन्न  हुई,,,*
*आपके लेख में कुछ विशिष्ठ शब्दो का आपने उल्लेख व वर्णन किया ।*
*यथा-* 
*रसोज़ - रस एवं ओज* 
*रसधातु का बल ,  एवं सर्वधातु बल,*
 *पर अपर ओज -  स्थिर गुण *ओज ,सर गुण युक्त ओज,,,*
*स्थिर ओज,,, जिसके क्षय पर मृत्यु ---*
*सर्व शरीर विसर्पित ओज  जिसके प्रमेह आदि रोगों में आचार्यो ने उल्लेख किया है ।*
*आपके इस लेख में मूल रूप से प्राणवह, रसवह एवं रक्तवह स्रोतस की विवेचना है ।*

*विशेषकर आप नाड़ी विज्ञान में पारंगत है नाड़ी विज्ञान के लिये हम स्पंदन को एक महत्वपूर्ण इकाई मानते है ।*

*स्पंदन सर्व शरीर मे होता है परन्तु इन्द्रियगम्यता, कर्म एवं लक्षण गम्यता से जाना है ।*

*जीवसाक्षिणी शब्द नाड़ी के लिये ही प्रयुक्त है जिसमे स्पंदन की गति पर विचार किया जाता है ।*
*प्राण (जीव आत्मा) की शरीर मे स्थिति का ज्ञान दर्शन स्पर्शन प्रश्न से करते है , स्पर्शन में संभवतः नाड़ी का भी समायोजन हो जाता है ।*
चरक इन्द्रिय स्थान में आचार्यो ने बहुत सूत्र लिखे है उसमें एक सूत्र - 
खलु वर्णश्च स्वरश्च गन्धश्च रसश्च *स्पर्शश्च* चक्षुश्च श्रोत्रं च घ्राणं च रसनं च *स्पर्शनं* च सत्त्वं च भक्तिश्च शौचं च शीलं चाचारश्च स्मृतिश्चाकृतिश्च प्रकृतिश्च विकृतिश्च बलं च ग्लानिश्च मेधा च हर्षश्च रौक्ष्यं च स्नेहश्च तन्द्रा चारम्भश्च गौरवं च लाघवं च गुणाश्चाहारश्च विहारश्चाहारपरिणामश्चोपायश्चापायश्च व्याधिश्च व्याधिपूर्वरूपं च वेदनाश्चोपद्रवाश्च----- च भेषजसंवृत्तिश्च [२]

 भेषजविकारयुक्तिश्चेति परीक्ष्याणि *प्रत्यक्षानुमानोपदेशैरायुषः प्रमाणावशेषं जिज्ञासमानेन भिषजा||३*


*तत्र यानि पुरुषमनाश्रितानि तान्युपदेशतो युक्तितश्च परीक्षेत, पुरुषसंश्रयाणि पुनः प्रकृतितो विकृतितश्च||४||*

*जातिकुलदेशकालवयःप्रत्यात्मनियता हि तेषां तेषां पुरुषाणां ते ते भावविशेषा*

*विकृतिः पुन-
*लक्षणनिमित्ता च,*
*लक्ष्यनिमित्ता च, निमित्तानुरूपा च||६|*
प्राण की स्थिति को समझने के लिए,,,

*अग्नि सोम वायु सत्व रज तम के साथ पंच बुद्धिइन्द्रिय भी  प्राण ही है यह सब स्व आयतन विशेष प्राणायतनो में अवस्थित रहते है ।*

*आप के इस लेख में स्रोत विवेचन रसोज़ रक्त एवं प्राण की आधुनिक परिपेक्ष्य में महत्वपूर्ण संकल्पना है ।*

*जो सोम,वायु एवं अग्नि प्राण तथा रक्त एवं ओज प्राणायतन पर आधारित है ।*

*जिन्हें रसवह (ओज) ,प्राणवह एवं रक्तवह स्रोतस में आपने समाहित किया है ।*

*जिसको आपने इस ज्ञान को बहुत ही बेहतरीन जिज्ञासा एवं ज्ञान को उल्लेख किया है ।*

इन्द्रिय शारीर इसमे महत्वपूर्ण है इस विषय पर मनन की जरूरत है ।
🌹🌹🙏🏼🌹🌹🙏🏼🌹🌹

[1/2, 11:07 AM] Vaidya Sanjay P. Chhajed: 

बहोत ही बेहतरीन गिरीराज जी, जैसे हमने आपसे अपेक्षा की थी। आप सभी ग्रंथविद हो। आपसे एक और अनुरोध है। 
शरीर के अलग-अलग अवयवों में  जो भी capillary जालक से वह अलग-अलग होता हैं। उनके कार्य भी अलग-अलग होते हैं। हरेक जालक में होने वाली विकृतियां और उनसे होनेवाली बिमारिया भी अलग-अलग हैं। इसपर भी विचार करना चाहिए।

[1/2, 6:09 PM] Dr. Pawan Madan:

 Namaste sanjay sir !

Very nicely compliled article shared by you. Thanks a lot.

With due respect I want to draw your attention to the fact that the the structures described in Shastras bear more importance physiologically rather than anatomically. 
This means that you will find such references where the word Dhamani would also mean nerve some time ... whenever the function of a tubular structure would be similar as Dhamnyaat.... it would be called as Dhamani.

Similar references could be found for Siraa, Srotas and even for Kandra.
🙏🙏
We cant fix an entity described in  shastra to only one prticular entity like you opined... Srotas equivalent to capillaries.
🙏🙏

Similarly is the four types of Srotodushti. Vimaargagaman is not only the Capillary leak syndrkme but it may refer to many other pathological processes.

Thanks 🙏🙏

[1/2, 6:37 PM] Vaidya Sanjay P. Chhajed: 

Thank you Pawan ji, I was really expecting inputs from you. 
But unfortunately it's too superficial.

I have never said that these are anatomical equivalents, rather I wanted to draw attention of all the learned scholars and Gurudev to the fact that
ध्मानात् धमन्या
स्त्रवणात्  स्रोतांसि
सरणात् सिरा 
Are all stated in one phrase  at ch. Su. 30 where components of blood circulation including Heart and dash mahamoola are mentioned and as reference provided by Giriraj sir that these are not seperable and also strotas and dhamani are sira vikara.

I understand fully that strotas can not be only an anotomical  entity, it has some inputs of physiology and a peculiar role in pathogenesis. 

When I quoted capillaries b, it does carry a specific meaning.

First capillaries needed to be understood in its dual role  providing nutrition and collecting waste.   This happens because of altered pressure. 
Secondly there are 3 distinct anatomical types of capillaries, having different functions. Moreover if you closely the observed  capillary beds of
Brain
Eyes
Nasal & paranasal area
Lungs
Heart
Liver & spleen
Stomach
Intestines
Kidney
Testicles
Penis
Uterus

Are not only different anatomically but has very distinct physiological functions.  Though cappilary is 2 cell thick  entity anatomically but is very different at every organ, the permeability alters, the function alters.

We can observe certain pathology in these organs which can be attributed to capillary involment.

We also get the 3 strotodushti laxna viz- Atipravritti, Sanga & Vimarggaman. 
The 4th one we get Siranaam granthi as venous thrombosis, which rarely occurs at Artery and capillarries.

I have not made any statement as such, but put forth an hypothesis to the August gathering of Kayasampraday, it's open for discussion. But I am slowly getting more convinced.

I would request you and all other scholars to get the references of
Sira
Dhamani
Strotas 
Used in different contexts and try to draw a conclusion.

I am neither scholar of shastra, nor well read. I have read only charak samhita and some part of hriday. So for me getting references across grantha is not only difficult but actually impossible.

[1/2, 7:25 PM] Dr. Pawan Madan: 

Sir...🙏🙏

That means you have pointed to one part / factor....that capillaries come under the category of Srotas.

I was busy in Opd and as such I have not memorized the refereces but I just presented my understanding.

There has been very good discussions on the Siraa Dhamani and Srotas in the group earlier and probably the discussion is available in blog or if not I will search my laptop, if I have that discussion copied, I will try to send .

Thanks..🙏

[1/2, 7:47 PM] Dr. D. C. Katoch sir: 

Well explained Acharya Sanjay. Further, it has to be understood that inflow towards the tissues is the function of Dhamani and outflow from the tissues is the function of Sira, whereas Srotas is responsible for dual function of simultaneous sravan and parinaman 
(oozing/permeation/
filtration/transfusion and transformation).

[1/2, 9:37 PM] Vaidya Sanjay P. Chhajed: 

I have not pointed out that capillaries comes under srotas, my hypothesis is that *can we consider capillary beds as srotas*?.

[1/2, 9:42 PM] Dr. D. C. Katoch sir: 

Arterial end of capillaries equates to Dhamani, Venous end to Sira and the capillary network to srotas jaal.

[1/2, 9:42 PM] Vaidya Sanjay P. Chhajed: 

Sir , I think you are making a small mistake - inflow towards the tissues is done by Arteries but actual supply to the tissue happens at capillaries ; similarly the outflow from the tissues happens only at the later part of capillaries, it is then passed on to veins.
Arteries and veins are non permeable but capillaries are semipermeable and as dual pressure system in the arterial half it is the positive pressure & the venous part it has the negative pressure. Besides that there is variations in the permeability / access to the nutrient material  and the waste at different organs.

[1/2, 9:45 PM] Vaidya Sanjay P. Chhajed: 

Even the capillaries have that kind of dual function while transportation of the material.

[1/2, 9:46 PM] Dr. D. C. Katoch sir:

 There is no mistake Vaidyavar. Transfusion/Exchange called Sravan happens in capillaries and cells.

[1/2, 9:47 PM] Prof. Giriraj Sharma: 

आदरणीय 
सिर्फ एक जिज्ञासा है
आचार्य चरक मतानुसार स्रोतस का प्रत्यात्म लक्षण क्या है ?

[1/2, 9:48 PM] Dr. D. C. Katoch sir: 

Sravan is not transportation, Charak says so.

[1/2, 9:55 PM] Vaidya Sanjay P. Chhajed:

 स्रवणात् स्रोतांसि  ।

[1/2, 9:55 PM] Prof. Giriraj Sharma: 

Transportation
Transfusion
Permeability
Secretion  
इनमें से आप क्या मान रहे है ?

[1/2, 10:02 PM] Vaidya Ashok Rathod Oman: 

🙏🙏🙏🌹🌹🌹
मूलात् खादन्तरं देहे प्रसृतं त्वभिवाहि यत् |
स्त्रोतस्तदिति विज्ञेयं सिराधमनिवर्जितम् ||१३||  - सुश्रुतसंहिता शारीरस्थानम् - ९

[1/2, 10:11 PM] Dr. D. C. Katoch sir: 

💡 sahi light daali hai.

[1/2, 10:12 PM] Dr. Surendra A. Soni:

 महामहिम आचार्य संजय जी !

नमो नमः ।🙏🏻🌹

आपको स्रोतस के ऊपर लिखे गए महालेख के लिए हार्दिक अभिनंदन और आभार ।
प्रत्येक चिकित्सक को इसको अच्छी तरह से समझना चाहिए तो स्रोतोमय पुरूष में होने वाली सभी व्याधियों की सम्प्राप्तियाँ बहुत अच्छी तरह से समझी जाएगी ।

I would like to add few of my hypothesis based on available description that    is mainly as per modern structural concept while there is another untouched aspect yet to be explored as well as explained and that is as per 13 srotasas mentioned by Acharya charak. The apparatus that used for said 13 srotasas are same as you mentioned in details but as per ancient classical description there is possibility of specific chennal for specific dhatu. We all know that endocrine glands does affect the sroras esp dhatuvah srotas and that chennal yet to be understood as per ayurved.
In the leadership of Great physiologist like Mahamahim Arun ji, you and we all collectively may make efforts to understand this concept properly.
Besides 4 types of mentioned srotodushti, srotas get affected at every moment of life like exercise, cold-warm environment, Dosha gati, aahar-pak, Avasthapak etc. All the movements of Dosha dhatu and malas are mainly in All srotasas inspired and controlled by Vat. If we see the all 4 types of srotodushti in modern pathology then we find that almost all 4 srotodushti types are present in almost all types of diseases at cellular levels. This concept yet to be elaborated properly.

You see example of tamak shwas...

1. Sang- at bronchioles
2. Vimarg gaman- pratilom vayu
3. Atipravritti- malarup shleshma and shwas gati
4. Siragranthi- structural changes in bronchioles

Srotasas are mainly responsible for dhatugatatva for a disease...

देहे स्रोतांसि रिक्तानि पूरयित्वाsनिलो बली ।

We would continue this interested discussion sharpened by Sanjay Sir.

🙏🏻🌹

[1/2, 10:15 PM] Dr. Pawan Madan: 

Nevermind sanjay sir.
You have put forth your view and it is in discussion.

[1/2, 10:18 PM] Dr. Pawan Madan: 

Srotas have been said different from Sira and dhamani by Sushrut seen from the angle of surgical description.

At physiological level.....the dhamani may sometime act as Siraa or siraa many act as srotas.

E.g. at cellular level.
Every cell wall has srotas.
That means every cell of a tule has srotas. So need to look in terms of physiological actions.

[1/2, 10:18 PM] Dr. Surendra A. Soni: 

We will have to include all mucrochennals under the sroras whether are superficial capillary system or intraorgenic micro circulation.

🙏🏻🌹

[1/2, 10:21 PM] Dr. Pawan Madan: 

🙏👌🏻🙏👌🏻🙏🙏

The four types act in every nook and corner of the body....gross as well as microscopic.

The 13 srotas cant be figured as separate entities.
The same tube or tubule carrying a particular dosh dhatu mal or other factor could be taken as that particular vahasrotas.

😌😌
[1/2, 10:25 PM] Vaidya Ashok Rathod Oman:

 🙏🙏🙏🌹🌹🌹
आचार्य जी आपकी संमती से इसे भी जोड दिया तो स्रोतस दुष्टी का foundation बन जायेगा।

*चरकसंहिता विमानस्थानम् - ५. स्रोतोविमानम् -सर्वस्रोतसां साधारणं प्रदूषककारणम्*

आहारश्च विहारश्च यः स्याद्दोषगुणैः समः|
धातुभिर्विगुणश्चापि स्रोतसां स प्रदूषकः||२३||

[1/2, 10:28 PM] Vaidya Ashok Rathod Oman:

 🙏🙏🙏🌹🌹🌹 सुश्रुत जी ने व्याधीचिकित्साए भी बतायी है उनको सिर्फ शल्य चिकित्सक ही बना के रखना योग्य है क्या आचार्य जी ?

[1/2, 10:50 PM] Vd. Divyesh Desai Surat:

 कई सारे रोगों में आचार्य सुश्रुत जी के चिकित्सा सूत्र ओर चिकित्सा काय चिकित्सको से भी अव्वल दर्जे की है,  षडविध क्रियाकाल भी आचार्य सुश्रुत की देन है,
स्वस्थ की व्याख्या भी सुश्रुत संहिता की बेहतरीन है,
प्रतिश्याय की चिकित्सा भी सुश्रुत ने  अच्छी तरह बताई है, ऐसे कई  पॉइंट्स है, जो सुश्रुत आचार्य को न केवल शल्य चिकित्सक किन्तु काय चिकित्सक के रूप में भी अव्वल दर्जे का स्थान मिलना चाहिए, सर आपकी ये बात के साथ मे पूर्ण रूप से सहमत हूँ।।🙏🏾🙏🏾
असाध्य रोगों में भी सुश्रुत आचार्य ने आतुर के प्राण बचाते हुए चिकित्सा करने का निर्देश किया है, जो अभी के समय मे भी उपयुक्त है।।
जय आयुर्वेद, जय धन्वंतरि
जय सुश्रुताचार्य, जय चरकाचार्य🌺🌺🌺

[1/2, 10:54 PM] Dr. D. C. Katoch sir: 

Fully agreed. Charak Samhita by and large provides Clinical Philosophy  whereas Sushrut Samhita is replete with Clinical Medicine.

[1/2, 10:55 PM] Vd. Divyesh Desai Surat: 🙏🏾🙏🏾

[1/2, 11:32 PM] Dr. Pawan Madan: 

Very right.
Many times without Sushrut we cant understand many principles and many of his treatments are superb.

But when it comes to the description of anatomy....he has seen and described everything from the surgical angle.

That was my point sir.
🙏🙏🙏😌

[1/2, 11:33 PM] Dr. Pawan Madan: 

Very right and wonderful...🙏🙏🙏

[1/3, 5:20 AM] Vaidya Sanjay P. Chhajed: 

🙏🙏🙏🙏🙏 आदरणीय सोनी सर, एक छोटी सी बिनती, यह आचार्य उपनाम हमारी व्यवसायीक व्यवस्था है, जैसे की आप सभी जानते हैं की हम एक छोटे-से प्रैक्टिशनर है। कभी पढ़ाया नहीं, नहीं संहीताओ को अभ्यासीत किया। जब आप श्री इस तरीके के शब्द प्रयोग करते हैं तो, हम बड़ा बौने से महसूस करते हैं।
नाड़ी परीक्षा को समझने हेतु गुरुदेव तपनजी तथा सौमेंद्रजी के निर्देशन पर च.सू.३० को पुनः अवलोकन करना शुरू किया तो अनायास ही यह विचार मन में आया। इसपर और पढ़ते हुए अधिक साम्य नजर आने लगा , तब हमने यह सोचकर की क्यों न काय संप्रदाय की विद्वत समाज में इसे रखा जाए। खासकर गिरीराज सर, और अरुण सर जो शरीरविद है। और भी सभी गुरुजनों के तथा आयुर्वेद चिकीत्सकों के सम्मुख जिससे मंथन हो। 
जितने भी संदर्भ मिले हैं वह श्री गिरिराज सर की मेहरबानी है।

पहली बात हम स्त्रोतस= capillaries नहीं कहते हैं।
परंतु स्त्रोतस की संकल्पना में इस जालक तथा वहां होने वाले कार्य का, गुणात्मक, रचनात्मक होने वाले परिवर्तनों को नजरअंदाज नहीं किया जा सकता है।  तो इस विषय पर और अधिक गहन चर्चा होनी चाहिए, आपके इस मत के लिए धन्यवाद।

[1/3, 5:25 AM] Vaidya Sanjay P. Chhajed: 

क्या आप इस से अधिक स्पष्ट कर सकते हैं? क्यों की स्त्रोतस तो सिरा एवं धमनी से अविभाग है। 

विवर्जित शब्द की निरूक्ती तथा ससंदर्भीत अर्थ इसे अधिक स्पष्ट करेगा यह हमारा मत है।

[1/3, 5:26 AM] Vaidya Sanjay P. Chhajed: 

Yes sir, I also feel same.

[1/3, 5:30 AM] Vaidya Sanjay P. Chhajed: 

Wonderful, may i request Khandel sir who is one of the strong pillars of the group to put some light on the subject. Also a humble request to respected Gurdipsingh sir, Gaur sir, Subhash sir , Chulet sir to enlighten us.

[1/3, 9:35 AM] Dr. Gurdip Sing Sir: 

Srotas literally means opening as 9 and 3 out side openings and innumerable inside opening. Then in cells through which materials for metabolism get in. Systems as pranavaha rasavaha etc and so on. it is physiological pratyang and found in life only. so It has to be considered as per context.


[1/3, 9:47 AM] Prof. Giriraj Sharma: 

सुप्रभातम 
आप का विमर्श 
*अपि चैके स्रोतसामेव समुदयं पुरुषमिच्छन्ति,* के परिपेक्ष्य में यदि यह सब
Transportation
Transfusion
Permeability
Secretion  आप इस अर्थ में ग्रहण कर रहे है तो,,,,

परन्तु आचार्य चरक सुश्रुत ने इन स्रोतस का लोक से लेकर सूक्ष्म तक एवं सूक्ष्म से लेकर लोक तक विवेचन किया है ।
आप *अपि चैके स्रोतसामेव समुदयं पुरुषमिच्छन्ति के परिपेक्ष्य में उचित विवेचन कर रहे है ।

परन्तु आचार्यो ने अति विशिष्ट सूक्ष्म प्रत्यात्म लक्षण स्त्रवण तक इसे विवेचित किया है ।

*यावन्तः पुरुषे (मूर्तिमन्तो) भावविशेषास्तावन्त एवास्मिन् स्रोतसां प्रकारविशेषाः|*
*सर्वे हि भावा पुरुषे नान्तरेण स्रोतांस्यभिनिर्वर्तन्ते, क्षयं वाऽप्यभिगच्छन्ति*|
         ⬇️

*स्रोतांसि खलु परिणाममापद्यमानानां धातूनामभिवाहीनि भवन्त्ययनार्थेन*
          ⏬

*अपि चैके स्रोतसामेव समुदयं पुरुषमिच्छन्ति, सर्वगतत्वात् सर्वसरत्वाच्च* *दोषप्रकोपणप्रशमनानाम्|*
*न त्वेतदेवं, यस्य हि स्रोतांसि,* यच्च [२]*  *वहन्ति, यच्चावहन्ति,*
*यत्र चावस्थितानि, सर्वं तदन्यत्तेभ्यः||४||*
*अतिबहुत्वात् खलु केचिदपरिसङ्ख्येयान्याचक्षते स्रोतांसि, परिसङ्ख्येयानि पुनरन्ये||५*

⬇️

*मूलात् खादन्तरं देहे प्रसृतं त्वभिवाहि यत् |*
*स्त्रोतस्तदिति विज्ञेयं सिराधमनिवर्जितम् ||*
            ⏬

*स्त्रवणात स्त्रोतांसी*
🙏🏼🙏🏼🙏🏼🌹🙏🏼🙏🏼🙏🏼

[1/3, 10:38 AM] Dr. D. C. Katoch sir: 

"Gurunaam Guru Dwe Ayurvedgyanvahanaam Srotomoole "

[1/3, 10:44 AM] Vaidya Sanjay P. Chhajed: 

प्रणाम गुरुवर, क्या सभी स्रोतसों को किसी बाह्य या अभ्यंतर छेद के रूप में ग्रहण करना चाहिए ? फिर सूत्रस्थान ३० में इन तीनों को एक साथ लिखने का औचित्य क्या होना चाहिए ? बड़े सवाल मन में उठे हैं।

[1/3, 12:00 PM] Prof.Vd.Arun Rathi: 

*आप क्यों my hypothesis कह रहे हो भाऊसा संजय जी !
आचार्य चरक और सुश्रुत ने स्वयं हदय, धमनियों और रक्तवाहिनियों को स्त्रोतस् मूल मे कहा है।*

*धमनी और रक्तवाहिनियों का समावेश स्त्रोतस् मे है*




[1/3, 12:24 PM] Vaidya Shrikrishna Khandel Sir:

 Right !
Srotas are for  transfer the sthir or finally transformed dhatu by various mode like infiltration permeation oozing etc. 
Sira and dhamni both are vascular measure but at the remote end terminal tails of dhamni and sira are turned into capillaries which are carrying transform and transfer the fluids in colonies of tissues organs simultaneously carry excreta bag to excretory or recycling organs 
Thats why sushrut seen it and differ with charak sira dhamni are not srotas no of sira is 700 dhamni is 24 srotas are 13 or 11 pair so the corrections done by shushruta.

[1/3, 12:33 PM] Prof.Vd.Arun Rathi: 

*आचार्य सुश्रुत ने स्वयं रक्त वह स्त्रोतस् मूल मे रक्तवाहिनियों को भी कहाँ है*

[1/3, 1:03 PM] Prof.Vd.Arun Rathi: 

*प्रिय अनुज*
*आप से विनती है, कम से कम मुझे महामहिम लिखना बन्द करे.*
*आप ने ख वैगुण्य पर तमक श्वास का अच्छा उदहारण दिया है*
*इसी तरह से वृक्क रोग, यकृत विकार, हदयरोग, अर्बुद आदि व्याधियों की सम्प्राप्ति को समझना चाहिए*
*ख वैगुण्य यह व्याधि के पुर्वरुप, रूपावस्था, भेदावस्था, उपद्रव, अरिष्ट, उदर्क आदि सभी अवस्था मे रहेगा।*
*लीन दोषावस्था उदाहरण बिज दोष, विरुद्धाहार आदि मे ख वैगुण्य शरीर में उपस्थित रहता है*

[1/3, 1:07 PM] Prof.Vd.Arun Rathi:

 *प्रणाम खाण्डल गुरवर*
🙏🏻🙏🏻🙏🏻

[1/3, 1:10 PM] Dr. Pawan Madan: 🙏🙏🙏🙏🙏

[1/3, 1:15 PM] Vaidya Ashok Rathod Oman: 🙏🙏🙏🌹🌹🌹

[1/3, 1:19 PM] Dr. D. C. Katoch sir: 

Srotas itself is a functional entity specifically meant for sravan karm (i.e. oozing, transfusion, filtration etc leading to transformation of the materials into new form/composition. Whereas Srotomools are anatomical structures/channels/organs to produce, carry,  transport or expel specific moieties in the body system.


[1/3, 1:25 PM] Prof.Vd.Arun Rathi:


चरक विमान ५ /९.


[1/3, 1:27 PM] Prof.Vd.Arun Rathi:

*यहाँ पर आचार्य चरक ने स्त्रोतस् के पर्याय वाची शब्दों मे सिरा, धमनी, रसवाहिन्य आदि दिये है।*


[1/3, 1:29 PM] Dr. D. C. Katoch sir: 

Yeh sab Srotas ke samanya synonyms hain aakashiye bhavon/sthanon ke liye,   vishist karm nahin.

[1/3, 1:29 PM] Prof.Vd.Arun Rathi:

 *प्रणाम सरजी*

🙏🏻🙏🏻🙏🏻

[1/3, 1:31 PM] Vd. Atul J. Kale: 

RASA-RAKTA SAMVAHANA :
As the blood pressure is a disease of circulatory system and Ayurveda doctrine believes in same constituents in morbid as well as physiological conditions by the only difference of normalcy and vitiation, it becomes necessary to discuss about Ayurvedic circulatory system.
Mula of Rasavaha srotasa is Hridaya and Dasha Dhamani.(Ch.Vi.5/8) Sushruta has accepted Hridaya and Dhamani as Mula Sthana for Pranavaha and Rasavaha Srotasa both. Sushruta has also accepted Raktavahi Dhamani as a Mula sthana for Raktavaha and Mansavaha Srotasa both.(Su.Sha.10/12). In the other hand Sushruta has described the arterio-venous system in 7th chapter of Sharira by the name of Sira. Therefore it is essential to discuss about the following organs as a part of circulatory system
Srotasas taking part in circulation
Sr. No. Srotasas
1. Rasavahi Srotasa
2. Raktavahi Srotasa
3. Manovahi Srotasa
Charaka has quoted Hridaya as the Mulasthana of Dasha Dhamani but Sushruta has given two opinions for the Mulasthana of Dhamani at different places along with their no. and directions :
Sr. No.
Mula Sthana of Dhamani No. of Dhamanis as per direction
1. Hridaya
(Su.Su. 14/3) Urdhvaga Dasha
Adhoga Dasha
Tiryagga Dasha
2. Nabhiprabhava
(Su.Sha.9/4) Urdhvaga Dasha
Adhoga Dasha
Tiryagga Dasha
According to Charaka Vata, Pitta, Kapha and Satva have no specific Srotasa and these entities circulate through all channels over the body, but Sushruta has differentiated the individual circulatory paths for Tridoshas through Sira and Dhamani.

Sira and Dhamani according to (Sushruta Sha. 9/4)
Sr. No. Sira Type No. Particulars
1. Vatavahi 10 Vatasthana Gatanam
2. Pittavahi 10 Pittasthane
3. Kaphavahi 10 Kaphasthane
4. Raktavahi 10 Yakrita-Plinho

Sr. No. Dhamani Type No.
1. Vatavahi 2
2. Pittavahi 2
3. Kaphavahi 2
4. Raktavahi 2
5. Rasavahi 2
Nevertheless Sushruta again has described that there is no specification about the circulation of particular Dosha through particular Sira, in fact all the Sira are Sarvavaha (Su.Sha.7/16). 
Along with Rasa Dhatu following entities circulate in the body
(Su.Su. 14/13, Ch.Vi. 5/7)
Sr. No. Circulating entities
1. Rasa
2. Rakta
3. Tridosha
4. Satva
5.Sapta-Dhatu   Poshakanshas
6. Kleda
Vata is complete in its constitution, but according to the different places it has different functional role and nomenclatures. In the function of circulatory system Vata works in unison with the help of other entities i.e. Sadhaka Pitta and Avalambaka Kapha.
Hridaya is the seat of Vyana Vayu, Sadhaka Pitta, Avalambaka Kapha, Mana and Chetana. Thus it can be concluded that these entities have vital role in circulation along with other types of Vayu.



Following are the Doshas and their types taking part in the circulation
Sr. No. Dosha Types of Dosha
1. Vata Vyana
Prana
Udana
Samana
Apana
2. Pitta Sadhaka
3. Kapha Avalambaka
Under scatter references all Vayu are seem to be essential for the physiology of circulation in unison. Description is given below :
 Vyana Vayu :
The Mulasthana of Rasavaha Srotasa  is Hridaya (Heart) and Dasha Dhamani (Arteries) (C.Vi.5) With the help of Vyana Vayu heart contracts and do the Vikshepana of Rasa -Rakta in Ekakala (Ugapata) and continuously to all over the body.(Ch.Ch.15/36). Hence nourishment of all Dhatus(tissues) take place continuously and in circulatory fashion(Ch.Ch15/21). Here Charaka has indirectly elaborated the theory of closed blood circulation. In this closed circulation certain entities circulate to nourish, to expel out excretory products and thus to maintain the homeostasis of body physiology.
In 15th chapter of Sutra. Sushruta implied that the Karma of Vyana Vayu is Praspandana. Thus conclusion can be made here that sympathetic nervous system remains under the control of Vyana Vayu and thus alterations in the physiological condition of Vyana Vayu can lead EHT.(S.Su.15/1)

 Prana Vayu :
Charaka has implicated Shira (Ch.Su.17/12) and Hridaya as the seat of Prana (Ch.Ch.28/6), later on Vagbhata has described that the Karma of Prana is the Dharana of Hridaya, Chitta, Buddhi and Indriya. After exertion, if one is opposing natural urge of extra air then he will suffer from Gulma, Hridroga and Sanmoha.(Ch.Su.7/24) Thus Prana Vayu is very closely related to the Karmas of Hridaya. By taking all reference in consideration along with ‘the Mula Sthana of Pranavaha Srotasa is Hridaya and Mahasrotasa’, it can be concluded that Prana is having the vital role in the control of Hridaya which organizes all the functions in Shira.   Thus Prana Vayu has an important role in the function of circulatory system.
 Udana Vayu :
Main seat of Udana Vayu is Urasthana (Ch.Ch.28/8), it does mean that the organs present in Urasthana are having the control of Udana Vayu. In the same context Charaka enunciates the Karmas of Udana Vayu which are Prayatna (Efforts), Urja (Energy), Bala (Strength) and Varna (Complexion) Poshana. All these Karmas are performed in whole body. All the organs, directly or indirectly do their function (Efforts) with the help of Energy and Strength provided by Udana. Heart beats approximately one lakh time per day, for this effort heart needs extra strength and energy which is provided by Udana. Contraction and relaxation of heart is continuous without taking the rest, it means there is more Chala Guna is present in the heart which is the Karma of Vyana Vayu but to alleviate the bad effects of this continuous movement Udana Vayu and Avalambaka Kapha have the vital role in maintenance.
 Samana Vayu :
The seat of Samana Vayu is Sweda, Dosha and Ambuvahi Srotasa (Ch.Ch.28/8) But before, Charaka himself has implicated that Tridosha and Mana has no specific Srotasa although these entities circulates  through all channels of body. (in Ch.Vi.5/7). In one reference Sushruta has implied that there is no specificity in Sira concern to circulation of Dosha, nevertheless these Siras are Sarvavaha including Tridosha (Su.Sha. 7/16). Therefore conclusion can be made that Samana may have an important role in the circulation.
According to Sharangadhara after the digestion process Samana helps in the transportation of Rasa in to the heart.(Sh.M.6/9) and from heart it circulates in the whole body.
 Apana Vayu  :
Normal functions of all Dosha including all Vayu are depending upon the normal direction of Apana Vayu. In Arsha Chikitsa Charaka has quoted that if there is obstruction in the path of Apana Vayu i.e. in Gudasthana then the reverse direction of Apana Vayu vitiates Samana, Vyana, Prana, Udana Vayu and Pitta-Shleshma. Thus it can be said that all the normal functions of body are depend upon the normalcy of Apana Vayu. Thus indirectly the circulatory functions remain under the control of Apana Vayu.
 Sadhaka Pitta :
The main seat of Sadhaka Pitta is Hridaya.(As.H.12/13). Vagbhata has elucided intellectuals, grasping and ego as the functions of Sadhaka Pitta. These all functions are very much related to mind. If these functions are not in normal state can alter the cardiac output and vascular resistance.
 Avalambaka Kapha :
The main seat of Avalambaka Kapha is Urasthana. It nourishes and supports all Kaphasthana (Avalambana) with the help of its Ambukarma property (As.H.12/15).  Thus it supports and nourishes heart to maintain circulatory system physiological balance. 
In this way all the Dosha takes part in the circulation and any morbidity appertain to them can lead the disease which is related to the circulatory system.
Sushruta has described the circulatory system in most flourish form. Mainly all the Acharya described the circulation of Rasa Dhatu only but we should consider Rakta along with Rasa. Rasa-Rakta Samvahana is multidirectional and its kinetics varies by the area of the body and according to the need of Sapta –Dhatu. Sushruta has tried to elaborate the theory of circulation on the basis of direction, thus 
Rasa, the resultant of digestion and very micro in nature circulates all over the body in three directions i.e. (Su.Su.14/3)
• Upward (Urdhvaga)
• Downward (Adhoga) and 
• Other than upward and downward direction (Tiryagga)
This quotation indicates the directions of Rasa Dhatu circulation, in the other reference Sushruta has given examples of Shabda, Archi and Jala for the indication of the same.
Thus circulation takes place in three directional way. (Su.Su.14/16: Dalhana)
• Shabdasantanavata  :  Tiryagagamitvam
• Archisantanavata     :    Urdhvagamitvam
• Jalasantanavata        :  Adhogamitvam
Here Dalhana has commented about direction of Rasa and it is seem to be more logical. Many other learned people correlate Shabdadi Gati with the actual Kinetics of Rasa and Rakta. In fact these two opinions are very important in the contribution of the Avarana concept related to EHT. Thus
• The direction and
• The intensity of kinetic force
These two factors concerned to Vata are having great pathophysiological importance in the Avarana process. Blood pressure in different places is different this can be understood by Rasa-Rakta Samvahana based on Shabda, Archi and Jala Santanavata and which is mainly related to the  intensity of kinetic force (cardiac output). If any how there is any alteration in the direction and kinetic force of Vata then it may lead high or low blood pressure. These conditions may be altered by the obstruction of there natural direction and kinetics leading hypertension.
[1/3, 1:33 PM] Dr. Pawan Madan: Any Siraa .... if the Parinaman ...function is happening in that..... may be called as Srotas for that particular reference.

This may be understood like this.

🙏

[1/3, 1:33 PM] Prof. Giriraj Sharma: 

*Life and Ayurveda principles are so deep and subtle they can not be understood with one stroke*
*आदरणीय  गुरुदीप सिंह जी*
🙏🏼🙏🏼🙏🏼🌹🙏🏼🙏🏼🙏🏼

[1/3, 1:38 PM] Dr. BK Mishra: 

👍🏼 स्रोतस का विकृतिविज्ञानीय व्यावहारिक स्वरूप तो यही है, जहां पर *खवैगुण्य* घटित होता है...🌹🌹

[1/3, 1:38 PM] Dr. D. C. Katoch sir: 

What a practitioner will achieve or attempt if all the entities synonymous  for Srotas are considered one as referred in Charak ?  Charak had been more philosophical than practical in his approach to explain the fundamentals of Ayurveda.

[1/3, 1:39 PM] Prof.Vd.Arun Rathi:

 सही है, 
*As Cell are the structural and Functional unit of body.*
*स्त्रोतस् also works in same manners*
*स्त्रोतस् मुल भी यही कार्य करते है*
*विशेष संदर्भ मे स्त्रोतस् कार्य अलग अलग होगा*

*उदाहरण के स्वरुप रसवाहिन्य और रक्तविहिन्य धातुओं के पोषकतत्व ( अस्थाई धातु )का वहन करती है*
🙏🏻🙏🏻🙏🏻

[1/3, 1:44 PM] Prof.Vd.Arun Rathi: 

*सिराओं व्दारा किट्ट भाग का वहन भी किया जाता है, यही किट्ट भाग मल वह स्त्रोतस् मे ले जाये जाते है और यथा समय मलवहस्त्रोतसों व्दारा शरीर के बाहर उत्सर्जित किये जाते है*

[1/3, 1:49 PM] Prof. Giriraj Sharma: 

*आदरणीय सिरा शब्द को बहुत सामान्य ले रहे है आप ,,,*
*सिरावर्ण विभाग, सिराव्यध, अर्धचिकित्सा जैसे शब्द आचार्य सुश्रुत ने उल्लेख किये है, ।।*

 *सिराव्यधश्चिकित्सार्धं शल्यतन्त्रे प्रकीर्तितः |*
*यथा प्रणिहितः सम्यग्बस्तिः कायचिकित्सिते ||२३*

[1/3, 2:25 PM] Dr. Bhadresh Naik Gujarat: 

Yes sir ✔️
I have experience of thousand patients after single siravedh
Instant results I have seen
May samprati vighatan take place.

[1/3, 2:26 PM] Prof.Vd.Arun Rathi: 

*मै आयुर्वेद मे कहे किसी भी शब्द को सामन्य मे नही लेता, बहोत सामन्य की तो बात ही नही है*
*वैद्यराज पवनजी मदान की पोष्ट पर अपना मन्तव्य देते हुए यह लिखा है,""सिराओं व्दारा किट्ट भाग का भी वहन किया जाता है.*

Prof. Giriraj  ji !

[1/3, 2:42 PM] Prof. Giriraj Sharma: 👌🏻👌🏻🌹👍🏻

[1/3, 3:35 PM] Vaidya Sanjay P. Chhajed: 

सिरा धमनी विवर्जित: का मतलब सिरा और धमनी के व्यतिरीक्त नहीं परंतु उनसे छुपाए हुआ ले सकते हैं।


[1/3, 4:47 PM] Dr. D. C. Katoch sir: 

Kaun si Sirayein hain vo ?

[1/3, 5:02 PM] Dr. Bharat Padhar: 

प्रत्येक स्रोत के मूल से उत्पन्न होने वाले मल को मलायन तक सिरायें ले जाती है एवं वहाँ से मलों का सरण शरीर के बाहर होता है।

[1/3, 5:05 PM] Dr. Bharat Padhar: 

स्रोतस के विभिन्न पर्याय arteries, veins, capillaries, nerves, channels, various types of receptors, etc ko ingit karte hai.

[1/3, 5:11 PM] Dr. D. C. Katoch sir: 

Gayi bhains paani mein 😔

[1/3, 5:12 PM] Prof. Giriraj Sharma: 

🙏🏼🙏🏼

[1/3, 5:13 PM] Dr. Bharat Padhar: 

प्रत्येक मूल में स्रोतस होगा ही, लेकिन कार्य के अनुसार वो कभी सिरा, धमनी, स्रोतस इत्यादि से पहचाना जाएगा।

[1/3, 5:14 PM] Dr. D. C. Katoch sir: 

Receptors sambhavtya aakashiye bhav nahin hai Dr Bharat.

[1/3, 5:15 PM] Dr. Bharat Padhar: 

Yes sir, they are not akashiya bhav but they can produce akash when stimulated.
 Or can allow to pass something creating space when stimulated by specific triggers.
 Otherwise they keeps the doors closed for entry.

[1/3, 5:16 PM] Dr. D. C. Katoch sir: 

Dekho Bhai- Jo khud aaya vo Khuda, theek usi prakar jahan sravan (oozing, transfusion, permeation, filtration etc) ho vo Srotas.

[1/3, 5:16 PM] Prof. Giriraj Sharma: 

आज कटोच सर, हम आपको ही सुनेंगे पढ़ेंगे रात भर ,,,,
😄😄😄😄🙏🏼

[1/3, 5:17 PM] Dr. Bharat Padhar: 🙏🏻🙏🏻🙏🏻

[1/3, 5:19 PM] Dr. D. C. Katoch sir: 

Ratri jagran ki jaroorat hi nahi. Raat ko tamsik bhav gyanarjan nahin karva sakate. Din mein hi kar lo jo karana hai.

[1/3, 5:19 PM] Dr. Bharat Padhar: 

Glucose transporter 2 also known as solute carrier family 2, member 2 is a transmembrane carrier protein that enables protein facilitated glucose movement across cell membranes

This is also type of ayan.
 Same way for other receptors too...They help in process of Ayana.
 यह भी अयन का एक उदाहरण है। यहाँ पर स्रवन नही हो रहा है, नही ध्मापन ओर नही सरण। अतः इस प्रक्रिया को बताने हेतु आचार्यो ने संवृत असंवृत इत्यादि स्रोतस के पर्याय दिए।
अयन शब्द अति विस्तृत है, अतः इसमे बहोत प्रकार के पाश्चत्य चिकित्सा शास्त्र के सर्कुलेशन, perfusion, transportation, transducers etc समाविष्ट हो जाते है।

[1/3, 5:44 PM] Dr. Bharat Padhar: 

Ion channel-linked receptors bind a ligand and open a channel through the membrane that allows specific ions to pass through. To form a channel, this type of cell-surface receptor has an extensive membrane-spanning region. When a ligand binds to the extracellular region of the channel, there is a conformational change in the proteins structure that allows ions such as sodium, calcium, magnesium, and hydrogen to pass through.

[1/3, 5:48 PM] Dr Mansukh R Mangukiya Gujarat: 

👌👌 Vaidya Kale sir

[1/3, 5:49 PM] Dr Mansukh R Mangukiya Gujarat: 

🙏 prof. Giriraj Sharma sir

[1/3, 5:51 PM] Dr. Pawan Madan: 

🙏🙏🙏

Srotas is a physiological entity.

Any entity can work as srotas at different times and at different places or locations.

Keeping and understanding all synonyms under the main category of Srotas may give a practioner an open view to understand the physiology behind the pathology of the presenting complaint or disease.

This is my understanding, please guide sir if wrong.

🙏🙏

[1/3, 6:04 PM] Dr. Pawan Madan: 

👏🏻👏🏻👏🏻👏🏻👏🏻

Sravan and Transformation also perhaps....

[1/3, 6:04 PM] Dr. Bharat Padhar: 

In type 2 diabetes mellitus there is dysfunction of srotasa specifically Marga which leads to insulin resistance and don't allow pass the glucose in to cell and thus Marga Avarodh janya Vata Prakop occurs, which flushes out glucose through Urine (Vimarga Gaman), Extra water is absorbed from cell leads to excessive urination which is Atipravruti. Less entry of glucose in cell leads to Sanga at cellular level. So, depending on site presentation of srotovikriti differs.

[1/3, 6:07 PM] Dr. Pawan Madan:

 Namaste....🙏🙏

Post by Respected Gurdeep Guru ji

👇👇

Srotas literally means opening as 9 and 3 out side openings and innumerable inside opening. Then in cells through which materials for metabolism get in. Systems as pranavaha rasavaha etc and so on. it is physiological prayang and found in life only. so It has to be considered as per context.

[1/3, 6:10 PM] Dr. Bharat Padhar: 

स्रोतस को सामान्य एवम विशेष भेद से समझने पर सभी समस्या का समाधान हो सकता है।
जैसे निदान पंचक में निदान।
स्रोतस सामान्य अर्थ: परिणाम आपद्यमान धातु का अयन
विशेष अर्थ- स्रवन करने वाला
सामान्य अर्थ संज्ञक स्रोतस में धमनी, शिरा, विशेष स्रोतों संज्ञक स्रोतस, मार्ग, पंथ, छिद्र, संवृत असंवृत इत्यादि समाविष्ट है।

[1/3, 6:15 PM] Dr. Pawan Madan: 

Pratyang, Part or structure or entity .....

[1/3, 6:16 PM] Dr. Bharat Padhar: 

स्रोतोमयं अयं शरीरं, यहाँ पर सामान्य स्रोतस की बात है।
शिरा धमनी विवर्जित, यहां पर विशेष स्रोतस की बात है।

[1/3, 6:19 PM] +91 79863 61270:

 Yes, it is same, that is srotas are structural entities, may be macroscopic or microscopic (histological )or we may just confirm by the function they do.

[1/3, 6:29 PM] Dr. Pawan Madan: 

SROTAS – 

स्रोतस की शास्त्रीय परिभाषा के अनुसार

⚜️
-स्रवणात स्रोतान्सि – 
चरक सूत्र ३०/१२
-स्रवणात इति रसादेव पोष्यस्य स्रवणात – चरक सूत्र ३०/१२ पर चक्रपाणि
-धातुवाहीनि रसादिधातून यानि वहन्ति – अष्टान्ग ह्र्द्य शारीर ३/४१ पर अरुणदत्त
⚜️


SROTAS is the name to all the collective passages, paths – seen or unseen – through which 
*something flows ,,,any DRAVYA,,, seen or unseen.*

👉👉👉👉
तेषां तु मलप्रसादाख्यानां धातूनां स्रोतांस्ययनमुखानि| 
तानि यथाविभागेन यथास्वं धातूनापूरयन्ति – चरक सम्हिता २८/५
*They have been classified as the KARMA they do. Sometimes there can be overlapping E.g. sometimes RASAVAHA must also be doing the same work as RAKTAVAHA SROTAS. One can`t make exact differentiation. PRANAVAHA and UDAKAVAH have been named observing the importance. Any SROTAS carrying the life force can be called as PRANAVAHA when without its existence body can’t sustain life. 


Most imp point to note...👇👇👇
यावन्तः पुरुषे मूर्तिमन्तो भावविशेषास्तावन्त एवास्मिन् स्रोतसां प्रकारविशेषाः| 
सर्वे हि भावा पुरुषे नान्तरेण स्रोतांस्यभिनिर्वर्तन्ते, क्षयं वाऽप्यभिगच्छन्ति| 
स्रोतांसि खलु *परिणाममापद्यमानानां* धातूनामभिवाहीनि भवन्त्ययनार्थेन||३|| - चरक सम्हिता विमान ५/३
🟠🟠
*One peculiar feature of SROTAS is the process of transformation occurring during the flow of DRAVYA (seen or unseen).*

 In that even some gateways in cell wall can also be called as SROTAS. Even a cell has contents moving in and out and in the process of transformation and can be called as SROTAS.

🔷🔷🔷

Srotas are
मुर्तिमन्था ,,, structural entities (they can be N number of types anatomically)

परीणाम आपाद्यमाना,,,,, means where the transformation is undergoing.

🙏🙏🙏

[1/3, 7:44 PM] Prof. Giriraj Sharma:

 मूल सूत्र  में 
*यावन्ते मूर्तिमन्त भाव विशेषा*
है या 
*यावन्ते भाव विशेषा,,,,,*

[1/3, 7:48 PM] Dr. Pawan Madan: 

Ji ye sandarbha copy paste hi kiyaa hai...🙏


[1/3, 8:00 PM] Prof.Vd.Arun Rathi:

*सिरा प्रमाण संग्रहः*                                                                      *पुरुषविचयः*                                           *लेखक : प्रो.विनायक जयानंद ठाकर.*






[1/3, 8:03 PM] Prof.Vd.Arun Rathi:

 *सिरा, धमनी और स्त्रोतस्, पुरुषविचयः इस किताब या ग्रन्थ  मे भी पढना चाहिए.*

[1/3, 9:25 PM] Vaidya Ashok Rathod Oman: 

आचार्य जी, सिरा एवं धमनी की संख्या बतायी गयी हैं। स्रोतस असंख्य बताये गये हैं। संहिताओ में जिन स्रोतस के वर्णन किये गये हैं वे केवल महत्वपूर्ण स्रोतस में गिने गये हैं। यह मुझे समझ आया। इसपर कुछ प्रकाश डाले।

[1/3, 9:32 PM] Vaidya Ashok Rathod Oman: 

अतिबहुत्वात् खलु केचिदपरिसङ्ख्येयान्याचक्षते स्रोतांसि, परिसङ्ख्येयानि पुनरन्ये||५||  तेषां तु खलु स्रोतसां यथास्थूलं कतिचित्प्रकारान्मूलतश्च प्रकोपविज्ञानतश्चानुव्याख्यास्यामः; ये भविष्यन्त्यलमनुक्तार्थज्ञानाय ज्ञानवतां, विज्ञानाय चाज्ञानवताम्||६||  चरकसंहिता - विमानस्थानम् - ५. स्रोतोविमानम्  स्रोतसां भेदाः

[1/3, 9:42 PM] Prof. Giriraj Sharma: 

प्राणवह स्रोतस में प्राण मूर्त है या अमूर्त,,,,
प्राण एकादश
प्राण वायु
यहां प्राण में क्या अर्थ ग्रहण करे ,,,,
मूर्त के परिपेक्ष्य में ,,,,
इसलिए मूर्तिमन्त भाव विशेष कह कर आचार्य,,,,

🙏🏼🙏🏼🙏🏼

[1/3, 9:48 PM] Vaidya Sanjay P. Chhajed: 

बहोत सही चर्चा चल रही है, अभी तो गुरुदेवो ने हिस्सा नहीं लिया है।

[1/3, 10:02 PM] Dr. Bharat Padhar: 

संज्ञावह, मनोवह इत्यादि में भी यही मूर्त एवम अमूर्त का issue आएगा सर जी।

[1/3, 10:03 PM] Dr. Bharat Padhar:

 एक बार मूर्त एवम अमूर्त पे भी अच्छी चर्चा चली थी। वह संदर्भ भी उपयोगी होगा इस परिपेक्ष्य में।

[1/3, 10:29 PM] Dr. BK Mishra:

 प्राणवायु
 प्राणवहानाम् इति प्राणसंज्ञक वातवहानाम्
   चक्रपाणि (च वि 5/8)

[1/3, 10:33 PM] Prof. Giriraj Sharma: 

मूर्तिमन्त भाव है क्या ?

[1/3, 10:45 PM] Dr. BK Mishra: 

अमूर्त

[1/3, 10:49 PM] Prof. Giriraj Sharma: 

फिर प्राणवह स्रोतस को आचार्य ने 
यावन्ते मूर्तिमन्त भाव विशेषा,,,
कहा होगा ,,,
या यावन्ते भाव विशेषा 
मूल पाठ में 
🙏🏼🙏🏼🙏🏼

[1/3, 10:56 PM] Dr. BK Mishra: 

मूलपाठ में तो 
यावन्तः *पुरुषे मूर्तिमन्तो भाव विशेषाः* ही है..
मूर्तिमन्तो से अर्थ *असर्वगतद्रव्यपरिणामवन्तो* लिखा है

[1/3, 10:58 PM] Prof. Giriraj Sharma: 

इससे गुरु वचन से ही समझा जा सकता है ।।
🙏🏼🙏🏼🙏🏼🙏🏼🙏🏼🙏🏼🙏🏼

[1/4, 7:48 AM] +91 79863 61270: 

Vayu is amoortiman dravya, may it be pranavayu, not bhav.

[1/4, 9:55 AM] Dr. Bharat Padhar: 

इस अर्थ से तो सिर्फ आत्मा, काल, दिशा एवम आकाश ही अमूर्त है। बाकी सब मूर्त है।

[1/4, 9:57 AM] Dr. Rajeshwar Chopde: 

This is right
Mana bhi Amoorta hai

[1/4, 10:06 AM] Dr. Bharat Padhar: 

मन सर्वव्यापक नही है, मन के गुण अणुत्व एवम एकत्व है। अतः वह अमूर्त की परिभाषा को पूर्ण नही कर रहा है।

[1/4, 10:07 AM] Dr. Bharat Padhar: 

इसीलिए मनोवह स्रोतस भी है।

[1/4, 10:16 AM] +91 79863 61270: 

Manovahi srotas are sarvashreergata , Charak

[1/4, 10:29 AM] Dr. Bharat Padhar:

 सभी स्रोतस सर्व शरीर गत ही है 

[1/4, 10:43 AM] Vaidya Sanjay P. Chhajed: 

Man - anupraman

[1/4, 12:12 PM] +91 79863 61270: 

Can capillary system be assumed as one type of Srotas ? 
One more question in same reference is, why capillary system can not be assumed as one type of Srotas ?
Another question is, if it can be assumed as one type of Srotas, then  in which type of Srotas we can list it and why ?
Thanks

[1/4, 12:31 PM] Dr. Gurdip Sing Sir: 

In Su.sharir 9 Shloka 10 it is said that as naturally in mrinal & bis the kha are present similarly in Dhamani kha are present by which upachaya of rasa is done. Hindi translater gives upachaya mean grahan & Dalhana says virya of abhayang parisek etc also. If the translation is correct then it is clear that in dhamani srotas are present & which is capalaries. on this basis you can name capabilities as dhamani srotas and to venules as sira srotas.

[1/4, 1:09 PM] Prof. Giriraj Sharma: 

सिराधमनीनामणुशः प्रविभज्यमानानां मुखाग्रपरिमाणं

[1/4, 2:08 PM] Vaidya Sanjay P. Chhajed: 

My question was whether *Capillary system is strotas ?* As it matches the references and information along with the activities in both physiological and pathological conditions


[1/4, 2:30 PM] Vaidya Ashok Rathod Oman:



[1/4, 2:32 PM] Vaidya Ashok Rathod Oman: 

अब इससे भी अधिक स्पष्ट और क्या हो सकता हैं ?

[1/4, 2:34 PM] Vaidya Ashok Rathod Oman: 

यादवजी त्रिकमजी और डॉ घाणेकर जी की टीका हमारी सदैव मार्गदर्शन मे समर्थ रही हैं।🙏🌹

[1/4, 2:45 PM] Prof. Giriraj Sharma: 

Capillaries धमनी नाम अणुश,,,,
Venules सिरानाम अणुश,,,,,

[1/4, 2:46 PM] Prof. Giriraj Sharma:

 स्रोतः ---- 
सिरा धमनी विवर्जितम आचार्य  सुश्रुत

[1/4, 2:48 PM] Prof. Giriraj Sharma: 

Capillaries bed and venules beds are not Srotas as per Aachary Sushrut,,,,,

 रक्तधरा कला
Capillaries bed and venules beds कला as per Aachary Sushrut,,,,,

भवतश्चात्र-
यथा हि सारः काष्ठेषु छिद्यमानेषु दृश्यते |
तथाहि धातुर्मांसेषु छिद्यमानेषु दृश्यते ||६||

यथा बिसमृणालानि विवर्धन्ते समन्ततः |
भूमौ पङ्कोदकस्थानि तथा मांसे सिरादयः

द्वितीया रक्तधरा मांसस्याभ्यन्तरतः, तस्यां शोणितं विशेषतश्च सिरासु यकृत्प्लीह्नोश्च भवति [१३] ||१०||

वृक्षाद्यथाभिप्रहतात् क्षीरिणः क्षीरिमावहेत् [१४] |
मांसादेवं क्षतात् क्षिप्रं शोणितं सम्प्रसिच्यते [१५] |


[1/4, 3:17 PM] Dr. Gurdip Sing Sir: 

Yes capillaries in dhamani are its srotas.

[1/4, 3:20 PM] Prof.Vd.Arun Rathi: 

*मेरी भी एक जिज्ञासा है, क्या शरीर के विविध अंग प्रत्यांग मे उपस्थित  "Vascular Bed" Functionally and Structurally Same होते है.*

*For Ex. : 
1. Vascular Bed in Eyes that to present in *Retina*
2. Vascular Bed present in *Liver*
3. Vascular Bed Present in *Brain*
4.Vascular Bed Present in *kidney* 
And so on are same Structurally and Functionally.
🙏🏻🙏🏻🙏🏻


[1/4, 3:38 PM] Vaidya Sanjay P. Chhajed: 

Reference please ?

[1/4, 3:42 PM] Prof. Giriraj Sharma:

 चरक शारीर 7/13

[1/4, 3:57 PM] Vaidya Sanjay P. Chhajed: 

Capillary bed or the mesh is different at different organs and perform different functions. Their permeability varies, so the nutrients passing through them alters; the same is true for removing the waste. There are different sets of capillaries having different absorption and the threshold to remove the waste at various sites. eg there is blood brain barrier, is there any barrier elsewhere in the body or when glomerular capillaries use the threshold of, 180mg%, which other organ has that? There are multiple examples, every organ is different capillary wise.

[1/4, 4:14 PM] Vd Raghuram Y. S, Banguluru: 

*Wonderful discussions on srotas vs capillaries topic* 🙏🙏💐

💐💐💐💐💐💐💐

*My additions (avoiding what has been discussed & theoretical details)* 👇

💐💐💐💐💐💐💐

👉 *Srotas synonyms include sira, dhamani. Sira-Dhamani network consists of structures resembling arterioles & venules - which comprise of ingredients of capillary bed*

👉 *Anuni i.e. microscopic - is one of the shapes and structures of srotas as mentioned by master Charaka. Anu srotas can be considered as capillaries. Sravana anyways is the key function of srotas and also happens in anu srotas.*

👉 *Hrdaya is explained to provide the entire body with nutrition through the functions of vyana vayu. Hrdaya itself is the moola of rasavaha srotas. The root meaning of hrdaya which includes hru & da which denotes give and take phenomenon ➡️ arterial & venous system ➡️ which also includes the arterioles and venules ➡️ which denotes capillary bed ➡️ anu srotas (by shape & srotas used as synonym to dhamani & sira)*

👉 *"Sravanaat Srotamsi" - Sravanaat rasaadi sraava pathatvaat srotamsi (Ganghadara)*
*Sravanaat iti rasaadihi iva poshakasya sravanaat (Chakrapani)*
Justifies the action at capillary bed, especially of arterioles. 

👉 *Indirect reference by inference* - *Yavantah hi moortimanto bhava visheshah tavanta Eva asmin srotasaam prakara visheshah* Cha.Vi.

👉 *Rasavahe Dwe Tayoh mulam hrdayam rasa vahinyascha dhamanyah* - Sushruta
*Rasavahanam moolam hrdayam rasavahinyascha dhamanyah*

The use of srotas and dhamani in the same context here - rasavahini dhamanis are moola of rasavaha srotas. 

👉 In the mechanism of *Kedara-Kulya Nyaya* wherein kulya which also means passage or ducts, if taken as microchannels of nutrition at tissue or cellular level, can be considered as capillary bed. Kulya - can be considered as srotas (anu srotas) here = capillary bed. 

👉 *Functions of dhamani - include puryate shariram (spread out all through the body) & poshyate va shariram (nourishes every bit of the body)*

👉 *Explanation of Tiryak Gata Dhamanis - They being 4 in number branch and re-branch into innumerable branches of dhamanis. Arterioles? Capillary Bed?*

👉 *Lotus Stalk example / analogy to explain the structure of dhamani - Dhamanis, just like lotus stalk have many minute pores through which the rasa infiltrates - action at capillary bed*

👉 *Indirect reference - Marmani mamsa sira snayu asthi sannipata - here let us focus on the word sannipata, the junction. So the concept of meeting place of many structures was well known. If we take sira separately (not in the context of marma), sira sannipata = capillary bed?*

👉 *The concept of kala, especially raktadhara kala can also be considered to understand the concealed capillary bed*

👉 *Sira perspective* Master Sushruta tells that there are a total number of 700 siras in the human body. The functions of the siras are to provide nutrition to every part of the body. Therefore they are spread out in the entire body and reach every corner of the body so as to provide nutrition. 
This has been explained with the help of suitable examples so as to make the concept of siras easy to understand. 
✔ *Understanding the functions of Siras with examples of water carriers and small channels* - Just like the women water carriers sprinkle and feed water to every plant in the garden / grove and just like the small canals / channels carrying water convey water to every part of the field and nourish the crops therein, the siras spread all through the body and nourish all structures therein. 
✔ *Understanding the structure of sira with examples of veins of leaves* – The siras are spread over the entire body. They branch and re-branch as they spread out. Just like the veins and venules are spread over the leaf, the siras spread all through the body.

If we carefully observe the meanings of the above mentioned 3 examples, the examples explain 3 different kinds of siras. 

1. Water carrier - Larger siras – probably the 700 siras mentioned by Sushruta

2. Channels / small canals conveying water to the plants / crops in the fields - the secondary branches of the above said 700 siras, siras of moderate size – size of these siras are lesser / smaller than the larger siras

3. Veins and venules of the leaves - 
The tertiary branches of the siras, the smallest / minute branches which probably branch out from the moderate sized siras mentioned above 

Note – *If sira is taken as vein*, the study should be done in a reverse way. Analogy / example 3 explains the formation of venules at the capillary level, joining to become the secondary tributaries (analogy / example 2). These secondary tributaries join together to become larger veins (water carrier example / analogy 1) which drain into the heart. If we need to use the word artery for sira we can also consider the word ‘branches’ instead of ‘tributaries’ and  their distribution can be understood in a descending way i.e. analogy 1 to analogy 3. 

*Also pittavaha and raktavaha siras explained by Sushruta indicates veins & arteries respectively. Since both are types of siras, their intersection / sannipata can take place - when we combine the above said analogies - capillary bed?*

💐💐💐💐💐💐💐

*This is my small understanding and interpretation* 

*With due respect to the classical reference which tells that srotas are different from siras and dhamanis and also considering the fact that they have been used as synonyms at certain places - contextual use and understanding of terms*

*Taking liberty of yukti and anumana / anumiti while respecting aptopadesha, a slight manipulation with courage, which needs to be done  while discussing such topics*

*Thanks to hon Sanjay sir fir raising this question*🙏💐❤

*Humble submission to KS elite platform*🙏🙏💐❤

💐💐💐💐💐💐💐

*_DrRaghuram.Y.S._*

[1/4, 4:20 PM] Prof.Vd.Arun Rathi: 

🙏🏻🙏🏻🙏🏻
Thanks

[1/4, 4:36 PM] Vaidya Shrikrishna Khandel Sir: 

Thanks Raghuram nicely concluded with classical reference and yukti
Just adding two three more pts
1. Srotas are transporters transformers and dispensers 
2. Srotas are platforms of pathology when any one or two or all three functions get hampered 
3. Srotas are diluters of mega energy produced in mahasrotas powerhouse for capacity of tiny cells
4. By long travelling in spiral bands(srotanshi deerghani pratan sadrish )like electricity production in mega giga units and travel in long lines through transformers for domestic supply.

[1/4, 4:46 PM] Vd Raghuram Y. S, Banguluru: 

Thanks so much Guruvar🙏💐❤️for your blessings..
Excellent additions dear sir...lovely analogies...and awesome perspectives👏🙏💐❤️
Thanks for enriching our knowledge with your unparalleled wisdom and constantly guiding us sir🙏

[1/4, 4:49 PM] Prof.Vd.Arun Rathi: 👌🏻👌🏻👌🏻

🌹🌹🌹

My humble query

*Siras and Dhamanis can be part of sortas or not*


[1/4, 4:51 PM] Prof. Giriraj Sharma:

सिरा सन्निपाता,,,,


[1/4, 4:51 PM] Dr. Bhadresh Naik Gujarat: 

Respected sir !
Kindly explain about raktmoxan effect on Srotas ?
Does Raktmoxan remove strotoavarodh?
Does remove obstruction to nourishment to organ?


[1/4, 4:53 PM] Prof. Giriraj Sharma:

आचार्य चरक की स्रोतः परिभाषा यहां कैसे उपयुक्त होगी,,,

[1/4, 4:54 PM] Prof. Giriraj Sharma: 

धातु की अवस्था परिवर्तन,,

[1/4, 4:54 PM] Prof.Vd.Arun Rathi: 

*Sushruta as a Surgeon aware of Anatomy want to tell that their is slight difference in structural and functional property of Vascular Bed at different site or organ in body.*

[1/4, 4:59 PM] Dr. BK Mishra: 

I am not Anatomist , but In my opinion Siras and Dhamanis  are part of srotas.. Because खवैगुण्य and स्रोतोदुष्टि is found in those..🙏🏼

[1/4, 5:07 PM] Vd Raghuram Y. S, Banguluru: 

🙏🙏🙏💐
If we are looking at and limiting to *can be* rather than *should be* yes...I can't jump into conclusive remark since there are mutually contradicting statements in the shastra itself, by various acharyas. *Can be*


[1/4, 5:11 PM] Vd Raghuram Y. S, Banguluru: 

Why not sir? We are not discussing only Acharya Sushruta here. And then it is one of the points I hv mentioned, value addition to other points. This doesn't explain all or conclude everything. And my points aren't conclusive. Experts may put forth their extended views and differ from me. I too am a learner and would gracefully accept meaningful contradictions and acceptable conclusions. 🙏💐

[1/4, 5:58 PM] Prof.Vd.Arun Rathi: 👍🏻

*Before, Conclusion "Can be" is Ok*

[1/4, 6:03 PM] Prof. Giriraj Sharma: 

आचार्य चरक एवं सुश्रुत को स्रोतस पर उनके सूत्र अगर देखे तो आचार्य सुश्रुत ने स्त्रवण शब्द उल्लेख नही किया और सिरा धमनी विवर्जित लिखा,,,
आचार्य चरक ने स्त्रवण लिखा तो सिरा धमनी पर कुछ  नही लिखा पर धातु आद्य,,, लिख कर संकेत कर दिया कि वहन ही नही धातु मलादि निर्माण प्रक्रिया भी है ।
जो सिरा में सिर्फ सरण का निर्देश किया 
उसी सिरा के सिराधमनीनामणुशः प्रविभज्यमानानां मुखाग्रपरिमाणं कह दिया जो लगभग capillaries के समक्ष कह दिया वहां भी सरणः ही होना चाहिए संभवत 

बाकी आयुर्वेद जितना गंभीर है उतना ही सतही ही भी,,,,
🙏🏼🙏🏼🙏🏼🌹🙏🏼🙏🏼🙏🏼

[1/4, 6:10 PM] Dr. BK Mishra: 

👍🏼Great analysis 🌹🙏🏼

[1/4, 6:23 PM] Prof.Vd.Arun Rathi:

 *सुश्रुतसंहिता के शरीर स्थान अध्याय ९ मे स्त्रोतस् का शल्यतंत्र मे अधिकार स्वरुप वर्णन करते हुए यह वर्णन मिलता है*

*प्राणान्नोदक- रस-रक्त-मांस- मेदो -  मुत्र - पुरीष - शुक्रार्त्तववहानि:, येष्वधिकार:, एकेषां बहुनि:, एतेषां विशेषा बहवः ।*
सु. शा. अ / ९ / १२.

*उदाहरणार्थ :*
१. *तत्र प्राणवहे द्वे तयोर्मूलं ह्रदयं रसवाहिन्यश्च (प्राणवाहिन्यश्च) धमन्यः,... ।*
सु. शा. अ ९ / १२.

२. *रसवहे द्वे, तयोर्मूलं ह्रदयं रसवाहिन्यश्च धमन्य:,.... ।*
सु. शा. अ. ९ / १२.

*इसे प्रत्येक्ष मे किस तरह समझे.*

[1/4, 6:26 PM] Dr. Pawan Madan: 

🙏🙏

If the capillaries fit as per the definition of srotas, they can be surely said as srotas.

If the function of परीणाम आपद्यामान is present , surely they are srotas..

🙏

 Any tubule/ aperture / opening / way which is carrying some entity and where the transformation is being happening is Srotas.

[1/4, 6:32 PM] Dr. Pawan Madan: 

Pranaam Guru ji khandal Sab !

🙏🙏🙏🙏🙏🙏🙏🙏🙏🙏🙏🙏🙏

....when there is no transformation while being tronsported, it is not Srotas
...and yes....dispensers....अयनार्थे
🙏🙏

Wonderful similys...
🙏🙏🙏

[1/4, 6:33 PM] Dr. Pawan Madan: 

Can be



Conditions Apply..😊


With conditions of Parnaam aapadyaamaana and ayanaarthe.
😊

[1/4, 6:59 PM] Dr. D. C. Katoch sir: 

Brilliant vivechan has clarified that Srotas, Sira and Dhamani are common theoretically but  distinct from each other functionally.  Great analysis Dr Raghu- kudos. 👍🏼👍🏼

[1/4, 7:09 PM] Prof. Giriraj Sharma: 

😄😄😄
सादर नमस्कार आचार्य श्री
*आ अब लौट चले*

तत्र केचिदाहुः- सिराधमनीस्रोतसामविभागः, सिराविकारा एव हि धमन्यः स्रोतांसि चेति |
तत्तु न सम्यक्, अन्या एव हि धमन्यः स्रोतांसि च सिराभ्यः; कस्मात्?
 व्यञ्जनान्यत्वान्, मूलसन्नियमात्, 
कर्मवैशेष्यात्, 
आगमाच्च; 
केवलं तु परस्परसन्निकर्षात् सदृशागमकर्मत्वात् 
सौक्ष्म्याच्च *विभक्तकर्मणामप्यविभाग इव कर्मसु भवति ||३||*

[1/4, 8:13 PM] Dr. D. C. Katoch sir: 

Kyo theek hai na Rachana Sharir Vishesgya. Srotas agar anatomical structures hote to unka ullekh Charak Samhita ke Shaarir Sthan mein swantantra roop se avashya hota.

[1/4, 8:18 PM] Dr. BK Mishra: 

आचार्यश्री इतना ही समझ लें तो पर्याप्त है कि जहां भी संवहन, धातुपोषण प्रक्रिया, स्रोतोवैगुण्य तथा स्रोतोदुष्टि होता है या सम्भव वही व्यावहारिक दृष्ट्या स्रोतस हैं, जिनका की स्वास्थ्य तथा चिकित्सा से सीधा सम्बन्ध है ...

[1/4, 8:30 PM] Prof. Giriraj Sharma: 

🙏🏼🙏🏼🙏🏼😷🙏🏼🙏🏼🙏🏼
सुश्रुत शारीर 
शारीरे सुश्रुत श्रेष्ठ,,,
स्रोतस विध्द लक्षण,,,,
विध्द रचनात्मक इकाई में ही सम्भव है ।
आचार्य श्रेष्ठ इतना जल्दी कहाँ अनुगामी होता है आपका ढीठ शिष्य
😄😄🙏🏼😄😄🙏🏼😄😄

[1/4, 8:31 PM] Dr. BK Mishra: 

प्रत्येक *ख* के लिये समझना अधिक उपयुक्त होगा...🙏🏼

[1/4, 8:32 PM] Prof. Giriraj Sharma: 

ख एवं खादान्तर 
एक है या अलग

[1/4, 8:33 PM] Dr. BK Mishra: 

ख वैगुण्य दोषों से नहीं, (बाह्य )निदान से होता है🙏🏼

[1/4, 8:36 PM] Dr. BK Mishra: 

मेरी अल्पमति से तो दोनों त्रयोदश स्रोतस में समाविष्ट हैं...

[1/4, 8:37 PM] Prof. Giriraj Sharma: 

अर्थात है तो दो ही,,,
आपके इसका यह अर्थ ग्रहण करूँ

[1/4, 8:38 PM] Dr. BK Mishra: 

दोनों एक ही भाव को व्यक्त करते हैं, यह अर्थ ग्रहण करें..

[1/4, 8:53 PM] Vaidya Sanjay P. Chhajed: 🙏🙏🙏🙏🙏🙏

[1/4, 8:57 PM] Prof. Giriraj Sharma: 

ख    
आदान्तर निवृतपथमिति,,,,

[1/4, 8:58 PM] Dr. D. C. Katoch sir: 

Tathastu!

[1/4, 9:03 PM] Dr. D. C. Katoch sir: 

Mujhe pata hai viddha lakshan srotas ki kriyatmak ikai ke vikrit hone se hi vyakt hote hain.

[1/4, 9:04 PM] Prof. Giriraj Sharma: 

विध्द आश्रय है आश्रय तो मूर्त ही है

[1/4, 9:08 PM] Prof. Giriraj Sharma:

 स्वेदवह स्रोतस का वर्णन 
क्रियात्मक इकाई ही है पूर्ण

[1/4, 9:09 PM] Vaidya Sanjay P. Chhajed: 

When I put forth the hypothesis, I was not aware that it can be so detailed, expressed in the samhitas so vividly
I must thank
Prof. Gurdipsingh sir

Prof. Khandel sir
Dr. D. C. Katoch sir
Prof. Dr. Arun Rathi ji
Prof. Surendra Soniji
Dr. Pawan Madanji
Prof. Giriraj Sharma sir,
B. K. Mishraji 
Raghuramji
And every one  else who has participated, added value to this discussion. I was  really thrilled to know .  

Please continue discussion, 🙏🙏🙏🙏🙏

[1/4, 9:10 PM] Prof. Giriraj Sharma: 

दोषदुष्य समुर्छना जहाँ आचार्य चरक ने बताई तो स्वेदवह बताया मगर आचार्य सुश्रुत ने विध्द परिपेक्ष्य में स्वेदवह का वर्णन नही किया ।

[1/4, 9:12 PM] Prof. Giriraj Sharma: 🙏🏼🙏🏼

[1/4, 9:15 PM] Dr. D. C. Katoch sir: 

Sushrut ki drishti ne svedvah srotas ko shayad srotas nahin samajha. 😅

[1/4, 9:17 PM] Prof. Giriraj Sharma: 

ज्ञानादान्तर 
😄😄😄🙏🏼😄😄😄

[1/4, 9:24 PM] Dr. D. C. Katoch sir: 

Theek hai - Sushrut ke shishya sadhak pitta se dekhane ki apeksha aakashiye srotas se sunane mein zyada daksh hain.

[1/4, 9:25 PM] Prof. Giriraj Sharma: 

सु  श्रुत
संजय सर 
फिर डांट देंगे 
इन घिसी पिटी चर्चाओं के लिये 
इसलिये अब ......

ॐ शांति !!

[1/4, 9:37 PM] Vd Raghuram Y. S, Banguluru: 

Thanks so much for your kind words dear sir...you hv indeed beautifully summed up my narration in 2 lines...Thanks again Guruvar🙏💐❤️🙏

[1/4, 10:00 PM] Vd. Divyesh Desai Surat: 

ब्रह्मांड/ पिंड के न्याय के हिसाबसे सुषुम्ना/पिंगला/इड़ा नाड़ी और सप्त चक्र 
सहस्त्राधार
आज्ञाहत
विशुद्धि
अनाहत
मणिपुर
स्वाधिष्ठान
मूलाधार 
चक्र को भी स्रोतस कह सकते है? आदरणीय गुरुजनो ने इतना सारा capillary बेड का डिटेल्स देकर स्रोतस पर डिसकस किया है तो, हरेक चक्र  में भी  और
मर्म पॉइंट  में भी  
स्रोतस नजर आने लगा है।।
🙏🏾🙏🏾स्रोतस मय पुरुष की बात हर एक एंगल से prove होती है।।👏🏻👏🏻
जय आयुर्वेद, जय धन्वंतरि
जय स्रोतस मय पुरुष की🙏🏾
नाड़ी गुरु संजय सर ने बहोत ही अच्छा पॉइंट देकर नया विचारने का एंगल दिया है
धन्यवाद सभी गुरुजनो का और सबको अंतःकरण से शत शत नमन👏🏻👏🏻

[1/4, 10:28 PM] Dr. BK Mishra:

 क्या चक्रों में भी वैगुण्य तथा सङ्गः आदि चतुर्विध दुष्टि होती है...

[1/4, 10:40 PM] Dr. Pawan Madan: 

There is  need to understand the division of Dhamni, Sira and Srotas on the basis of the functions they perform and not on anatomical basis only.

[1/4, 10:42 PM] Dr. Gurdip Sing Sir:

 It seems to me that Khavaigunya occurs at cellular level and srotodushti at the sthul level I. e.  system level as evident from the symptoms of various srotodushti in texts. Khavaigunya causes dosh prakop which in turn produces srotodushti. The point may be discussed.

[1/4, 10:43 PM] Dr. Pawan Madan: 

Definitely srotas are rachnatamak but the point is.....which anatomical part to be called as Srotas. 
The discussion is this only..

And the reply has been given repeatedly and even reply has been given emphatically by Resp Gurdip sir and Resp Khandel sir also.

[1/4, 10:46 PM] Dr. Pawan Madan:

 क्या किसी चक्र में स्रवण, परीणाम आपद्यामांता व अयनार्थे का कार्य होता है ?

[1/4, 10:52 PM] Prof. Giriraj Sharma: 

जिस धातु आदि के नाम से जो स्रोतस है उस धातु की आद्य अवस्था से परिणाम अवस्था तक जिस जिस अंग अवयव में  वह धातु आदि अपनी पूर्ण अवस्था को प्राप्त करते है वो सब अंग प्रत्यंग उस स्रोतस के रचनात्मक इकाई है जिनमे मूल अंग प्रत्यगो का आचार्यो ने उल्लेखित कर दिया ।

[1/4, 10:54 PM] Dr. Gurdip Sing Sir: 

kha is empty space because it is synonym of akash also. At cellular level these holes don't allow everything to enter, Vata controls it. In modern it is Na k system and receptors also have very much role. Hence help of physiology and pathology may be taken to understand the phenomenon.

[1/4, 10:58 PM] Dr. Pawan Madan: 

👏🏻👏🏻👍👍

And this can include many different entities. And even the same one entity can be there in more than one type.

[1/4, 10:59 PM] Dr. Pawan Madan:

 🙏🙏🙏🙏🙏🙏

[1/4, 11:10 PM] Prof. Giriraj Sharma: 

आचार्य चरक के मेद अस्थि मज्जावह स्रोतस के मूल में 
👏🏻👏🏻👍👍

And this can include many different entities. And even the same one entity can be there in more than one type.
यह सब मिल जाएंगे


[1/5, 12:06 AM] Vd. Divyesh Desai Surat: 

प्रणाम, सर🙏🏾🙏🏾
चक्रो में भी विगुणता आ सकती है और अतिप्रवृति, संग, विमार्गगमन की *अनुभूति* हो सकती है,
आज दूसरे ग्रुप में चक्रो और नाड़ी पे चर्चा हो रही थी, इस लिए मेरे मन मे ये प्रश्न आया, science के हिसाब से इसका उत्तर ना है, लेकिन आज जो चक्र के विशेषज्ञ की बात सुनी तो अनायास तुलना हो गई और चक्रो के कार्य, आकार के बारे में सुना तो capillary Bed का चित्र मस्तिष्क में आ गया
नाड़ी शास्त्र में
सुषुम्णा को वात से
पिंगला को पित्त से और
इड़ा नाड़ी को कफ के कंट्रोल करने वाली नाड़ी बताई गई है, तो ये प्रश्न मन में आया, अगर प्रश्न गलत है तो क्षमा चाहता हूं, ये केपिलरी का डिटेल्स नाड़ी गुरु आचार्य संजय जी ने बताया तो, सुषुम्णा, पिंगला और इड़ा ये चक्रो में से ही पास होती है, चक्रो में भी सब भावों की अनुभूति हो सकती है,  आज चक्रो में गुजराती भाषा में डिसकस हुई तो में बिना ट्रांसलेट किये ये पॉइंट यहाँ रखता हूँ, तो चक्रो से कौन से अंगों का कंट्रोल हो सकता है, ये ही बात बताई है, तो हमारे नाड़ी गुरु संजय सर ही ये विषय को ट्रांसलेट करके और भी अच्छी तरह समझा सकते है, क्योंकि गुरुजी की सारी भाषा मे पकड़ है।
🙏🏾🙏🏾

[1/5, 12:51 AM] Vaidyaraj Subhash Sharma: 

*अद्वितीय एवं अभूतपूर्व चर्चा स्रोतस पर, सभी विद्वानों का ह्रदय से आभार 👌👍❤️🙏*

[1/5, 12:54 AM] Vaidyaraj Subhash Sharma: 

*स्रोतस पर चर्चा का महत्व इसलिये और अधिक है कि वृद्ध दोषों को शाखाओं से कोष्ठ में लाने के लिये स्रोतस की भूमिका अति महत्व रखती है, 

'वृद्ध्या विष्यन्दनात् पाकात् स्रोतोमुखविशोधनात् शाखा मुक्त्वा मलाः कोष्ठं यान्ति वायोश्च निग्रहात्' 

च सू 28/33 

दोषों की वृद्धि होने पर, परिपाक हो जाये तब, वात को रोकने से और स्रोतस के मुखों के शोधन से दोष रसादि धातुओं को त्याग कर कोष्ठ में चले आते है जिस से इन वृद्ध दोषों का निर्हरण सरल हो जाता है। ।*

*स्रोतस को उनके कार्यों के अनुसार अगर ले कर चलें तो ...*
*प्राण, अन्न और उदकवाही स्रोतस बाह्य पदार्थों को ग्रहण कर रहे हैं।*
*मूत्र, पुरीष, आर्तव और स्वेदवाही स्रोतस त्याज्य पदार्थों का शरीर से निर्हरण कर रहे है, यहां हम स्तन्यवाही स्रोतस को अभी नहीं ले कर चले हैं।*
*शरीर का पोषण करने वाले स्रोतस रस, रक्त, मांस, मेद, अस्थि, मज्जा, शुक्र, ओजोवाही स्रोतस हैं।*
*सब से महत्वपूर्ण चिंतन, विचारवान मनोवाही स्रोतस, इसके अतिरिक्त संज्ञावाही, चेतनावाही, दोषवाही आदि अन्य स्रोतस ।*

*लगातार  चल रही चर्चा में अनेक मत ऐसे हैं जो विभिन्न तर्कों एवं प्रमाणों से सिद्ध किये जा सकते है जैसे अमूर्त भाव भी है, चेतना और मन की सत्ता भी क्योंकि आयुर्वेद मे अमूर्त भाव भी चिकित्सा का ही भाग है। ऐसे अनेक भाव हैं जो दिखेंगे नही पर इन्द्रियों से ही आप अनुभव कर सकते हैं क्योंकि उनके भी स्रोतस है जो अत्यन्त अणु रूप में है।*

[1/5, 12:55 AM] Vaidyaraj Subhash Sharma: 

*मन को 'अस्पर्शमणु चानन्तं' कहा गया है अर्थात यह अणु रूप और स्पर्श रहित है, शरीर के बाह्य करण ज्ञानेन्द्रियों में कर्ण, त्वचा, चक्षु, रसना और घ्राण हैं तथा कर्मेन्द्रियों में हस्त, पाद, गुदा, उपस्थ और वाक् हैं।*

*अन्तः करण में मन, बुद्धि और अहंकार हैं । आत्मा का संबंध तो सभी इन्द्रियों से सदैव ही बना रहता है पर उन्हें समस्त विषयों का हर पल ज्ञान नही बना रहता क्योंकि इसके पीछे का कारण मन है। मन जब इन्द्रियों से संबंध स्थापित करेगा, इनके सान्निध्य में आयेगा तभी इन्द्रियां अपने विषयों को ग्रहण करेंगी। इन इन्द्रियों के द्वारा अपने अपने विषयों का ज्ञान तो हो रहा है पर नेत्र, कर्ण आदि के द्वारा सुख- दुख का अनुभव नही हो रहा जिस से यह सिद्ध हुआ कि इन दस इन्द्रियों के अतिरिक्त भी कोई और इन्द्रिय है जिसे 'सुखदुखद्युपलब्धिसाधनमिन्द्रियं मनः' अर्थात सुख दुख की उपलब्धि का साधन मन कहा है।*

*संसार में किसी भी पुरूष को ज्ञान का होना या ना होना मन का लक्षण है, आत्मा,.इन्द्रियां और इनके विषयों का संयोग मन से होने पर ही ज्ञान होगा।महाभूतों से उत्पन्न होने पर जैसे शेष इन्द्रियां बाह्य करण है यह अन्तः करण और स्थूल रूप का नही है, इसे शेष इन्द्रियों के द्वारा नही जाना जा सकता इसलिये यह अतीन्द्रिय है।*

*'प्रकुपिता हृदयमुपसृत्य मनोवहानि स्रोतांस्यावृत्य जनयन्त्युन्मादम् ' 

च नि 7/4 

उन्माद में मनोवाही स्रोतस का भी उल्लेख है।

'प्रकुपिता हृदयमुपसृत्य मनोवहानि स्रोतांस्यावृत्य जनयन्त्युन्मादम्'। 

मनोवाही स्रोतस की सब से गंभीर अवस्था आधुनिक समय में अवसाद है, 
च सू 16/14 में आचार्य चक्रपाणि 'अवसाद को मनोऽवसादः' कहा है।*

*स्रोतस को जानने के लिये हमें एकौ महागद 
'एको महागद इति अतत्त्वाभिनिवेशः' च सू 19/8
 के खवैगुण्य, दोष दूष्य सम्मूर्छना और मनोवाही स्रोतस की पूरी प्रक्रिया को जानना आवश्यक है, अतत्वाभिनिवेश चक्रपाणि के अनुसार 

'अतत्त्वाभिनिवेशो मानसो विकारः, स च सर्वसंसारिदुःखहेतुतया गद इत्युच्यते'

 वो मानस विकार है जो सब दुखों का कारण है जिसमें मन रज और तम से आच्छादित है और कुपित वातादि दोष ह्रदय का आश्रय ले कर मनोवाही स्रोतों को दूषित करते हैं।*

[1/5, 12:55 AM] Vaidyaraj Subhash Sharma: 

*उन्माद - स्रोतस -- मनोवाही और दूष्य 
'उन्मादं पुनर्मनोबुद्धिसञ्ज्ञा- ज्ञानस्मृतिभक्तिशीलचेष्टाचारविभ्रमं  विद्यात्' 

च नि 7/5 
अर्थात 8 दूष्य बताये है जो मन, बुद्धि, ज्ञान, स्मृति, भक्ति, शील आदि हैं।*

*अपस्मार -- स्रोतस - संज्ञावाही तथा दूष्य हैं स्मृति*

*मूर्छा -- स्रोतस - मनोवाही तथा दूष्य हैं इन्द्रियों के कर्म और चेतना*

*चिंतन, संकल्प, तर्क, सुख, दुख, यह पदार्थ भाव पदार्थ है या अभाव पदार्थ इसका अनुभव, स्रोतस पर हम और अधिक चिंतन करें या ना करें, खवैगुण्य पर अब कुछ चर्चा करें ना करें ? ये सब मन के कर्म या विषय हैं जो मनोवाही स्रोतस के द्वार संभव है।*

[1/5, 1:00 AM] Vaidyaraj Subhash Sharma: 

*खवैगुण्य के हम कुछ clinical उदाहरण ले कर चलेंगे तो हमारे कुछ साथी जिन्होने स्नातक अध्ययन काल में अच्छी तरह नही समझा वो इस प्रकार समझ सकते है, खवैगुण्य कहां हो रहा है ? स्रोतस जिनमें धातुओं का संवहन हो रहा है वो किस प्रकार से दूषित हो कर व्याधि को कहां तक ले कर जा रहे हैं ये स्रोतस के तारतम्य से ही संबंधित है।

'कुपितानां हि दोषाणां शरीरे परिधावताम्, यत्र संगः खवैगुण्याद्व्याधिस्तत्रोपजायते' 

सु सू 25/10, 

इस पर आचार्य डल्हण की व्याख्या है

 'खवैगुण्यात् स्रोतोवैगुण्यादित्यर्थः'।

 आचार्य चक्रपाणि 

'खवैगुण्य खवैगुण्यादिति स्रोतोवैगुण्यात्' 

च चि 15/37 

दोनों आचार्यों के मत से खवैगुण्य स्रोतस की विगुणता है क्योंकि खवैगुण्य स्रोतस में ही होगा जो उनके प्राकृत कर्मों में व्यवधान उत्पन्न करेगा और एक स्रोतस दूसरे को दूषित भी कर रहा है।*

*अतिसार - स्रोतस -- पुरीषवाही और दूष्य मल एवं जलीय धातु *

*आमवात - स्रोतस -- रसवाही और अस्थिवाही तथा दूष्य रस, स्नायु, अस्थि संधि, रक्त और मांस*

*प्रमेह - स्रोतस -- रसवाही, मेदोवाही और मूत्रवाही तथा दूष्य हैं मेद, रस, रक्त, ओज, जल, शुक्र, वसा, लसीका, मांस और मज्जा*

*ग्रहणी रोग - स्रोतस -- अन्नवाही एवं पुरीष वाही तथा दूष्य अन्न और रस*

*अर्श - स्रोतस -- पुरीषवाही तथा दूष्य त्वक्, रक्त, मांस और मेद*

*मेदोरोग - स्रोतस -- मेदोवाही और दूष्य मेद है।*

*मूत्रकृच्छ - स्रोतस -- मूत्रवाही और दूष्य हैं जल और मूत्र *

*कुष्ठ -- स्रोतस -- रस और रक्तवाही तथा दूष्य होते हैं रस(त्वक्), रक्त, अम्बु और मांस*

*पाण्डु - स्रोतस -- रस और रक्तवाही तथा दूष्य आचार्य चरक के मत से त्वक्, रक्त और मांस तथा आचार्य सुश्रुत के अनुसार रक्त*

[1/5, 1:03 AM] Vaidyaraj Subhash Sharma: 

*खवैगुण्य हमेशा स्रोतस मे होगा, वहीं पर स्थानसंश्रय होगा और दोष-दूष्य सम्मूर्छना भी वहीं पर होगी, यह प्रायः निज व्याधियों में मिलेगा, रोग के लक्षण जहां जहां व्याधि का प्रसरण होगा वहां मिलेंगे ।*
 *यहां शास्त्रीय जटिलताओं से बाहर आ कर हमने स्रोतस पर अपने चिकित्सक के दृष्टिकोण को सामने रखा है जिसे प्रायः हम clinic में प्रत्यक्ष अनुभव कर सकते हैं।*
*खवैगुण्य का उदाहरण*



[1/5, 1:41 AM] Vaidyaraj Subhash Sharma: 

*स्रोतस - रसवाही*
*दूष्य - रस (अन्न रस - त्वक्) और रक्त*
*यह गोधूम असात्म्य रोगी है।*

[1/5, 1:42 AM] Vaidyaraj Subhash Sharma: 

*संग और विमार्ग गमन*

 *दोष - जीर्ण रोगी है, त्रिदोषज लक्षण मिले और ग्रहणी रोग की चिकित्सा से एक महीने में इतना लाभ मिल चुका है।*

[1/5, 5:50 AM] Vd. Divyesh Desai Surat: 

🙏🏾🙏🏾सहर्ष सहृदय नमन गुरुश्रेष्ठ, आपकी पोस्ट का ही इंतजार था कि आप जो भी स्रोतस पे चर्चा चल रही है, इसमें आप अपना अनुभवजन्य ज्ञान को क्लीनिकली एविडेंस के साथ प्रस्तुत करेंगे ही, आपसे पदार्थविज्ञान, रोगविज्ञान, और आयुर्वेद के सिद्धांतों को कहाँ apply करना है, वो सीखने को मिलेगा ही,
संजय सर ने अपना हाइपोथिसिस रखा,
इससे आदरणीय गिरिराज सर, कटोच सर एवं बाकी के गुरुजनो ने शारीर रचना (Anatomical) और शारीर क्रिया (Physiological) के दृष्टिकोण से अपना मत प्रकट किया, रघु सर का पॉइंट ऑफ view भी as usual लाजवाब था, बाकी के आचार्यो ने अपना मत प्रकट किया,तो ये एक इम्पोर्टेन्ट तद्वित संभाषा बन गई
🙏🏾🙏🏾🙏🏾सभी आदरणीय गुरुजनो को नमस्कार एवं सुप्रभात💐💐💐
जय आयुर्वेद, जय धन्वन्तरि !

[1/5, 6:43 AM] Vaidya Sanjay P. Chhajed: 

Respected sir, pranam. What you are suggesting is evident in the cases of DM2, the deficiency occurs at cell level, where the cellular receptors being absent the use of sugar molecules continue to remain in the circulation. It disturbs the physiology & despite of the fact of hyperinsulanimea hyperglycemia is also evident.

[1/5, 7:24 AM] Vaidya Sanjay P. Chhajed: 

एकदम सही। क्लीनिकली यही दिखता है, और इस विषय वस्तु पर दुमत भी नहीं। जैसे गुरुदेव गुरुदिपसिंह सर ने कहा है सेल्युलर ख वैगुण्य आगे जाकर या तो उनके आसमंत में स्रोतस विगुणता  निर्माण करने में सक्षम होते हैं, या फिर यह ख वैगुण्य दोषों को विगुण कर स्रोतस दुष्टी करते हैं यह दो स्थितियां संभव है। इन दोनों परंपरा में स्रोतस यह एक्स्ट्रा सेल्युलर संरचना की तरफ इंगित करती है। अब प्रश्न यही बाकी रहता है की क्या स्रोतस यह रचनात्मक/ क्रियात्मक / उभयात्मक/ संभवनात्मक  है। उपरोक्त जितनी भी शारीरिक बिमारीया  वर्णित है वहां केपेलरी सिस्टम की इन्व्हालमेंट दृष्य है। मानस रोगों के बारे में पता नहीं, परंतु अपस्मार में मस्तिष्क की रक्तसंचरण को कारण माना जाता है।

 बड़ा जटील होते जा रहा है। 

जिन्हें हम चिकित्सा करते समय आसानी से समझा करते थे और यशस्वी चिकित्सा करते थे, वह वास्तविकता में इतनी दुर्धर है ? 

यह तो इस मोबाइल जैसी है, हम आसानी से उपयोग करते हैं पर इसके पीछे की टेक्नोलॉजी बहोत अधिक जटिल है।

[1/5, 8:21 AM] Dr. Bhadresh Naik Gujarat: 

According ayurved
Apsmar consider as a hrudgat vyaghi.

[1/5, 8:42 AM] Vaidyaraj Subhash Sharma: 

*आयुर्वेद दोष - दूष्य पर आधारित ज्ञान है और दोषों में रज और तम मानसिक दोष भी हैं, पंचभौतिक तल पर जाते हैं तो कार्य द्रव्यों की भौतिकता अनेक प्रमाणों से स्वतन्त्र सत्ता के साथ सिद्ध होती चली जाती है। आयुर्वेद में निरंतर अनुसंधान की प्रक्रिया जारी रहनी चाहिये जिस से जितना अधिक कार्य करते हैं हमें नवीन ज्ञान बहुत मिलता चला जाता है। स्रोतस के सूक्ष्म तल पर जाने से चर्चा के द्वारा अनुसंधान के अनेक मार्ग प्रशस्त हुये हैं। मैं तो स्वयं एक विद्यार्थी हूं जो काय सम्प्रदाय की चर्चा से अपने अपने विषयों के विशेषज्ञों से सिद्धान्त ले कर practice में उतारता हूं।*

*खवैगुण्य और स्रोतस पर चर्चा जितनी होगी उतने ही ज्ञान के मार्ग प्रशस्त होंगे ।*

[1/5, 8:51 AM] Vaidyaraj Subhash Sharma: 

*चिकित्सा के दृष्टिकोण से सभी व्याधियों के खवैगुण्य, पूर्वरूप, चिकित्सा सूत्र आदि उस प्रकार से ग्रन्थों मे उपलब्ध नही हैं जैसा सब चाहते हैं और पुरूषं पुरूषं वीक्ष्य अनुसार इनका निर्धारण स्वयं ही करना पड़ेगा क्योंकि हजारों अनुक्त व्याधियां अब आ चुकी हैं।*

*किसी भी रोग का आप खवैगुण्य history ले कर जान सकते हैं और सम्प्राप्ति का निर्माण कर सकते हैं बस प्रयास अवश्य करना चाहिये जिस से हम व्यर्थ की अनेक ढेरों औषधियां देने से भी बच जाते हैं और अपनी चिकित्सा पर विश्वास भी दृढ़ हो जाता है जैसे पिछले दिनों डॉ अतुल काले जी ने case present कर किया था, ऐसे उदाहरण अनुकरणीय होते हैं।*


[1/5, 9:26 AM] Dr. Bhadresh Naik Gujarat:


 Classic despriction of apsmar
Periodic
On and off
Status epilepsy
👆

[1/5, 9:27 AM] Dr. Mukesh D Jain, Bhilai: 

*The root cause of all chronic diseases is Khavaigunyas, as they attract the most ama (toxins) and excess doshas (functional energies). Can we refer  Kh’gunya to a weak or defective part of the Cellular apparatus, related to a genetic defect* ?.

[1/5, 9:35 AM] Dr. BK Mishra: 

👌🏼👍🏼 
But Genetic defect may be a separate point..

[1/5, 9:54 AM] Vaidya Sanjay P. Chhajed: 

Sounds good, it is now well understood fact that the manifestation of the disease may occur at later stage of life but it's predisposing factors are there either in the genetic form or in cosmic form. Otherwise genetic study would not have been usefull to understand the disease that might occur in later phase of life. This must have been imprinted, and must be evolved and retained in some sort that is the reason *The Nadi Pariksha/ Pulse reading* is able to carry this information to be detected by trained examiner. In similar way the Karmik interpretation of the Birth chart or Samudrik can tell you about past & future diseases including the accidents. 

My take is we need to look into this subject with multiple aspects and come to the conclusion- This is for highly evolved Vaidya.

For the Vaidya like me - नाना तंत्र विहीनानाम् भिषजाम् अल्पमेधसाम् ---- we should not bother about underlying phenomenon in depth but with presentation of the patient understand as an application & treat. That would suffice.

[1/5, 9:58 AM] Dr. Mukesh D Jain, Bhilai: ✅

[1/5, 10:18 AM] Prof.Vd.Arun Rathi: 

*रक्तवाही सिरायें, जो मुख्यतः यकृत् और प्लीहा मे रहती है*

*सुश्रुत संहिता शारीरस्थान अध्याय ७ / ६.*

🙏🏻🙏🏻🙏🏻


[1/5, 10:21 AM] Prof.Vd.Arun Rathi:

*सु. शा. ७ / ६.                              
*सिरावर्णश - विभक्ति नाम शारीरं।*


[1/5, 10:24 AM] Dr. Ashwani Kumar Sood: 

Absolutely right, there are many new presently recognised health issues not known previously like NON ALCHOLIC FATTY LIVER DISEASE but applying principles of NIDAN PANCHAK they can be cured .

[1/5, 10:24 AM] Dr. Pawan Madan: 

चरण स्पर्श व प्रणाम गुरु जी।
🙏

[1/5, 10:29 AM] Dr. Pawan Madan: 

स्रोतोमयं अयं पुरुष:

के अनुसार सम्पूर्ण शरीर स्रोतोमय है या ऐसे कह सकते हैं के स्रोतस सर्व शरीर व्याप्त हैं।

खवैगूण्य स्रोत्सों में ही होता है, फलस्वरूप खवैगूण्य शरीर में कहिन भी हो सकता है।

🙏🙏🙏🙏🙏🙏🙏

[1/5, 10:40 AM] Dr. Pawan Madan: 

Good mng Raghu sir.
Yesterday could not go through your specialized post due to time constrinct.

🙏

💐 Some arteries, veins and cappillaries can act like Srotas.

💐Sravanaat....Capillaries 🙏

🙏👏🏻👌🏻
Siraa and Dhamani.......cant be said directly as veins and arteries.....because the naming of Siraa and Dhamni has been done based on the peculier character as seen and felt by the Acharyas as explained in the texts.

Any Sira or Dhamni if fullfilling the functions of Transportation, Parnaam aapadyataa and Ayanaarthe, that particular Siraa or Dhamani or Vein or Artery or Cappillary will be called as Srotas.

That is the reason
..there are not only microscopic but also macroscopic srotas in the body as well.

Thanks sir..🙏

[1/5, 10:43 AM] Dr. Pawan Madan: Namaste...🙏

I think we can understand the srotomool in two ways..

..one....the mool may be the origin of that srotas (but this is not true for all srotas)

..second.....Clinically Srotomool are those parts which directly effect that particular srotas and and doing the treatment of that srotomool may coorect the srotodushti of that particular srotas.

🙏🙏🙏

[1/5, 10:45 AM] Prof.Vd.Arun Rathi:

 *नमस्ते पवनजी*

*मुझे तो प्रत्यक्ष मे दो हदय समझना है.*

[1/5, 10:46 AM] Dr. Pawan Madan:

 Namaste sir.

Khavaigunya can be cellular as well as extracellular, the only requirment is that particular part should fullfill the 3 conditions as explained in the texts and that which has been cleared in many posts by Resp Khandel sir ji and Rep Gurdeep sir ji.

🙏🙏

[1/5, 10:46 AM] Vd. Mohan Lal Jaiswal: 

स्रोतोमयं  अयं पुरुषः की व्यवहारिक अभिव्यक्ति तब होती है जब इन्द्रायण सुपक्व फल मज्जा को पादतल के नीचे कुछ समय मसलते रहने पर उसकी तिक्तरस की अनुभूति मुख में होने लगती है।

[1/5, 10:48 AM] Dr. Pawan Madan: 

दो हृदय,,,, अपस्मार के संदर्भ में ?

[1/5, 10:50 AM] Dr. Satish Sharma: 

Very much practical knowledge sir 🙏🏻🙏🏻🙏🏻

[1/5, 10:53 AM] Dr. Pawan Madan:

 Good mng sir.

Yes sir...it is like that only.

External hetus first produce Khavaigunya, after that the pathological process begings as explained by Resp Katoch sir one day back.
🙏

[1/5, 10:55 AM] Prof. Giriraj Sharma: 

चेतकी हरीतकी भी संभवत !


Respected Jayaswal Sir !


🙏

[1/5, 11:05 AM] Prof. Lakshmikant Dwivedi Sir: Shrotas dushti lakshan- 4 unme se charcha matra 2 par. par (Vyadhi utpatti "khavaigunya"-yatra *sangah*) Sira par?.charcha hai.
Shesh *Atipravratti*, and *granthi* par ? Vicharenge.
Kha & shrotas  
Lakshna- *Atipravratti* /adhishthan  -*granthi* vichar.😛🙏☺️

[1/5, 11:12 AM] Dr. BK Mishra: 

चक्रों की विगुणता का ज्ञान किसी न किसी स्रोतस की दुष्टि द्वारा ही होगा।

[1/5, 11:20 AM] Vd. Atul J. Kale: 

All body parts have minute subtle channels as well as micro spaces between. Every Sthool srotas obviously contains many sukshma srotas. As every sthoola srotasa, anga-pratyangas have arteries, arterioles, capillaries, venules, veins.... we can understand these as sthoola to sukshma and sukshma to sthoola srotasas. 
      Every space present in our body, may be sanvruta or avruta is srotasa.

[1/5, 11:23 AM] Dr. Pawan Madan: 👏🏻👏🏻👏🏻👌🏻👏🏻👌🏻👌🏻👏🏻🙏👏🏻🙏👏🏻


[1/5, 12:28 PM] Prof.Vd.Arun Rathi: 

*Introduction to Kayachikitsa*   
                         *Acharya C Dwarkanth*

[1/5, 12:34 PM] Dr. Pawan Madan: 

Very good ref..

The point to be noted is ...Srotas are dyanamic entities, they actively metabolite.

परीणाम आपद्यमाना

🙏

[1/5, 12:41 PM] Dr. D. C. Katoch sir: 

To my understanding  Kha Vaigunya is subtle initial most pathophysiology at the cellular level and Srotodusti (structural and/or functional  derangement) is the result of accomplished dosh-dusya sammoorchhna at the level of tissue/system.

[1/5, 12:44 PM] Prof.Vd.Arun Rathi:

*Chapter 21 of Introduction to Kayachikitsa is worth reading to understand "Kha Vagunya.*

[1/5, 12:45 PM] Vaidya Sanjay P. Chhajed: 

Wonderful boss...

[1/5, 12:45 PM] Prof.Vd.Arun Rathi: 🙏🏻🙏🏻🙏🏻

[1/5, 12:48 PM] Vaidya Sanjay P. Chhajed: 

सर, ह्रदय तो ६ है।
उरस्थित, मस्तिष्क स्थित, दो हाथ के तलहृदय, दो पांव के तलहृदय।

[1/5, 12:50 PM] Vaidya Sanjay P. Chhajed: 

Pawanji, Namaskar can you provide me some example of  intracellular deficiency causing any pathogenesis- samprapti?

[1/5, 12:50 PM] Prof.Vd.Arun Rathi:

 *नाडी गुरु संजयजी मै भी आप की तरह संस्कृत भाषा मे अल्पमति हूँ।*
*संस्कृत के श्लोक कण्ठस्थ नही है.*
😃😃😃

[1/5, 12:52 PM] Prof.Vd.Arun Rathi: 

*तो इन्हें (6 ह्रदय) भी प्रत्यक्ष में समझना होगा*
🙏🏻🙏🏻🙏🏻

[1/5, 12:53 PM] Vaidya Sanjay P. Chhajed:

 क्या सही मे ऐसा होता है? हमने मात्र हथेली पर लगे महानारायण तैल को कूर्पर पर मिलते देखा है और अभ्यंग के पश्चात गैस्ट्रिक द्रव में स्नेह का पाना सुना है।

[1/5, 12:56 PM] Dr. D. C. Katoch sir: 

In Hridyon ko naam se samajhane mein Confusion hoga,  samuchit hoga unke karmon se samajhana aur un karmon mein vibhed karana.

[1/5, 12:56 PM] Dr. Pawan Madan: 

If we see the details of the homeostasis....it includes the balance between intracellular and extracellular spaces and it is very important in many diseases. Gene rally extracellular space is a potential space but it take parts in normal physiology.

Lets say if the action potential in a nerve diminishes due to loss of some ions within the cell.

Or
Loss of acetylcholine in the nerve endings resulting in pathologies like GB syndrome.
Many such examples be given I think...🙏

[1/5, 12:57 PM] Dr. Pawan Madan: 

The effects of Basti karma can better be explained on this principle ....

[1/5, 12:57 PM] Vaidya Sanjay P. Chhajed: 

हथेली के प्रत्यक्ष कर सकते हैं। एक बहुत ही छोटे प्रयोग से ।अगर हम रेडियल आर्टरी पर अपनी तीन उंगलियां रखें और पिछले दो उंगलियों के द्वारा ह्रदय से आने वाले प्रवाह को रोके, तब तलहृदय से आने वाला प्रवाह आपको तर्जनी में दिखाई पड़ता है।

[1/5, 12:58 PM] Vaidya Sanjay P. Chhajed: 

With hypothesis, has anyone did the study, if yes please share.

[1/5, 12:58 PM] Vd. Atul J. Kale: 

Every Srotasa is having their own contents. Accordingly there names have decided.

 स्रवणात् स्रोतांसी gives the appropriate idea about कार्यकारणभाव also. 

      There are certain clues which gives different ideas about srotasas I.e......

 A) Which oozes, 
B) Spaces between
C) Ekamukha, Dwimukha etc. 
    
*-Parinaman, Vahan and utsarjan*
these functions take place in Srotasas which need *Agni Sahacharya.* Different Agnis at different Srotasas have different seats of dravyas and act according to their Prakruti. E.g. Annavaha Sotasa, Raktavaha Srotasa etc. 

ख१) 

Micro channels 'ख' present in macro srotasas 'ख'... *transports to ooze* Srotodravya in to macro cavity of macro srotasas. E.g. bile juice, HCL, pancreatic juice, Mutra, Bodhak kapha, different types of hormones, aqueous humor and so on. 

 ख२)
   
There are certain cells which secrets their respective secretions. For proper secretions and spaces ...micro pores are present in particular cell indicates micro 'ख' , 

ख३) 

after secretion.... for transportation again needs micro pipe like 'ख' ,  

ख४)

For store these transported secretions again it needs macro 'ख'. 

ख५) 

For secretions there is action of Vayu which needs Chala Guna along with 'ख' to move.

ख६)

For प्राग् देशविभाग and अन्यदेश संयोग needs 'ख'. I.e. for transportation.  

ख७) 

Lack of 'ख' does not allow transportation hence कर्ता Vayu will be obstructed. 
      
ख८)

यथासन्नं निर्हरेत which indicates purification of 'ख'. 
उदा. वमनादी.

Why 'ख' वैगुण्य
What's 'ख' वैगुण्य

In Dhatus, In Srotowalls...
Proper accumulation of dhatu particles in normal form is संहनन. 

There are certain combinations of different Mahabhutas with different permutation and combinations. 

-If Prithvi is more, 'ख' will be more more less. 
-If Jala is more 'ख' will be more less. 
-If Agni is more 'ख' will be more and by Vayu and Akash it will be more and more.

Different Dhatus have different संहनन which again indicates specific 'ख' between two adjacent dhatu particles (कणाद)  in their normal form.


खवैगुण्य -

1) धातुशैथिल्य 
means increased spaces between two adjacent particles than their normal forms. So these increased  spaces allows doshas to penetrate inside. 

2) धातु संघात
Which indicates decreased spaces between two adjacent particles than their normal form.

3) All Srotodushtis can be calculated under the umbrella of 'ख' वैगुण्य.

So 'ख' वैगुण्य means

Changing the proportion of 'ख' between two adjacent DHATU particles. 
And
Depletion of Dhatu particles.

[1/5, 12:59 PM] Dr. Pawan Madan: 

Yes sir.
I have one book of Dr Mahadevan, I can shate the exact name on friday as I am out to Jammu today for opd.

[1/5, 1:03 PM] Vd. Atul J. Kale: 

साष्टांग नमन गुरूवर्य 🙏🏻🙏🏻🙏🏻🙏🏻
🌹🌹🌹

[1/5, 1:04 PM] Dr. Pawan Madan: 

Very nice Atul Sir.

Elaboration of 

संवहन, 
परीणाम आपाद्यमां
मार्ग अयनार्थे

..in detail. 

Now there should be clarity that any structure having these three functions will be called srotas.
And Khavaigunya occurs in srotas prior to the sroto dushti as has been repeatedly said in the discussion by various stalwarts.

Thanks for the detailed version sir.

🙏👌🏻👏🏻

[1/5, 1:52 PM] Prof.Vd.Arun Rathi: 

*प्रणाम सरजी*

🙏🏻🙏🏻🙏🏻

[1/5, 1:56 PM] Prof.Vd.Arun Rathi:

 *ख वैगुण्य, यह स्त्रोतस्  दुष्टी के केवल पूर्व ही नही, अपितु स्त्रोतस् दुष्टी की सभी अवस्था मे रहेगा*

[1/5, 1:58 PM] Vd. Atul J. Kale: 

*'ख' तो आखिर 'ख' है*
*यत्र तत्र 'ख' ही है ।*
*यही सूक्ष्म है*
*यही विभू भी है*
*सर्वभूती यही है।*

*'ख' तो आखिर 'ख' है*
*यत्र तत्र 'ख' ही है।*
*मुझमें तुझमें 'ख' है*
*जीवनका आधार 'ख' ही है।*

*'ख' तो आखिर 'ख' है*
*यत्र तत्र 'ख' ही है।*
*शारीरका आधार 'ख' है*
*रचनाका कारण भी 'ख' ही है*
*दर्शन 'ख' के बीना अधुरे है।*

*'ख' तो आखिर 'ख' है*
*यत्र तत्र 'ख' ही है।*
*पिण्डमें भी 'ख' है*
*ब्रह्माण्डमें तो है ही है।*

*'ख' तो आखिर ख है*
*यत्र तत्र 'ख' ही है।*
*सर्वस्थ व्यापक 'ख' ही है*
*'ख' तो परम ईश्वर है।*

*'ख' तो आखिर ख है*
*यत्र तत्र 'ख' ही है।*
*मठाकाशमें विद्यमान है*
*पुरे विश्वमेंभी 'ख' ही है।*

Vaidya Atul J Kale

[1/5, 2:02 PM] Dr. Pawan Madan: 

Waaah waaah 
waaah waaah

[1/5, 2:33 PM] वैद्य सुखबीर सोनी: 

सही से तलं करने पर भी मुख में स्वाद आता है ।

[1/5, 2:59 PM] Dr. D. C. Katoch sir: 

🤗 Dil mein Kha, Dimag mein Kha; Phuphphus mein bhi Kha hai, Vrikka mein bhi Kha hai ------  phir to Kha nahin, Kha jisme hai vo mahatvapoorna hai. 🤓

[1/5, 3:02 PM] Dr. BK Mishra: 

*खं ब्रह्म* इति श्रुतिः
ख सबमें नहीं सब कुछ *ख में* अवस्थित है...🙏🏼🙂

[1/5, 3:20 PM] Dr. Vinod Sharma Ghaziabad: 

बहुत खूब अतुल जी ।🙏🏾🙏🏾👏👏

[1/5, 4:10 PM] Vaidya Sanjay P. Chhajed: 

You will have to be very specific when we are discussing intracellular deficiency. First- it is non measurable
Second - we need to identify them

Let us take the example of neurotransmitter deficiency - it has to be at gross level to manifest the disease and definitely not at intracellular level. Having hypothesis is fine but getting it reported seems impossible.

[1/5, 4:13 PM] Vaidya Sanjay P. Chhajed: 

ख से तुम, ख से हम, ख से द्रव्य,गुण और कर्म। ख से उत्पन्न पांचों महाभूत, ख से है सब कुछ, यही आयुर्वेद का मर्म।।

[1/5, 4:17 PM] Vaidya Sanjay P. Chhajed: 

आत्मन: आकाश संभूत: -----

[1/5, 4:22 PM] Vaidyaraj Subhash Sharma: 

*नमस्कार एवं शुभ सन्ध्या 🌹🙏*

[1/5, 4:22 PM] Vaidya Sanjay P. Chhajed: 

🌹🌹🌷🙏 नमस्कार सर !

[1/5, 4:26 PM] Dr. Shekhar Singh Rathore Jabalpur:

 नहीं सर। पर सुना है कई लोगों से, इस प्रकार के अनुभव, जब हाथ पर मलने से तिक्त रस जिव्हा पर अनुभव हुआ।।

[1/5, 4:32 PM] Vaidyaraj Subhash Sharma: 

*प्रतिदिन ऐसे अनेक रोगी आ जाते है जो modern diagnosis में होते हैं पर दोष दूष्य और स्रोतस पर सम्प्राप्ति सब कार्य सरल कर देती है जैसे आज hunters syndrome का एक रोगी आया तो आदिबल प्रवृत्त व्याधि का स्मरण हो आया कि यह व्याधि ग्रन्थों में इस प्रकार नही है पर हम इस रोगी को बहुत लाभ दे सकते है।*

[1/5, 4:34 PM] Vaidyaraj Subhash Sharma: 

*शास्त्र वाक्य 👌👌 रोगी को स्वस्थ करना है और हम अपनी सरलता के अनुसार अनेक मार्ग बना सकते है।*


[1/5, 4:36 PM] Dr. Govind Ojha:

 ✅✅100%सत्य, मूल एवं गूढ़ रहस्य👏🌹

[1/5, 4:36 PM] Vaidyaraj Subhash Sharma: 

*सादर सप्रेम नमन आचार्य बृज किशोर जी 🌹🙏*

[1/5, 4:37 PM] Dr. D. C. Katoch sir: 

Bilkul sahi, Agar Ayurvedic treatment karani hai to Ayurvedic drishtikon se hi nidan panchak dwara hetu- vikriti- lakshan sammuchchaye ka vishleshan karana hoga.

[1/5, 4:39 PM] Vaidyaraj Subhash Sharma: 

*जी आचार्य लक्ष्मी कान्त जी, अति प्रवृत्ति और ग्रन्थि पर भी विचार और चर्चा करनी ही  चाहिये ।*


*शुभ सन्ध्या युक्त सादर नमन सर, ग्रुप में चर्चा चलती है तो लगता है कि आयुर्वेद को समझने के लिये 5-6 जीवन भी कम हैं। स्रोतस ही पूरा महाज्ञान है, जितना इस पर कार्य करें और बढ़ जाते हैं।*

[1/5, 5:15 PM] Dr. BK Mishra: 

🙏🏼आपके अनुभव शास्त्रीय सिद्धान्तों का स्मरण करा देते है..सादर नमन💐

[1/5, 5:23 PM] Dr. BK Mishra: 

🙏🏼यही आयुर्वेद की महत्ता है।
आचार्यों ने प्रकृतिस्थापन के लक्ष्यप्राप्ति के लिये अनेक मार्गों के अनुसरण (*युक्ति प्रयोग*) की खुली छूट प्रदान कर रखी है...
 *तिष्ठत्युपरि युक्तिग्यो*...💐🙏🏼

[1/5, 5:29 PM] Dr. Shekhar Singh Rathore Jabalpur: 

आभार सर। आपके वक्तव्य की ही प्रतीक्षा थी।। क्लिनिकल उपयोगिता ही किसी विषय को सुग्राह्य बनाती है।।
🙏🏻🙏🏻🙏🏻

[1/5, 5:45 PM] Dr. Shashi: 

When we go through shoth samprapti, बाह्य सिरायें   word is used, which vessel can be referred here ? May it be veins, venules and capillary net work. Shoth I think is the basic pathology present in almost every disease, because shoth rog described in Charak is not inflammation only. That is involvement of small vessels as srotas in almost all diseases, from jvar to tumours.
Thanks

[1/5, 5:52 PM] Dr. Vinod Sharma Ghaziabad: 

अभ्यंग की कार्य प्रणाली इस प्रश्न का उचित उत्तर होगा ।

[1/5, 6:31 PM] +91 89014 54426: 

Ye to bhi hua jo awyakta hai,jaha se srishti ki utpatti hui hai.

[1/5, 6:36 PM] Dr. Ramesh Kumar Pandey: 

अर्क पत्र जूतों में रखने पर मुंह का स्वाद तिक्त होने को किस प्रकार से समझ सकते हैं ??

[1/5, 6:42 PM] Dr. Suneet Aurora, Punjab: 

I new a naturopath who used to tell patients to stand and stomp on a tub filled with aloe vera/ katu patol pulp.
Patient had to do it till he/she felt bitterness in mouth. 
He used this to treat liver disorders, diabetes and skin disorders.
Most patients reported bitterness in mouth in few minutes, and some didn't.
I understood it as treatment with the _prabhaav_ of the medicine _dravya_.

[1/5, 6:44 PM] Dr. Mamata Bhagwat Ji: 

स्मृतेरपगमनं ....
अपगतः स्मारं  
Temporary loss of smriti as result of vyadhi Vega is cardinal feature in Apasmara. 

गते वेगे सुप्तवत् प्रतिबुध्यते। 
The patient wakes up as a normal being after the Vega is relieved. 
This is another cardinal feature of Apasmara as well as Apatantraka. 

Respected Sir, 
With a humbled hearr, i would like to request some elaboration about,

1. Involvement of Hridaya, its pattern as variable in Apasmara and Apatantraka both. 

2. Hridaya is avrita avastha by doshas in both diseases. How to save hridaya from avarana the n further course. ?

3. Both are neurological entities according to modern science, 
To see .. memory loss is present in both for the very moment of the attack. 
One is Manasa roga, other is shiro marmagata roga (A. Charaka ) and Vatavyadhi ( A Sushruta) 
As a clinical manifestation why this difference exist ? 

I understand, that my queries are demanding for elaborate answers. Please take these at your leisure time or audio record them and share. 
Would be much obliged Sir. 

Thank you for always keeping high spirits to energise the group and enthusiastic to share several ideas with the members🙏🏻🙏🏻🙏🏻

[1/5, 6:50 PM] Dr. Mamata Bhagwat Ji: 

Pranam sir🙏🏻

ऐसा नही है। ख वैगुण्य निदान और दोष दोनों से हो सकता है। 
Dosha does it as a result of Dosha vikopa. But that's before sthana samshraya. The viguna Dosha can alter the being of normal metabolic activity ..the chaya apachaya in the sookshma srotas. This becomes a cause for sroto vaigunya. 
For example, rooksha guna vriddhi leads to sneha guna hrasa in the srotas. That's rikta srotas. Riktata is a kha vaigunya which is primary predisposition for manifestation of Vata vyadhis.

[1/5, 6:51 PM] Prof. Giriraj Sharma: 

भाव  Cell including Gene
⬇️
अवयव ( चिकित्सा) Tissued
⬇️
अंग-प्रत्यंग
Organs
⬇️
स्रोतस  (चिकित्सा मूलतया) 
Systems
⬇️
शरीर
Body

आयुर्वेद चिकित्सा शास्त्र में प्रायः स्रोतस विशेष की समझ पर चिकित्सा की जाती है, अंग प्रत्यंग की चिकित्सा प्रायः नही करते,
परन्तु आचार्य चरक अवयव विशेष के स्तर की चिकित्सा के लिये भी निर्देश करते है ।

*शरीरसँख्या यो वेद सर्वावयवशो भिषग ।*
*तद ज्ञान निमित्तेन स मोहेन न युज्यते ।। च शा 7/18*
स्रोतस स्थूल सूक्ष्म या संवृत असंवृत स्वरूप को समझे तो सूक्ष्म स्रोतस अवयव विशेष की चिकित्सा होनी चाहिए क्योंकि अवयव स्तर का ज्ञान होने पर किसी भी प्रकार का संशय नही रहता ।

अवयव पर एक चिंतन 

*अवयव - A Histological Aspect of AVAYAVA in reference of Tissues.*

*शरीरसँख्या यो वेद सर्वावयवशो भिषग ।*
*तद ज्ञान निमित्तेन स मोहेन न युज्यते ।। च शा 7/18*
*The Knowledge of tissues (अवयव ) lavel  in the body can only understand perfectly without any doubts.*

*शरीरावयवास्तु परमाणुभेदेन अपरिसंख्येया भवन्ति तर्क्यअवयवेषु ।*
*The  (अवयव )tissues are minutes, uncountable in numbers.*

*अतिबहुत्वाद अति सौक्ष्म्याद अति इन्द्रियात्वाच्च ।*
*तेषां अपि संयोगविभागे परमाणूनां कारणम वायु ।*
*Tissue - A collection of the particular cells (भाव) which fills the interstices between more specialized elements and serves to hold them together and support them.*

 *These tissues  (अवयव) can't visible by naked eyes. It's identified by microscope like instruments.*

*अपरिसंख्येयावयवो लोकः,   अपरिसंख्येया लोकावयवविशेषा,  पुरुषावयव अपि अपरिसंख्ययाः* 
*These (अवयव )elements are available in huge  numbers. These special elements present in human being and this universe. These(अवयव ) tissues are imperceivable in human being. Human being can perceived it's by तप चक्षु  or great practice and mechanically*

*Formations of Avayav(अवयव )*
*These avayavas(अवयव ) are forms by Matrijaadi Shadbhava (Cells with Genes)*
*मातृजादि षडंभावा,,,, ,,, अवयव प्रभृत,*
*As per Aacharyal Charak Matrajaadi Shadbhav are main causes of Avayav of Heart liver spleen bone ligaments etc.*

 *These Avayav forms by structurally by maternal and paternal factors मातृज एवं पितृज भाव*
*it's size shape colors and grade of functions affected by Progenitor and psychological factors, आत्मज एवं सत्वज भाव*
*it's growth and development depends on Nutritional & Environmental Factors रसज एवं सात्म्यज भाव*  

*न शक्यश्चक्षुषाम द्रष्टु देहे सूक्ष्मतमा विभू ।*
*दृश्यतेज्ञानचक्षुचक्षुर्भिस्तपचक्षुर्भिरेव चः ।।
  सु शा 5/52*
*These (अवयव ) ailments or tissues can not perceived by naked eyes it can perceived by mechanically/  microscopes etc like instruments and तप चक्षु /with great practicing*

*Further developments of Avayava in embryonic stage.*

*प्रथमे मासि,,,, सदसदभूतांगवयवा ।* 
*तृतीये मासि,,, सर्वांगावयवाश्च ।*  *च शा 4/6*
*In embryonic stage these tissues (अवयव )appears by a series of cell division of zygote and got differentiate, migrated, multiplied, transformed  and compacted*

*अंगप्रत्यंग विभाग सूक्ष्मो भवति ।  सु शा 3/9*
*असंजात अंगप्रत्यंग प्रविभागः,,,, सर्वावयवानुसारिणाम ,,,
 सु शा 3/31*
*These tissues(अवयव) develop in primordial stage, further growing and developing by the Nutrition of mother causes the primordial stage development of all organs अंग-प्रत्यंग and systemsस्रोतस*

*सर्वे अंगप्रत्येगेषु सत्स्वपि सौक्ष्म्यादनुपलब्धि, तानेव्य काल प्रकर्षात प्रव्यक्तानि भवन्ति ।
 सु शा 3/32*
*These tissues(अवयव) modified into different shape, size, location, function and forms different different organs and organs forms systems by developing phases and natural rules of governing the composition and sequential growth of all structures*

 *These variation, differentiation and different forms of tissues (अवयव) depends on the nature of it's basic components षडभाव.*

*1.Intrinsic components*

*2.Staining characters*

*3.Physical property*

*4.Chemical nature*

*5.Intracellular ground substance*

*6.Patterns of molecules arrangements*

*7.Varieties of nutrition and their distribution*

*यस्य यस्य हि अंगावयवस्य बीजे बीज उपत्तप्तो भवति तत्र विकृतिर्ज़ायते,,,
 च शा 3/17*
*Whichever tissues (अवयव) and cells भाव afflicted by intrinsic and extrinsic causes, the same parts of the devlop organs and systems are affected.*

*शरीरसँख्या यो वेद सर्वावयवशो भिषग ।*
*तद ज्ञान निमित्तेन स मोहेन न युज्यते ।। 
च शा 7/18*

*So we must think about अवयव चिकित्सा with स्रोतस based चिकित्सा उपक्रम ।*

🙏🏼🙏🏼🙏🏼🌹🙏🏼🙏🏼🙏🏼

[1/5, 6:58 PM] Dr. Surendra A. Soni: 

नमो नमः आचार्यश्रेष्ठ !

अत्यंत सारगर्भित और प्रायोगिक विवरण ।

🙏🏻👍🏻🌹
बहुत पीछे छूट गया बहुत सारे पोस्ट पढ़ने हैं ।

🙏🏻🌹☺️

[1/5, 7:04 PM] Dr. Surendra A. Soni:


[1/5, 7:04 PM] Dr. Suneet Aurora, Punjab:

 Sir, I saw it around 15y back. I couldn't get to ask objectively for the results then. But a lot of patients followed it and reported relief. After they felt bitter taste in mouth, they were given some juice/ kwath.

Prof. M. L.  Jayaswal  Sir !

[1/5, 7:15 PM] Dr. Surendra A. Soni: 

रसो निपाते द्रव्याणाम् ।
विपाको कर्मनिष्ठया ।

🙏🏻

[1/5, 7:16 PM] Dr. D. C. Katoch sir: 

Samuchit Nirdesh !👍🏼

[1/5, 7:19 PM] Dr. Surendra A. Soni: 

Samprapti vighatan is impossible.
These are so called designer treatment modules, get popularity so early but disappear quickly too...
Anything can happen in India.

[1/5, 7:24 PM] Dr. Surendra A. Soni:

 यह एकान्तिक पक्ष मात्र है । विस्तृत ज्ञानार्जन हेतु ज्वर सम्प्राप्ति अवलोकनीय है ।

🙏🏻🌹

[1/5, 7:33 PM] Dr. D. C. Katoch sir: 

Appreciate your well thought, deeply scientific and focused conclusion about Chikitsa approach based on principles of Shaarir - 
*So we must think about अवयव चिकित्सा with स्रोतस based चिकित्सा उपक्रम ।*

Bravo Shaarirvid!!⚘⚘

[1/5, 7:33 PM] Dr. Suneet Aurora, Punjab: 

Yes sir, those were the times of Ramdev baba's yoga-ayurved. Everyone was lining up for miracle cures. 
Then we saw an era of _ghar ghar mei naturopaths and herbal experts_, thanks to qualified/ non qualified people competing on social media with their miracle cure videos n posts.

And here, on this great platform, as we start about the _srotas_ and reach the 'ख', we feel like a novice sophomore again...

😌🙏🏼


[1/5, 7:39 PM] Dr. BK Mishra: 

🙏🏼💐  मैंने प्रो ध्यानी जी के मतानुसार लिखा है...
ज्वर सम्प्राप्ति का पुनरावलोकन करता हूं..
त्रुटि इंगित के लिये धन्यवाद..💐

[1/5, 7:45 PM] Dr. Mamata Bhagwat Ji: 

Among 10 patients you see, 9 would bring a bundle of investigations. 
We cannot just go by the reports and write our prescription. That wouldn't be Ayurvedic approach. 
We need to translate these reports, conditions, and diagnosis into Ayurvedic concepts and try to analyse Dosha dooshyadi in the same. Then we may approach with Ayurvedic Nidana and think about Ayurvedic protocol of management. 

Otherwise it will be just a herbal medicine for modern diagnosis which may gravely mislead us. 

This I could learn to extent from my Gurus and Aaptas like you in the field of Ayurveda🙏🏻🙏🏻

[1/5, 7:50 PM] Dr. Pawan Madan:

 👏🏻👏🏻👏🏻🙏🙏🙏

Srotas based chikitsa is one of the Vidhaa of deciding the chikitsa sutra.
In many cases....we need to employ the Srotas based chikitsa, which works on the Avyavas in that srotas.

How the Avyava based chikitsa be done more efficiently?

🙏🙏

[1/5, 7:50 PM] Dr. Mamata Bhagwat Ji:

 Sir, 
While highlighting on the action of Hetu, respected Dhyani sir mentions that hetu can act as Srotovaigunya karaka. 
But he never denies the role Dosha in causing khavaigunya as well. 
Dosha can do 
1. Other Dosha vaigunya
2. Dhatu pradooshan
3. Khavaigunya
4. Dosha dooshya            sammoorchana
5. Vyadhi
6. Udarka
7. Upadrava
8 Asadhya
9 Marana. 

All possible by doshas. 

Capable of Building while prakruta,

 demolition when vikruta.... both possible...

[1/5, 7:50 PM] Dr. Surendra A. Soni: 

I thank you for initiation of a great discussion.
It gave great opportunity to learn for everyone.

Namo namah !!🙏🏻🌹

[1/5, 7:52 PM] Dr. Pawan Madan: 

Namaste..🙏

In a way, Dosha act as Hetus many times..
😊

[1/5, 7:53 PM] Dr. Mamata Bhagwat Ji: 

True

[1/5, 7:53 PM] Dr. Surendra A. Soni: 

आ. ध्यानी जी और आपको नमन करते हुए प्रस्तुत है....


संसृष्टाः सन्निपतिताः पृथग्वा कुपिता मलाः ॥१२९॥
रसाख्यं धातुमन्वेत्य पक्तिं१ स्थानान्निरस्य च ।
स्वेन तेनोष्मण चैव कृत्वा देहोष्मणो बलम् ॥१३०॥
स्रोतांसि रुद्ध्वा सम्प्राप्ताः केवलं देहमुल्बणाः ।
सन्तापमधिकं देहे जनयन्ति नरस्तदा ॥१३१॥
भवत्यत्युष्णसर्वाङ्गो ज्वरितस्तेन चोच्यते ।

🙏🏻🌹☺️

[1/5, 7:54 PM] Dr. Mamata Bhagwat Ji: 

🙏🏻👌🏻💐

[1/5, 7:58 PM] Dr. D. C. Katoch sir: 

Dosha on becoming Mala (getting vitiated) only act as Hetus for vikar uttapati.


[1/5, 8:04 PM] Dr. Surendra A. Soni:

 ब्रह्माण्ड में भी परंतु इस ख में कभी आयुर्वेद विलुप्त नहीं हुआ और न आगे भी कभी होगा ।

🙏🏻🤓

[1/5, 8:08 PM] Vd. Atul J. Kale: 

Morbid doshas may cause खवैगुण्य. Or genetically some body may have weakness of particular srotas. Or continuous consuming of अपथ्यकर आहार विहार may cause depletion in dhatus.

[1/5, 8:42 PM] Prof. Giriraj Sharma:

 आदरणीय शुभ संध्या
गर्भ व्याकरण शारीर अध्याय में सुश्रुत शारीर में अंग-प्रत्यंग की निर्माण में धातु अवयव का स्पष्ट विवेचन है ।

गर्भस्य यकृत्प्लीहानौ शोणितजौ, शोणितफेनप्रभवः फुप्फुसः [२८] , शोणितकिट्टप्रभव उण्डुकः
रक्तमेदःप्रसादाद्वृक्कौ;
 मांसासृक्कफमेदःप्रसादाद्वृषणौ; शोणितकफप्रसादजं हृदयं

*शोणित, मांस, मेद (धातु अवयव) मातृज भाव जन्य है, ये सब भावप्रवृत्त अवयव है जिनसे अंग प्रत्यंग बन रहे है*

*एक उदाहरण...*
 *रक्त मेद धातु अवयव मृदु भाव है  जो मातृज भाव से भावप्रवृत्त होकर  वृक्क प्रत्यंग का निर्माण करते है वृक्क प्रत्यंग मेदवह स्रोतस का मूल है, मेदवह स्रोतस की चिकित्सा करते है हम तो वहां वृक्क की चिकित्सा भी हो जाती है यह स्रोतस चिकित्सा में ही समाविष्ट हो जाता है ।*

*रक्तमेदःप्रसादाद्वृक्कौ*

*अवयव स्तर पर मेद के साथ रक्त की चिकित्सा को भी ध्यान रखना अवयव चिकित्सा है*

 *क्योकि वृक्क मेदोवह स्रोतस का मूल है परन्तु मेद के साथ रक्तव अवयव भी उसके निर्माण में अहम भूमिका निभा रहा है ।*

*मेद के साथ रक्त का चिन्तन भी अवयव स्तर की चिकित्सा में समाविष्ट हो जाएगा,,,,*
एक प्रयास मात्र

[1/5, 8:58 PM] Dr Satish:

 रस औषधि का कार्मुकत्व भी स्रोतस के आधीन है। पारद और सूक्ष्म भस्म सेल्यूलर लेवल पे माइक्रो चैनल्स में cellular permeability change कर देता है। इसलिए रस औषधि अल्प मात्र उपयोगित्वद ... स्मॉल  dose l, quick action, fast action with absorption, 
और जब पारद अपथ्य ककाराष्ट आदि गण के द्रव्य दिए जाते है तो वो रस औषधि अवशोषित नही होती और मूत्र मार्ग से निसरण हो जाती है। 
रस चिकित्सा पूरी स्त्रोतस के आधारित है। मेरा अंगत मत है कृपया और संभासा से में भी लाभावनीत हुवा और नम्र निवेदन गुरजनो से और रस औषधि परिपेक्ष में स्त्रोतस विज्ञान के अंतर्गत ज्ञान दे 🙏🏻

[1/5, 9:03 PM] Vaidya Sanjay P. Chhajed: 

Sir you have made a great statement. Can you please elaborate it? 
You have said that bhasma will alter the cellular permeability. Which bhasma or every bhasma? Is there any variations in different bhasma? How does it achieves it?

Does it have some action on the srotas? If yes what type & how?

[1/5, 9:03 PM] Dr Satish: 

मूलत पारद भस्म 🙏🏻

[1/5, 9:04 PM] Dr Satish: 

Combination me रससिंदुर+ any dhatu bhasma 🙏🏻

[1/5, 9:04 PM] Vaidya Ashok Rathod Oman: 

अधिक आसान शब्दो में त्रिदोष के गुणो वाले (समान गुण वाले) द्रव्य का अतिसेवन और धातुओ के विपरीत गुण के आहार-विहार का अधिक सेवन ही स्रोतस में *ख* वैगुण्य निर्मित होने का कारण हैं।*

[1/5, 9:07 PM] Dr Satish: 

इसीलिए शास्त्र में पारद से निर्मित धातु भस्म उपयोग करने का विधान है। नहीं के अरिलोह और गंधकादि से   
दुर्गुण प्रदम है। 
लोहना मारना में सर्वेषाम रसभस्मानम🙏🏻

[1/5, 9:10 PM] Vd. Atul J. Kale: 

इसीलिए संभवत: वृक्क विकारों में रक्त अंतर्भाव के लक्षण मिलते है।

[1/5, 9:32 PM] Dr. Surendra A. Soni:

 आचार्य जी ।

आपका कथन विचारणीय है ।
रस, गुण, वीर्य, विपाक, विषोपविष और सभी आधुनिक द्रव्यों की कार्मुकता अग्नि के माध्यम से सर्व शरीरगामित्व स्रोतस् के माध्यम से ही होता है । अग्नि भी महास्रोतस में स्रोतस् के द्वारा ही पहुंचती है । इसी लिए स्रोतोमय पुरूष कहा गया है ।

Dr. Satish !

[1/5, 9:37 PM] Dr Satish: 

प्रणाम गुरुजी !
मेरी जिज्ञासा यह है कि पारद अपथ्य
(ककराष्टक) से स्त्रोतस के द्वार बंद हो जाते है (माइक्रो चैनल) तो फिर रस, गुण, वीर्य, विपाक, विष उपविष और काष्ठ औषधि संबधी ऐसा कोई प्रावधान ?? 🙏🏻

[1/5, 9:37 PM] Dr. Surendra A. Soni:

 नमो नमः आचार्य संजय जी !

यथोचित प्रश्न है और आपश्री और आ. गिरिराज जी से मार्गदर्शन की प्रार्थना है ।

🙏🏻🌹😌

[1/5, 9:40 PM] Dr. Surendra A. Soni: 

पूरा प्रावधान है तभी तो अग्नि, आम, शोधन, अभिष्यंद और स्रोतोदुष्टि आदि विविध भावों की विवेचना की गई है । मूल में स्रोतस ही है ।

[1/5, 9:46 PM] Dr Satish: 

लेकिन गुरुजी blood brain barrier को पारद भस्म क्रॉस कर जाती है। स्रोतस भाव इस संदर्भ में प्रकाश डाले क्योंकि बाकी काष्ट औषधि उस सूक्ष्म स्रोतस क्रॉस कर जायेगी ?🙏🏻

[1/5, 9:48 PM] Dr. Surendra A. Soni:

 प्रत्यक्ष तो आप से जानना अपेक्षित है पर मेरी क्षुद्र बुद्धि से निम्नलिखित सिद्धांत इस द्वित्व में प्राणापान को ध्यान में रखकर स्थापित किया गया है ।

*प्राणापानसमानैस्तु सर्वतः पवनैस्त्रिभिः ।* 
*ध्मायते पाल्यते चापि स्वां स्वां गतिमवस्थितैः ।।२८।।*
सु सू 35

🙏🏻🌹

[1/5, 9:48 PM] Vaidya Sanjay P. Chhajed: 

Sir you are missing the main point. You made a point that the bhasma - after correction Parad Bhasma - Bhasma created using parad - will alter the cellular permeability, can you please share the details of the study ? Or it's just a hypothesis? Only a thought ?
Dr. Satish ji !

[1/5, 9:49 PM] Dr. Surendra A. Soni:

 मेध्य रसायन की कार्मुकता विचारणीय आचार्य ।

[1/5, 9:53 PM] Vaidyaraj Subhash Sharma: 

*अगर आप 20 गुणों का ध्यानपूर्वक अध्ययन करें तो आपके सभी प्रश्नों के उत्तर आपको स्वयं ही मिल जायेंगे । व्यवायी, विकासी, सूक्ष्म, उष्ण, तीक्ष्ण पर चिंतन करें तो घृत, भल्लातक, पारावत शकृतादि भी ब्रेन बैरियर cross करते हुये मिलेंगे।*

[1/5, 9:53 PM] Dr Satish: 

Ji guruji dhanyawad
🙏🏻💐💐💐

[1/5, 10:06 PM] Prof. Giriraj Sharma: 

*सादर नमस्कार आचार्य जी,*

*आपकी जिज्ञासा में दो ह्रदय में दो अर्थ निहित है ,,,,*
*प्रथम---*
*तत्र प्राणवह द्वे तयोर्मुलं ह्रदय,,,*
*दो प्राणवह स्रोतस है तो दो ह्रदय होने चाहिए , प्रत्यक्षतः ह्रदय एक ही है ।*
*सिर्फ एक कोमा ( , ) मात्र में इस जिज्ञासा का उत्तर है*
*तत्र प्राणवहे द्वे ,*  *तयोर्मुलं ह्रदय*
*को*
*तत्र प्राणवहे,   द्वे तयोर्मुलं ह्रदय रस वाहिन्य*
*पढ़ना है बस* 

*द्वितीय*
*संभवत द्वितीय जिज्ञासा ज्यादा महत्वपूर्ण है वो है कि रसवह एवं प्राणवह का स्रोतस का मूल ह्रदय है दो स्रोतस का एक ह्रदय मूल है या दोनो अलग अलग है अर्थात दो है संभवत यह जिज्ञासा है*
*इसका उत्तर आचार्य चरक ने स्रोतस मूल  बताते हुए निर्देश कर दिया, जिसमे मेद अस्थि मज़्ज़ावह स्रोतस के मूल जिसमे मेदवह स्रोतस का मूल अस्थि एवं मज्जा का भी मूल है यथा-*

*मेदोवहानां स्रोतसां वृक्कौ मूलं वपावहनं च*
*अस्थिवहानां स्रोतसां मेदो मूलं जघनं च*
*मज्जवहानां स्रोतसामस्थीनि मूलं सन्धयश्च*
*मज्जावह स्रोतस मूल में संधि जघन वृक्क एवं वपावहन यहां वृक्क वपावहन तीन स्रोतस के मूल है*

*तीसरी जिज्ञासा/*
*ह्रदय जिस पर आचार्य ने ह्रदय , शिरो ह्रदय, तलह्र्दय  आदि का विवेचन पूर्व में कर दिया ।*

*अन्य जिज्ञासा अगर है तो उस जिज्ञासा का हल भी है आपके पास ही ,, क्योकि प्रश्न की गंभीरता में उत्तर भी निहित होता है वो सिर्फ प्रश्नकर्ता द्वारा ही समझा एवं समझाया जा सकता है ।*

सादर नमन प्रो. अरुण जी !
🌹🌹🌹🙏🏼🌹🌹🌹

[1/5, 10:09 PM] Vaidya Sanjay P. Chhajed: 

Do you want to present this study for Bhasma? 
I think you are making a mistake. 
This study is about inorganic mercury compound and it's toxic effects. No bhasma or any ayurvedic procedure involved. And moreover there is  no activity explored for intracellular/ alteration in  cell wall permeability. 
I am sorry to say, but you have not provided right study to support your claim.

[1/5, 10:09 PM] Dr. BK Mishra: 

@Dr Surendra Soni Ji
@Dr Mamta Mam

ज्वर सम्प्राप्ति का अवलोकन करने पर मुझे कुछ शास्त्रीय मिले हैं-

 *रसस्वेदवहानि स्रोतांसि पिधाय...*   च.नि. 1/20
 *स्रोतांसि रुद्ध्वा संप्राप्ताः ...* 
च.चि. 3/131
 *स्रोतासां सन्निरुद्धत्वात्...* च.चि.3/132
 *पिधाय च स्रोतांसि....* 
अ. हृ. नि. 2/4
 *पिधाय छादयित्वा...*    अरुण दत्त

इनसे पुष्टि होती है कि खवैगुण्य दोषों से भी होता है/सम्भव है। 
भ्रमनिवारण तथा ज्ञानवर्धन करने के लिये आप आचार्यों का धन्यवाद...
कायसम्प्रदाय का सदस्य होने पर गर्व अनुभव हो रहा है..

🙏🏼💐     🙏🏼💐

[1/5, 10:11 PM] Dr. BK Mishra: 

यह भी समझना उचित होगा कि क्या निदानार्थकर रोग भी ख वैगुण्य का कारण बनते है....

[1/5, 10:15 PM] Prof. Giriraj Sharma: 

*चतुर्थ जिज्ञासा* 
*संख्या जन्य जिसमे आचार्य ने अस्थि मज्जा एवं स्वेदवह का वर्णन नही किया तो वहां विध्द भाव का अभाव हेतु है साथ मे कला का वर्णन करते समय आचार्य ने मज्जा अस्थि एवं स्वेद कला का वर्णन भी नही किया जो डेवलपमेंट आयुर्वेद शारीर है जब गर्भ व्याकरण में ही नही तो धमनी शारीर में कैसे हो सकता है ।*


 *पंचम जिज्ञासा*
*धमन्य शब्द*
*सिरा धमनी विवर्जित के परिपेक्ष्य में दृढ़ता से लिख कर आचार्य धमन्य शब्द लिख दिया,,,,*
*उसके लिये* 
*धमनी एवं सिरा रचनात्मक इकाई है परन्तु धमन्य* *क्रियात्मक इकाई है वहां रचना को गौण स्वरूप में लिया गया है ,*
*इससे बड़ी बात तो यह है कि ह्रदय जो दो स्रोतसो का मूल है उसे मर्म में सिरा मर्म कहा है वहां सिरा कहा फिर सिरा विवर्जित भी कहा---*
*तो प्रधानता सरण है वहा भी क्रियात्मक इकाई मुख्य है ।*
🙏🏼🙏🏼🙏🏼🌹🙏🏼🙏🏼🙏🏼

[1/5, 10:26 PM] Prof. Giriraj Sharma: षष्ठम,,,,,,,,,

[1/5, 10:44 PM] Dr. Pawan Madan: 

प्रणाम आचार्य

अर्थात स्रोतस की चिकित्सा करते समय उसके मूल अवयव क्या है और किस भाव से बने हैं, का भी ध्यान रखें तो अधिक सुविधा जनक हो जायेगा

कोशिश करते हैं।🙏

[1/5, 10:49 PM] Vaidya Sanjay P. Chhajed: 

प्रणाम गुरुदेव, शायद हम विवर्जित शब्द को हिंदी वर्जित से तो नहीं जोड़ रहे?
धमनी, धमन्य, ध्मान आदी सभी शब्द किसी एक संस्कृत धातु के वैयाकरणीय उपाधेय  प्रतित होते हैं। कोई संस्कृत का विद्वान ही इसे सुलझा सकता है।

[1/5, 10:55 PM] Prof. Giriraj Sharma: 

सिराविकारा हि धमनी स्रोतांसि
सूत्र आचार्य ने *तत्र केचिदाहू* इस लिये ही प्रयुक्त किया है ।

[1/5, 10:59 PM] Vd. Sameer Shinde MD(K.C.) satara: 

अतिसुंदर एवं अभ्यासपूर्ण विवेचन आचार्य गिरिराज जी ।

[1/5, 10:59 PM] Prof. Giriraj Sharma: 

विवर्जित से अगर distribution स्वीकार करे तो आनन्द ही आनंद है ।
 प्रसृत शब्द स्रोतस के लिये सिरा धमनी के लिये विवर्जित डिस्ट्रीब्यूटेड ,,,,

[1/5, 11:01 PM] Dr. Surendra A. Soni: 

आचार्य गिरिराज जी !👏🏻🌹🙏🏻

[1/5, 11:02 PM] Prof. Giriraj Sharma: 

ह्रदय सिरा मर्म पर भी दृष्टिकोण दृढ़ हो जाएगा ।।
🙏🏼🙏🏼🙏🏼🌹🙏🏼🙏🏼🙏🏼

[1/5, 11:02 PM] Prof. Giriraj Sharma: 

Heart is a Vascular organ ,,,,,

[1/5, 11:03 PM] Dr. Bharat Padhar: 

विवर्जित/ वि- mfn. distributed, given Ma1rkP.

[1/5, 11:11 PM] Dr. Surendra A. Soni: 

निश्चित रूप से आचार्य जी ।

🙏🏻🌹

[1/5, 11:18 PM] Vaidyaraj Subhash Sharma: 

*आचार्य गिरिराज जी, सादर नमन वो भी ह्रदय तल से ❤️🙏 शारीर विषय को समझाने मे आप से श्रेष्ठ शायद कोई इतना बढ़ा विद्वान मिलना कठिन है। जब आप पूर्ण लय मे आते है तो कठिन से कठिन जिज्ञासा भी सरल बन जाती है ।* 👌👌👌👌


[1/5, 11:31 PM] Prof.Vd.Arun Rathi: 

*नमस्कार प्रो. गिरिराजजी*

🙏🏻🙏🏻🙏🏻

*मेरी जिज्ञासा केवल एक ही थी, प्रत्यक्ष मे दो हदय किस तरह समझे*

*एक अवयव अन्य स्त्रोतस् का भी मूल हो सकता है, इस पर जिज्ञासा या संशय का तो प्रश्न  ही नही उठता. आचार्य सुश्रुत ने कह दिया तो बात खत्म.*

*शास्त्र वचन शिरोधार्य*

*प्रत्यक्ष मे दो हदय ही समझना था, मै U G LEVEL के बच्चों को ही पढाता हूँ.*

*अब केवल आचार्य सुश्रुत व्दारा कहे 11 स्त्रोतस् मूल मे*

*तत्र ....वहे द्वे , .....*

*के जगह*

*तत्र .....वहे, द्वे ......।  पढना होगा.*

🙏🏻🙏🏻🙏🏻

[1/6, 12:02 AM] Vaidyaraj Subhash Sharma: 

*मैं प्रायोगिक वैद्य हूं और इस प्रकार के रोगी आते रहते है, मात्र ग्रन्थों को पढ़कर उत्तर देने के स्थान पर जो रोगी में मिला उसे भी साथ ले कर चलेंगे तो और अच्छा होगा जिस से हम वर्तमान काल मे जो history, हेतु और संपूर्ण लक्षण मिल रहे है, वेग कितने दिन में और कितने समय तक रहा इस पर नये सिरे से मंथन भी हो जायेगा । थोड़ा समय चाहिये इसके लिये प्रोफेसर जी ... 🌹🙏*

[1/6, 12:07 AM] Vaidyaraj Subhash Sharma: 

*आपकी दिशा और चिंतन बिल्कुल सही है और अपने students को भी इसी मार्ग पर ले कर चलेंगे तो कुछ वर्षों मे आयुर्वेद की जो नई पीढ़ी आयेगी उसका आधार भी आयुर्वेद ही होगा।*

[1/6, 12:08 AM] Prof. Giriraj Sharma: 

🙏🏼🙏🏼🙏🏼🌹🙏🏼🙏🏼🙏🏼
नमस्कार आचार्य श्री
धन्यवाद
महामहिम को महामहिम ही समझू ,,, ना
😄

[1/6, 12:48 AM] Dr. Sanjay Khedekar: 

भाई सतिश, तुम्हारे जिज्ञासापूर्ण विवेचन के लिए सलाम. रसशास्त्री में आवश्यक उत्सुकता तुम में भरपूर भी है. 
रही बात cell permeability की तो वह बढती ही या कम होती है, यह सिध्द करना बहुत आसान तो नहीं लेकीन मुश्कील भी नहीं. 

Permeability depends mainly on the electric charge and polarity of the molecule and to a lesser extent the molar mass of the molecule.
Due to the cell membrane's hydrophobic nature, small electrically neutral molecules pass through the membrane more easily than charged, large ones.

अब हर कोइ कहता है की भस्मों में nano particles होते है. यदी है तो permeability बढनी चाहिए. रही बात neutral charge की तो वह भी सिध्द हो सकता है.

यह सिद्धांत सिर्फ भस्मों को ही नहीं तो सिध्द स्नेह, आसव, क्वाथ इ. वानस्पितज कल्पों को भी लागू होती है. The preparation process actually turns particles to nano particles, which normally assimilates with the media and suitable for permeability. Rate of absorption increases with the preparation procedure (so our medicines are more depending on the quality manufacturing processes).
 
सिर्फ भस्में ही नहीं तो वनस्पतिजन्य द्रव्य भी blood brain barrier cross करती है.

यह तो हुआ सैद्धांतिक पक्ष, लेकीन प्राचीन एवं आर्वाचीन मतों को एकत्र करने से हम अधिक उलझ जाते है. हमारा सिध्दांत अधिक स्पष्ट है 
"अल्पमात्रोपयोगित्वात क्षिप्रमारोग्यदायित्वात." 
यहा तक हमें पहुचना है कैसे भी. इसलिए औषध निर्माण उत्कृष्टता हमारा ध्येय होना चाहिए, जो कभी १00% पूर्ण तो नहीं होगा लेकिन सफर रोचक होता है. रसशास्त्र या औषध निर्माण किताबों से अधिक  उसके बाहर है, यह साधना है हर रोज बढानी होगा.

जब तक उत्कृष्ट औषधी नहीं निर्माण तब तक उनका मूल्यांकन या प्रभाव हम तय नहीं कर सकते. और आयुर्वेद औषधी निर्माण व्यक्ती विशेष के अधीन होता है. इसे किसी भी system की तरह standardized नहीं किया जा सकता. क्यों की अग्नी को नियंत्रण करेंगे तो chemistry तो पूर्ण होगी लेकीन हम biochemistry से relate करते है, हमें औषध में गुण भी चाहिए, जो सिर्फ नियंत्रीत अग्नी से नहीं तो अनियंत्रित अग्नी को सुयोग्य दिशा देने से आते है. तात्पर्य यह है, दोहरे मापदंडोसे हम भटक सकते है. क्षिप्रमारोग्यदायित्वात... अल्पमात्रोपयोगित्वात बस यहीं ध्येय होना है...  अल्प मात्रा का सही अर्थ है यदी ग्रंथ में बतायी मात्रा 120 mg है तो हमारा भस्म  1 mg से भी कम मात्रा में काम करे इतना उत्तम निर्माण करने का हमारा उद्देश हो. क्षिप्रमारोग्यदायित्वात में यदि प्रहर में उपशय कहा है तो वह कुछ मिनिटों मे आ सके यह उद्देश अपेक्षित है. और हम यह कर सकते है. प्राकृतिक निसर्ग में भटकेंगें तो बहुत से प्रश्न के उत्तर मिल जाते है (औषध निर्माण संबंध), पन्सारी तो वही बेचता जिसे वह जानता नहीं.

यदी उपरोक्त सिद्धांत सिद्ध हो रहें है तो permeability भी change हो रहीं है और अपना काम भी.
बिना स्रोतस के तो यह संभव नहीं है. 

रसशास्त्र शब्द तो रसयुक्त है लेकिन उसके निर्माण की चर्चा सदैव निरस रहती है.

[1/6, 12:51 AM] Dr Satish: 

Guruji ko naman 👌🏻👌🏻👌🏻
💐💐💐

[1/6, 1:02 AM] Vaidyaraj Subhash Sharma: 

*नमस्कार डॉ सतीश जी, अपना परिचय भी दीजिये की कहां रहते हैं एवं शिक्षण क्षेत्र में हैं या चिकित्सा क्षेत्र में ?*

[1/6, 1:05 AM] Vaidyaraj Subhash Sharma: 

*काय सम्प्रदाय के माध्यम से पूरे भारत वर्ष के विद्वानों से परिचय हो गया ।*

[1/6, 1:06 AM] Dr. Sanjay Khedekar: 

आयुर्वेद औषध निर्माण की कला कोइ किसी से चुरा नहीं सकता, और न ही इसे परावर्तित इर सकते है. क्यों की इसे समझनें में ही 10-15 वर्ष लगते है, और समझने के बाद वह व्यक्ती विशेष के अधीन होती है. हर कोइ एक जैसे ही उत्तम गुणवत्तापूर्ण उत्पादन तो नहीं कर सकता. इसिलिए यह विशिष्ट से अतिविशिष्ठ होती है...🙏🏻🙏🏻

[1/6, 1:06 AM] Dr. Sanjay Khedekar: 

सादर प्रणाम गुरुजी सुभाष जी🙏🏻🙏🏻

[1/6, 1:08 AM] Dr Satish: 

प्रणाम गुरुजी 
डॉ सतीश सोलंकी (भावनगर)
MD Ayu. (RSBK) 
UG : GOVT AKHANDANAD AYURVEDIC MAHAVIDHYALAY, AHMEDABAD
PG: IPGT & RA 

Recently working as assistant professor in netra chikitsa ayurvedic College, amreli, Gujarat 
💐💐💐🙏🏻🙏🏻🙏🏻

[1/6, 1:08 AM] Vaidyaraj Subhash Sharma: 

*आपको भी नमस्कार आचार्य संजय जी 🌹🙏 इस समय तीनो स अक्षर के कुंभ राशि वाले एक साथ जाग रहे हैं 🤣🤣🤣*

[1/6, 1:10 AM] Vaidyaraj Subhash Sharma: 

*हार्दिक प्रसन्नता हुई 🌹🙏 हमारे सुरेन्द्र सोनी जी भी यहीं है।*


[1/6, 1:13 AM] Prof.Vd.Arun Rathi: 

*मेरा सौभाग्य है।*
बहोत दिनों आयुर्वेद की संहिताओं को पढ रहा हूँ, विगत दो तिन दिनों से

[1/6, 6:30 AM] Vaidya Sanjay P. Chhajed: 

There are multiple studies on bhasma including suvarna and hirak  being a nano medicine. You can search VD. Yogesh Bendale, and find many. The bhasma can permeate through interstitial membranes or sometimes cell membrane also, but it alters the permeability of cell wall is a non supported tall claim. We should restrain ourselves from making unsupported claims. That creates negative notion. It can be our idea that it might be working with that principle or can be an hypothesis. It is always better to state that way. It is what I feel as a student of Ayurved. 
If you recall someone questioned the basic integrity of my method of Nadi Parikshaa and training I provide. As I refused to accept and make the claims which are non supported by studies and work of my students. 

The entire discussion on strotas began with my hypothetical question, which I felt during study of Nadi Parikshaa, *whether  capillaries, the integral part of circulatory system, equates the concept of strotas ?* I posed my question with some supporting information. 
But I never knew that there is such a huge and vivid information, discussion, thought process about these concepts. I had a very limited reading and so limited thoughts. 
I don't deny that bhasma action might do the things which are beyond  our imagination, but my humble request is unless it is explored & documented,  we  as a student of Ayurved must restrain ourselves before making the statement.

[1/6, 6:37 AM] Vaidya Sanjay P. Chhajed: 

गुरुदेव हम कहां इतना संस्कृत या ग्रंथों को जानते हैं, हमने तो डिक्शनरी में मिला वह सामने रखा है। शब्द कल्पद्रुम में हम खोज नहीं पाये।  हमें जो महसूस हुआ वह स्पष्ट रूप से लिखा, इसी अपेक्षा से की संस्कृत और आयुर्वेद विद्वान इसपर टिपण्णी करें। हम कुछ मानते नहीं है। इतना शास्त्र को पढा नहीं। क्लिनिक चलाते हैं, रूग्णानुभव थोड़ा बहुत है।

गुरुदेव ! शायद इसलिए जब ह्रदय तल से आभार व्यक्त करना होता है तब "🙏" दोनो तल ह्रदयों को जोड़कर अभिवादन किया जाता है। 😇😄😇✌️✌️✌️👍👍

[1/6, 6:46 AM] Dr. Shashi: 

Good morning Prof giriraj sir, Heart development refers to the prenatal development of the heart. This begins with the formation of two endocardial tubes which merge to form the tubular heart, also called the primitive heart tube. The heart is the first functional organ in vertebrate embryos. Wikipedia
May this be the reason ?

[1/6, 6:47 AM] Dr. Mamata Bhagwat Ji: 

Pranam subhash sir, 
Shubha prabhat. 
This jijnasa is in mind sir. At your leisure and liberty sir🙏🏻🙏🏻

[1/6, 6:50 AM] Vaidya Sanjay P. Chhajed: 

बिल्कुल सही बात, हमारा मात्र यही कहना था की जब हम किसी मापदंड का निर्देश करते हैं तब वह सिद्ध होने चाहिए उन्हीं मापदंडों पर। तबतक हमारी परीभाषा तो है ही। अब अगर हमने नाड़ी परीक्षा द्वारा  हार्मोन / इलेक्ट्रोलाइट की बात की तो वह पहले सिद्ध होनी चाहिए। सिद्ध तो दोषों का गुणात्मक परिवर्तन है, उसपर बात करेंगे तो प्रश्न भी नहीं उठेंगे।

[1/6, 6:51 AM] Dr. Mamata Bhagwat Ji: 

I am trying to sir. 

Whatever i try to teach, even the very basic of Dosha dhatu or kriyakala my efforts will be on to make it applicable on patients. 
Chikitsa is the ultimate goal of learning everything by any means 🙏🏻🙏🏻

[1/6, 6:54 AM] Vaidya Sanjay P. Chhajed: 

हम भी *स*  अक्षर वाले हैं पर सिंह राशी से होने से रात्रि में नहीं सबेरे कार्यरत रहते हैं।

[1/6, 7:01 AM] Vaidya Sanjay P. Chhajed: 

*कार्यं धातुसाम्यमिहोच्यते|*
*धातुसाम्यक्रिया चोक्ता तन्त्रस्यास्य प्रयोजनम्*

यही तो हम सब वैद्यों का निहीत कर्त्तव्य है।

[1/6, 7:23 AM] Vd DARSHAN KUMAR PARMAR: 

U had open the chief desires of a Rasa Shastri👌😇
 That's why ..
Multiple folds of Puta to achieve the desire is there....

Also multiple times Paka in sneha Kalpana, multi times distillation in asava-arishta is....
Dr. Satish & Dr.  Khedekarji !

[1/6, 7:36 AM] Vaidya Sanjay P. Chhajed: 

Multiple distillation in Asava / arishta? Can you explain this.

[1/6, 7:41 AM] Vd DARSHAN KUMAR PARMAR: 

In Desire of doing more potent drug delivery systems in water & self generated alcohol base...

Asava - arishta can be utilised to prepare more potent after multi times distillation done & that distillate can also kept within its same content herbs....

[1/6, 8:05 AM] Vaidya Sameer Shinde
MD(K. C. ) satara:

 नमस्ते
पुनः पाक (जैसे शतपाकी बला तैल) या पुनः पुनः पुट से (शतपुटी अभ्रक) जरूर औषधी की कार्यकारी शक्ती वर्धन होती है साथमे मात्रा भी कम लगती है।

आसव के मामले मे तो यही ज्ञात था की जितना पुराना आसव उतनी ज्यादा कार्यकारी।
🙏🙏🙏

[1/6, 8:55 AM] Dr Mansukh R Mangukiya Gujarat: 

🙏 नमस्कार गुरुजनों को 🙏
" मर्दनं गुणवर्धनं "
यह सिद्धांत पर महाराष्ट्र के वैद्यो औषधि को मर्दन करके सुक्ष्मिकरण करते हैं। शत पुटी या सहस्त्र पुटी भस्म की जगह और कम मात्रा में इच्छित परिणाम प्राप्ति होती है।

[1/6, 9:28 AM] Vaidya Sanjay P. Chhajed: 

I think asava and arishtas are never distilled. It is the Arka which is distilled. 
Arka are never multy distilled but the amount extracted is counted. This is well explained by Subhash sir while explaining the Gulab jal/ arka.

Oils are multy paki but never heard of distilled asav arishtas. 

I am not an expert in drug manufacturing. Bheshaj kalpana experts or those who make it can throw some light.

May I take this opportunity to request khedekar ji or dwivedi sir to comment?

[1/6, 9:37 AM] वैद्य मेघराज पराडकर गोवा: 

ककाराष्टक से स्रोतस के द्वार बंद हो जाते है, इसका क्या आधार है आचार्य सतीश जी ?

[1/6, 9:45 AM] Dr. D. C. Katoch sir: 

Critical issue of serious concern for practitioners is whether Shatputi Sahsraputi bhasmas, Shatpaki oils/ghritas, puran asava-arishts etc available in the market are actually manufactured according to their labeled claims. Many times in my official capacity  I have come across incidences of manufacturing and Government supply of such Ayurvedic medicines in much lesser time than the required time of their pharmaceutical processing.

[1/6, 9:53 AM] Prof. Lakshmikant Dwivedi Sir: 

अल्पमात्रोपयोगित्वात बस यहीं ध्येय   अल्प मात्रा का सही अर्थ है *यदि ग्रंथ में बतायी मात्रा 120 mg है तो हमारा भस्म  1 mg से भी कम मात्रा में काम करे इतना उत्तम निर्माण करने का हमारा* उद्देश हो.। 👆☺️🙏डा.संजय जी✓✓✓ 
किन्तु उद्देश्य पूरा करने केलिए--
 *"**यथा *स्याज्जरणा* बह्वी तथा *स्यद्गुणदो रस:।*** 
य,  गंधक जारण मात्र के लिए नहीं।
जैसा पाठ्यक्रम में व्यवहार में लाते हैं।
अतिरिक्त जानकारी मात्र परिभाषा (बालानां (नवागन्तुक)/ मूर्खाणां) सुखबोधाय पढ़ते पढ़ते हैं।
निश्चित रूप से व्याधि निवारण ,एवं रसायन गुण प्राप्ति के लिए,
भूचरी व खेचरी जारणा के लिए उपयुक्त उद्घोष किया जाता है।
इसके व्यवहार में दक्षता से लाना है ।
जो श्रम/समय/अर्थ साध्य है।
आप के विचारों की पुष्टि के लिए
 *रसार्णव व रस योग सागर* के क्रमश: ," *नवचित्रार्थदा गुटिका* " व " *क्षेत्रीकरण रस"* । ये प्रक्रिया
कितने दक्षता से करने के लिए सक्षम हैं ? पहले प्रक्रिया कर लेते थे।
अब योग सूत्र को मात्र पढ़ कर, उसके अनुवाद मात्र, को पढ़ कर भैषज विनिर्माण करते देखा है, शोधकर्ताओं में भी।
योगसूत्र प्रसिद्ध है तभी तो  युगानुकुल प्रतिसंस्कार के समय समादृत किया, समकालीन साहित्य में।
वो योगसूत्र, असं, अहृ, वृ.संहिता, रहृत, रसार्णवादि रस ग्रंथ में उपलब्ध मानक विनिर्माण विधान भी विहित है।जो ग्रंथ मौलिक न हो कर संग्रह ग्रंथ है अथवा अपूर्ण उपलब्ध है,
*"शिलाजतु क्षौद्र विड़ंगसर्पिलोहाभया पारदताप्य भक्ष:।।
आपूर्यते दुर्बल देह धातून् त्रिपंचरात्रेण यथा शशांक:।। 
"(सैकविंशतिरहानि पा०)।
मानक विनिर्माण विधान को अनुगमन करने वाले बिरले (रसाचार्य) हों

🙏☺️

[24/03, 00:19] Vaidyaraj Subhash Sharma:

*दोष स्वयं दूषित हो कर अन्य धातुओं को दूषित करते हैं इनके दूषित होने को कैसे समझें ? पहले पित्त का उदाहरण लेते हैं चिकित्सा के दृष्टिकोण से clinical practice में हमें पित्त तीन स्वरूप में प्राप्त होता है, साम, विदग्ध और निराम पित्त ...*

*1- साम पित्त - 

इस पित्त का एक विशिष्ट प्रत्यात्म लिंग है जो शेष पित्त से इसे प्रथम दृष्टि में ही पृथक करता है 

'दुर्गन्धमसितं पित्तं कटुकं बहलं गुरु' 

अ स सू 21/18 

वह है दुर्गंध । यह पित्त प्रत्यक्ष मल भूत अपक्व रूप में  देखने को मिलता है जो हरित एवं श्याव वर्ण का हो सकता है, अम्लरसाधिक्य, गुरूता के साथ, अम्लोद्गार युक्त, कंठ और ह्रदय में दाह के साथ तथा स्थिर हो जाता है।*

*छोटे बच्चों में दंतोद्गम काल में मल के साथ उपरोक्त वर्ण में यह पित्त प्रायः मल के साथ निकलता देखा जाता है, पाण्डु रोग में हरित पीत वर्ण, वाहनादि में यात्रा के समय, अनेक स्त्रियों में गर्भावस्था काल में कुछ विशिष्ट द्रव्यों में अरूचि के कारण भी यह बहुत देखने को मिलता है।*

*2- विदग्ध पित्त -

'विदग्धन्तु तद् अम्लिका कण्ठह्ददाहकृत्' 

यह अधिक अम्लता को प्राप्त कर ह्रदय प्रदेश और कण्ठ में दाह करता है और इसकी उद्गार भी अम्लीय होती है। विभिन्न समारोह में दिन में गोलगप्पे, टिक्की, पावभाजी, chinese food, दाल मखनी शाही पनीर, पूड़ी, कचौड़ी, छोले भठूरे और ऊपर से मिष्ठान सेवन कर शैय्या पर चले जाना के सहस्त्रों रोगी आप अपने आस पास प्रतिदिन इस स्थिति से ग्रसित देख सकते है । अगर दुर्गंध हो तो इसे सामयुक्त मानें ।*

*3- निराम पित्त - 

यह कुछ ताम्र वर्ण का, उष्ण, 

'रसे कटुकमस्थिरम्' 

रस में कटु और स्थिर, गंधहीन, भोजन में रूचि उत्पन्न करता है तथा भुक्त अन्न का पाचन करता है और साथ ही बल की वृद्धि करता है।*

*पित्त प्रकोप या पित्त वृद्धि वह साम हो या विदग्ध हमें इन हेतुओं से मिलती है...*
*शरद, वर्षा और ग्रीष्म ऋतु, मध्याह्न काल और मध्य रात्रि*
*आहार की विदाह अवस्था*
*क्रोध, उपवास, भय और ईर्ष्या*
*उष्ण,तीक्ष्ण गुण*
*अम्ल, लवण और कटु रस*
*स्त्री प्रसंग या मैथुन*
*विभिन्न क्षार*
*वेगावरोध*
*अनेक औषध एवं रसायनिक द्रव्य *

*मलों एवं आर्तव के स्रोतस या दुष्टि का निर्णय इन सिद्धान्तों के अनुसार प्राप्त दुष्टि या रोगों के अनुसार करें, मल लोम केश श्मश्रु तथा उत्पादक धातु*

*'स्यात् किट्टं केशलोमास्थ्नो ...' च च 15/19*

*'तत्र हि नखलोम च इत्येनेनास्थिमलत्वं नखस्योक्तं

 .. सु सू 46/527 

'किट्टात् केशनखादय: पुष्यन्ति..' 

च सू 28/4 पर चक्रपाणि।*

*आयुर्वेदानुसार केश, लोम श्मश्रु (मूंछ और दाढ़ी) अस्थि के मल है जबकि आधुनिक मतानुसार इनके रोगों को skin disorder में लिया जाता है।*

*अस्थिक्षय के लक्षण-
 
'केशलोमनखश्मश्रु द्विजप्रपतनं श्रम:, ज्ञेयमस्थिक्षये ...' 
च सू 17/67 

अस्थि क्षय होने पर केश, लोम और श्मश्रु का पतन ।*
* अस्थि वृद्धि पर डल्हंण ने सु सू 15/14 में

 'चकारात् केशनंखयोरतिवृद्धिर्ज्ञेया' 

केश की वृद्धि भी बताई है। 

च सू 28/16 में 

'केशलोमनखश्मश्रु दोषाश्चास्थिप्रदोषजा:।'

 केश, लोम और श्मश्रु अस्थि प्रदोषज विकार बताये हैं।*

*आर्तव-

*'रसादेव रज: स्त्रींणां मासि मासि त्रयहं स्रवेत' 

सु सू 14 

आर्तव की उत्पत्ति रस से होती है, 

'मासेन रस: शुक्रं स्त्रीणां चार्तवं भवति ...' 

सु सू 14/14 

इस पर डल्हण टीका करते हैं कि 

'मासेनार्तवस्य भवनमुपचयोऽभिप्रेत: प्रकाशश्च, तस्मादार्तवस्य रक्तवत् सप्ताहेनैवोत्पत्तिरिति' 

सर्व शरीर में भ्रमण करने वाले रक्त से आर्तव का रक्त भिन्न है और इस आर्तव का निर्माण एक मास मे होता है।*

*फिर भी कोई शंका है तो निःसंकोच लिख सकते हैं।*

[24/03, 00:50] Vaidyaraj Subhash Sharma: 

*रोग के विषय में कहा गया है कि रोग सभी त्रिदोषज होते हैं, वो जो दोष संचित हुआ था ना वो प्रकुपित हो अन्य दोषों को साथ ले कर चला और कहीं तो रूकेगा ही और जहां रूकेगा अर्थात स्थान संश्रय लेगा वहां अपने लक्षण पूर्ण रूप से व्यक्त करने लगेगा।*










******************************************************************************************************************************************************************


Above discussion held on 'Kaysampraday" a Famous WhatsApp-discussion-group  of  well known Vaidyas from all over the India. This Megha-discussion held in December 2021 & January 2022. It is delayed to be published due to various reasons. 

*****************************************************************************************************************************

Compiled & Uploaded by

Vd. Rituraj Verma
B. A. M. S.
ShrDads aji Ayurveda & Panchakarma Center,
Khandawa, M.P., India.
Mobile No.:-
 +91 9669793990,
+91 9617617746

Edited by

Dr.Surendra A. Soni

M.D.,PhD (KC) 
Professor & Head
P.G. DEPT. OF KAYACHIKITSA
Govt. r Ayurveda College
Ahmedabad, GUJARAT, India.
Email: kayachikitsagau@gmail.com
 

Comments

Popular posts from this blog

Case-presentation : 'Pittashmari' (Gall-bladder-stone) by Vaidya Subhash Sharma

[1/20, 00:13] Vd. Subhash Sharma Ji Delhi:  1 *case presentations -  पित्ताश्य अश्मरी ( cholelithiasis) 4 रोगी, including fatty liver gr. 3 , ovarian cyst = संग स्रोतोदुष्टि* *पित्ताश्य अश्मरी का आयुर्वेद में उल्लेख नही है और ना ही पित्ताश्य में gall bladder का, आधुनिक चिकित्सा में इसकी औषधियों से चिकित्सा संभव नही है अत: वहां शल्य ही एकमात्र चिकित्सा है।* *पित्ताश्याश्मरी कि चिकित्सा कोई साधारण कार्य नही है क्योंकि जिस कार्य में शल्य चिकित्सा ही विकल्प हो वहां हम औषधियों से सर्जरी का कार्य कर रहे है जिसमें रोगी लाभ तो चाहता है पर पूर्ण सहयोग नही करता।* *पित्ताश्याश्मरी की चिकित्सा से पहले इसके आयुर्वेदीय दृष्टिकोण और गर्भ में छुपे  सूत्र रूप में मूल सिद्धान्तों को जानना आवश्यक है, यदि आप modern पक्ष के अनुसार चलेंगें तो चिकित्सा नही कर सकेंगे,modern की जरूरत हमें investigations और emergency में शूलनाशक औषधियों के रूप में ही पड़ती है।* *पित्ताश्याशमरी है तो पित्त स्थान की मगर इसके निदान में हमें मिले रोगियों में मुख्य दोष कफ है ...* *गुरूशीतमृदुस्निग्ध मधुरस्थिरपि

Case presentation: Vrikkashmari (Renal-stone)

On 27th November 2017, a 42 yrs. old patient came to Dept. of Kaya-chikitsa, OPD No. 4 at Govt. Ayu. College & Hospital, Vadodara, Gujarat with following complaints...... 1. Progressive pain in right flank since 5 days 2. Burning micturation 3. Dysuria 4. Polyuria No nausea/vomitting/fever/oedema etc were noted. On interrogation he revealed that he had h/o recurrent renal stone & lithotripsy was done 4 yrs. back. He had a recent 5 days old  USG report showing 11.5 mm stone at right vesicoureteric junction. He was advised surgery immediately by urologist. Following management was advised to him for 2 days with informing about the possibility of probable emergency etc. 1. Just before meal(Apankal) Ajamodadi choorna     - 6 gms. Sarjika kshar                - 1 gm. Muktashukti bhasma    - 250 mgs. Giloyasattva                 - 500 mgs. TDS with Goghrita 20 ml. 2. After meal- Kanyalohadi vati     - 2 pills Chitrakadi vati        -  4 p

Case-presentation: Management of Various Types of Kushtha (Skin-disorders) by Prof. M. B. Gururaja

Admin note:  Prof. M.B. Gururaja Sir is well-known Academician as well as Clinician in south western India who has very vast experience in treatment of various Dermatological disorders. He regularly share cases in 'Kaysampraday group'. This time he shared cases in bulk and Ayu. practitioners and students are advised to understand individual basic samprapti of patient as per 'Rogi-roga-pariksha-vidhi' whenever they get opportunity to treat such patients rather than just using illustrated drugs in the post. As number of cases are very high so it's difficult to frame samprapti of each case. Pathyakram mentioned/used should also be applied as per the condition of 'Rogi and Rog'. He used the drugs as per availability in his area and that to be understood as per the ingredients described. It's very important that he used only 'Shaman-chikitsa' in treatment.  Prof. Surendra A. Soni ®®®®®®®®®®®®®®®®®®®®®®® Case 1 case of psoriasis... In this

WhatsApp Discussion Series: 24 - Discussion on Cerebral Thrombosis by Prof. S. N. Ojha, Prof. Ramakant Sharma 'Chulet', Dr. D. C. Katoch, Dr. Amit Nakanekar, Dr. Amol Jadhav & Others

[14/08 21:17] Amol Jadhav Dr. Ay. Pth:  What should be our approach towards... Headache with cranial nerve palsies.... Please guide... [14/08 21:31] satyendra ojha sir:  Nervous System Disorders »  Neurological Disorders Headache What is a headache? A headache is pain or discomfort in the head or face area. Headaches vary greatly in terms of pain location, pain intensity, and how frequently they occur. As a result of this variation, several categories of headache have been created by the International Headache Society (IHS) to more precisely define specific types of headaches. What aches when you have a headache? There are several areas in the head that can hurt when you have a headache, including the following: a network of nerves that extends over the scalp certain nerves in the face, mouth, and throat muscles of the head blood vessels found along the surface and at the base of the brain (these contain delicate nerve fibe

WhatsApp Discussion Series 47: 'Hem-garbh-pottali-ras'- Clinical Uses by Vd. M. Gopikrishnan, Vd. Upendra Dixit, Vd. Vivek Savant, Prof. Ranjit Nimbalkar, Prof. Hrishikesh Mhetre, Vd. Tapan Vaidya, Vd. Chandrakant Joshi and Others.

[11/1, 00:57] Tapan Vaidya:  Today morning I experienced a wonderful result in a gasping ILD pt. I, for the first time in my life used Hemgarbhpottali rasa. His pulse was 120 and O2 saturation 55! After Hemgarbhapottali administration within 10 minutes pulse came dwn to 108 and O2 saturation 89 !! I repeated the Matra in the noon with addition of Trailokyachintamani Rasa as advised by Panditji. Again O2 saturation went to 39 in evening. Third dose was given. This time O2  saturation did not responded. Just before few minutes after a futile CPR I hd to declare him dead. But the result with HGP was astonishing i must admit. [11/1, 06:13] Mayur Surana Dr.:  [11/1, 06:19] M gopikrishnan Dr.: [11/1, 06:22] Vd.Vivek savant:         Last 10 days i got very good result of hemgarbh matra in Aatyayik chikitsa. Regular pt due to Apathya sevan of 250 gm dadhi (freez) get attack asthmatic then get admitted after few days she adm

UNDERSTANDING THE DIFFERENTIATION OF RAKTAPITTA, AMLAPITTA & SHEETAPITTA

UNDERSTANDING OF RAKTAPITTA, AMLAPITTA  & SHEETAPITTA  AS PER  VARIOUS  CLASSICAL  ASPECTS MENTIONED  IN  AYURVEDA. Compiled  by Dr. Surendra A. Soni M.D.,PhD (KC) Associate Professor Head of the Department Dept. of Kaya-chikitsa Govt. Ayurveda College Vadodara Gujarat, India. Email: surendraasoni@gmail.com Mobile No. +91 9408441150

DIFFERENCES IN PATHOGENESIS OF PRAMEHA, ATISTHOOLA AND URUSTAMBHA MAINLY AS PER INVOLVEMENT OF MEDODHATU

Compiled  by Dr.Surendra A. Soni M.D.,PhD (KC) Associate Professor Dept. of Kaya-chikitsa Govt. Ayurveda College Vadodara Gujarat, India. Email: surendraasoni@gmail.com Mobile No. +91 9408441150

Case-presentation: 'रेवती ग्रहबाधा चिकित्सा' (Ayu. Paediatric Management with ancient rarely used 'Grah-badha' Diagnostic Methodology) by Vd. Rajanikant Patel

[2/25, 6:47 PM] Vd Rajnikant Patel, Surat:  रेवती ग्रह पीड़ित बालक की आयुर्वेदिक चिकित्सा:- यह बच्चा 1 साल की आयु वाला और 3 किलोग्राम वजन वाला आयुर्वेदिक सारवार लेने हेतु आया जब आया तब उसका हीमोग्लोबिन सिर्फ 3 था और परिवार गरीब होने के कारण कोई चिकित्सा कराने में असमर्थ था तो किसीने कहा कि आयुर्वेद सारवार चालू करो और हमारे पास आया । मेने रेवती ग्रह का निदान किया और ग्रह चिकित्सा शुरू की।(सुश्रुत संहिता) चिकित्सा :- अग्निमंथ, वरुण, परिभद्र, हरिद्रा, करंज इनका सम भाग चूर्ण(कश्यप संहिता) लेके रोज क्वाथ बनाके पूरे शरीर पर 30 मिनिट तक सुबह शाम सिंचन ओर सिंचन करने के पश्चात Ulundhu tailam (यह SDM सिद्धा कंपनी का तेल है जिसमे प्रमुख द्रव्य उडद का तेल है)से सर्व शरीर अभ्यंग कराया ओर अभ्यंग के पश्चात वचा,निम्ब पत्र, सरसो,बिल्ली की विष्टा ओर घोड़े के विष्टा(भैषज्य रत्नावली) से सर्व शरीर मे धूप 10-15मिनिट सुबज शाम। माता को स्तन्य शुद्धि करने की लिए त्रिफला, त्रिकटु, पिप्पली, पाठा, यस्टिमधु, वचा, जम्बू फल, देवदारु ओर सरसो इनका समभाग चूर्ण मधु के साथ सुबह शाम (कश्यप संहिता) 15 दिन की चिकित्सा के वाद

Case-presentation- Self-medication induced 'Urdhwaga-raktapitta'.

This is a c/o SELF MEDICATION INDUCED 'Urdhwaga Raktapitta'.  Patient had hyperlipidemia and he started to take the Ayurvedic herbs Ginger (Aardrak), Garlic (Rason) & Turmeric (Haridra) without expertise Ayurveda consultation. Patient got rid of hyperlipidemia but hemoptysis (Rakta-shtheevan) started that didn't respond to any modern drug. No abnormality has been detected in various laboratorical-investigations. Video recording on First visit in Govt. Ayu. Hospital, Pani-gate, Vadodara.   He was given treatment on line of  'Urdhwaga-rakta-pitta'.  On 5th day of treatment he was almost symptom free but consumed certain fast food and symptoms reoccurred but again in next five days he gets cured from hemoptysis (Rakta-shtheevan). Treatment given as per availability in OPD Dispensary at Govt. Ayurveda College hospital... 1.Sitopaladi Choorna-   6 gms SwarnmakshikBhasma-  125mg MuktashuktiBhasma-500mg   Giloy-sattva-                500 mg.  

WhatsApp Discussion Series:18- "Xanthelasma" An Ayurveda Perspective by Prof. Sanjay Lungare, Vd. Anupama Patra, Vd. Trivendra Sharma, Vd. Bharat Padhar & others

[20/06 15:57] Khyati Sood Vd.  KC:  white elevated patches on eyelid.......Age 35 yrs...no itching.... no burning.......... What could be the probable diagnosis and treatment according Ayurveda..? [20/06 16:07] J K Pandey Dr. Lukhnau:  Its tough to name it in ayu..it must fall pakshmgat rog or wartmgat rog.. bt I doubt any pothki aklinn vartm aur klinn vartm or any kafaj vydhi can be correlated to xanthelasma..coz it doesnt itch or pain.. So Shalakya experts may hav a say in ayurvedic dignosis of this [20/06 16:23] Gururaja Bose Dr:  It is xantholesma, some underline liver and cholesterol pathology will be there. [20/06 16:28] Sudhir Turi Dr. Nidan Mogha:  Its xantholesma.. [20/06 16:54] J K Pandey Dr. Lukhnau:  I think madam khyati has asked for ayur dignosis.. [20/06 16:55] J K Pandey Dr. Lukhnau:  Its xanthelasma due to cholestrolemia..bt here we r to diagnose iton ayurvedic principles [20/06 17:12] An